INCLUDES
Course framework
Instructional
section
Sample exam
questions
AP
®
Human
Geography
COURSE AND EXAM DESCRIPTION
Effective
Fall 2020
AP COURSE AND EXAM DESCRIPTIONS ARE UPDATED PERIODICALLY
Please visit AP Central (apcentral.collegeboard.org) to determine whether a
more recent course and exam description is available.
AP
®
Human
Geography
COURSE AND EXAM DESCRIPTION
Eective
Fall 2020
About College Board
College Board is a mission-driven not-for-prot organization that connects
students to college success and opportunity. Founded in 1900, College Board was
created to expand access to higher education. Today, the membership association
is made up of over 6,000 of the world’s leading educational institutions and is
dedicated to promoting excellence and equity in education. Each year, College
Board helps more than seven million students prepare for a successful transition to
college through programs and services in college readiness and college success
including the SAT® and the Advanced Placement® Program. The organization also
serves the education community through research and advocacy on behalf of
students, educators, and schools.
For further information, visit collegeboard.org.
AP Equity and Access Policy
College Board strongly encourages educators to make equitable access a guiding
principle for their AP programs by giving all willing and academically prepared
students the opportunity to participate in AP. We encourage the elimination
of barriers that restrict access to AP for students from ethnic, racial, and
socioeconomic groups that have been traditionally underrepresented. Schools
should make every eort to ensure their AP classes reect the diversity of their
student population. College Board also believes that all students should have
access to academically challenging coursework before they enroll in AP classes,
which can prepare them for AP success. It is only through a commitment to
equitable preparation and access that true equity and excellence can be achieved.
Designers: Sonny Mui and Bill Tully
© 2020 College Board. College Board, Advanced Placement, AP, AP Central, and the acorn logo are
registered trademarks of College Board. All other products and services may be trademarks of their
respective owners.
Visit College Board on the Web: collegeboard.org.
Contents
v Acknowledgments
1 About AP
4 AP Resources and Supports
6 Instructional Model
7 About the AP Human Geography Course
7 College Course Equivalent
7 Prerequisites
COURSE FRAMEWORK
11 Introduction
12 Course Framework Components
13 Course Skills
15 Course Content
18 Course at a Glance
21 Unit Guides
22 Using the Unit Guides
25 UNIT 1: Thinking Geographically
37 UNIT 2: Population and Migration Patterns and Processes
55 UNIT 3: Cultural Patterns and Processes
69 UNIT 4: Political Patterns and Processes
85 UNIT 5: Agriculture and Rural Land-Use Patterns and Processes
103 UNIT 6: Cities and Urban Land-Use Patterns and Processes
119 UNIT 7: Industrial and Economic Development Patterns
andProcesses
INSTRUCTIONAL APPROACHES
135 Selecting and Using Course Materials
139 Instructional Strategies
143 Developing the Course Skills
EXAM INFORMATION
155 Exam Overview
160 Sample Exam Questions
SCORING GUIDELINES
171 Question 1: No stimulus
174 Question 3: Two stimuli
THIS PAGE IS INTENTIONALLY LEFT BLANK.
Acknowledgments
Much of the work done on this and previous editions was informed by the
valuable input of Curriculum Development and Assessment committees.
College Board would like to thank Allison Hunt, duPont Manual High School,
Louisville, KY, for her assistance with the development of this curriculum.
College Board Sta
Erica T. Appel, Associate Director, AP Curricular Publications
John C. Baran, Jr., Director, AP Instructional Design and PD Resource
Development
Cheryl Harmon, Senior Director, AP Instructional Design and
PD Resource Development
Brett Mayhan, Senior Director, AP Human Geography
Content Development
Dan McDonough, Senior Director, AP Content Integration
SPECIAL THANKS
Christopher Budano, Lawrence Charap, Krista Dornbush, and
John R. Williamson
Return to Table of Contents
© 2020 College Board
AP Human Geography Course and Exam Description  V.1
|
v
THIS PAGE IS INTENTIONALLY LEFT BLANK.
Return to Table of Contents
© 2020 College Board
About AP
College Board’s Advanced Placement® Program (AP®)
enables willing and academically prepared students
to pursue college-level studies—with the opportunity
to earn college credit, advanced placement, or
both—while still in high school. Through AP courses
in 38 subjects, each culminating in a challenging
exam, students learn to think critically, construct solid
arguments, and see many sides of an issue—skills
that prepare them for college and beyond. Taking AP
courses demonstrates to college admission ocers
that students have sought the most challenging
curriculum available to them, and research indicates
that students who score a 3 or higher on an AP Exam
typically experience greater academic success in
college and are more likely to earn a college degree
than non-AP students. Each AP teacher’s syllabus
is evaluated and approved by faculty from some of
the nation’s leading colleges and universities, and AP
Exams are developed and scored by college faculty and
experienced AP teachers. Most four-year colleges and
universities in the United States grant credit, advanced
placement, or both on the basis of successful AP
Exam scores—more than 3,300 institutions worldwide
annually receive AP scores.
AP Course Development
In an ongoing eort to maintain alignment with best
practices in college-level learning, AP courses and
exams emphasize challenging, research-based
curricula aligned with higher education expectations.
Individual teachers are responsible for designing their
own curriculum for AP courses, selecting appropriate
college-level readings, assignments, and resources.
This course and exam description presents the content
and skills that are the focus of the corresponding
college course and that appear on the AP Exam. It also
organizes the content and skills into a series of units
that represent a sequence found in widely adopted
college textbooks and that many AP teachers have
told us they follow in order to focus their instruction.
The intention of this publication is to respect teachers’
time and expertise by providing a roadmap that they
can modify and adapt to their local priorities and
preferences. Moreover, by organizing the AP course
content and skills into units, the AP Program is able
to provide teachers and students with free formative
assessments—Personal Progress Checks—that
teachers can assign throughout the year to measure
student progress as they acquire content knowledge
and develop skills.
Enrolling Students:
EquityandAccess
College Board strongly encourages educators to
make equitable access a guiding principle for their
AP programs by giving all willing and academically
prepared students the opportunity to participate
in AP. We encourage the elimination of barriers
that restrict access to AP for students from ethnic,
racial, and socioeconomic groups that have been
traditionally underserved. College Board also believes
that all students should have access to academically
challenging coursework before they enroll in AP
classes, which can prepare them for AP success. It is
only through a commitment to equitable preparation
and access that true equity and excellence can be
achieved.
Oering AP Courses:
The AP Course Audit
The AP Program unequivocally supports the principle
that each school implements its own curriculum that will
enable students to develop the content understandings
and skills described in the course framework.
While the unit sequence represented in this publication
is optional, the AP Program does have a short list of
curricular and resource requirements that must be
fullled before a school can label a course Advanced
Placement” or AP.” Schools wishing to oer AP
courses must participate in the AP Course Audit, a
process through which AP teachers’ course materials
are reviewed by college faculty. The AP Course Audit
was created to provide teachers and administrators
with clear guidelines on curricular and resource
requirements for AP courses and to help colleges and
universities validate courses marked “AP” on students’
transcripts. This process ensures that AP teachers’
courses meet or exceed the curricular and resource
expectations that college and secondary school faculty
have established for college-level courses.
 V.1
|
1AP Human Geography Course and Exam Description
Return to Table of Contents
© 2020 College Board
The AP Course Audit form is submitted by the AP teacher
and the school principal (or designated administrator) to
conrm awareness and understanding of the curricular
and resource requirements. A syllabus or course outline,
detailing how course requirements are met, is submitted
by the AP teacher for review by college faculty.
Please visit collegeboard.org/apcourseaudit for more
information to support the preparation and submission
of materials for the AP Course Audit.
How the AP Program
IsDeveloped
The scope of content for an AP course and exam is
derived from an analysis of hundreds of syllabi and course
oerings of colleges and universities. Using this research
and data, a committee of college faculty and expert AP
teachers work within the scope of the corresponding
college course to articulate what students should know
and be able to do upon the completion of the AP course.
The resulting course framework is the heart of this course
and exam description and serves as a blueprint of the
content and skills that can appear on an AP Exam.
The AP Test Development Committees are responsible
for developing each AP Exam, ensuring the exam
questions are aligned to the course framework. The AP
Exam development process is a multiyear endeavor; all
AP Exams undergo extensive review, revision, piloting,
and analysis to ensure that questions are accurate, fair,
and valid, and that there is an appropriate spread of
diculty across the questions.
Committee members are selected to represent a variety
of perspectives and institutions (public and private,
small and large schools and colleges), and a range of
gender, racial/ethnic, and regional groups. A list of each
subject’s current AP Test Development Committee
members is available on apcentral.collegeboard.org.
Throughout AP course and exam development, College
Board gathers feedback from various stakeholders
in both secondary schools and higher education
institutions. This feedback is carefully considered to
ensure that AP courses and exams are able to provide
students with a college-level learning experience and
the opportunity to demonstrate their qualications for
advanced placement or college credit.
How AP Exams Are Scored
The exam scoring process, like the course and exam
development process, relies on the expertise of
both AP teachers and college faculty. While multiple-
choice questions are scored by machine, the free-
response questions and through-course performance
assessments, as applicable, are scored by thousands
of college faculty and expert AP teachers. Most are
scored at the annual AP Reading, while a small portion
is scored online. All AP Readers are thoroughly trained,
and their work is monitored throughout the Reading
for fairness and consistency. In each subject, a highly
respected college faculty member serves as Chief
Faculty Consultant and, with the help of AP Readers
in leadership positions, maintains the accuracy of
the scoring standards. Scores on the free-response
questions and performance assessments are weighted
and combined with the results of the computer-scored
multiple-choice questions, and this raw score is
converted into a composite AP score on a 1–5 scale.
AP Exams are not norm-referenced or graded on a curve.
Instead, they are criterion-referenced, which means that
every student who meets the criteria for an AP score of
2, 3, 4, or 5 will receive that score, no matter how many
students that is. The criteria for the number of points
students must earn on the AP Exam to receive scores of
3, 4, or 5—the scores that research consistently validates
for credit and placement purposes—include:
§ The number of points successful college students
earn when their professors administer AP Exam
questions to them.
§ The number of points researchers have found to
be predictive that an AP student will succeed when
placed into a subsequent, higher-level college course.
§ Achievement-level descriptions formulated by
college faculty who review each AP Exam question.
Using and Interpreting AP Scores
The extensive work done by college faculty and AP
teachers in the development of the course and exam and
throughout the scoring process ensures that AP Exam
scores accurately represent students’ achievement in the
equivalent college course. Frequent and regular research
studies establish the validity of AP scores as follows:
AP Score
Credit
Recommendation
College Grade
Equivalent
5
Extremely well qualied A
4
Well qualied
A, B+, B
3
Qualied
B, C+, C
2
Possibly qualied n/a
1
No recommendation n/a
 V.1
|
2AP Human Geography Course and Exam Description
Return to Table of Contents
© 2020 College Board
While colleges and universities are responsible for
setting their own credit and placement policies, most
private colleges and universities award credit and/
or advanced placement for AP scores of 3 or higher.
Additionally, most states in the U.S. have adopted
statewide credit policies that ensure college credit
for scores of 3 or higher at public colleges and
universities. To conrm a specic college’s AP credit/
placement policy, a search engine is available at
apstudent.org/creditpolicies.
BECOMING AN AP READER
Each June, thousands of AP teachers and college
faculty members from around the world gather for
seven days in multiple locations to evaluate and score
the free-response sections of the AP Exams. Ninety-
eight percent of surveyed educators who took part in
the AP Reading say it was a positive experience.
There are many reasons to consider becoming an AP
Reader, including opportunities to:
§ Bring positive changes to the classroom:
Surveys show that the vast majority of returning
AP Readers—both high school and college
educators—make improvements to the way they
teach or score because of their experience at the
AP Reading.
§ Gain in-depth understanding of AP Exam and AP
scoring standards: AP Readers gain exposure to
the quality and depth of the responses from the
entire pool of AP Exam takers, and thus are better
able to assess their students’ work in the classroom.
§ Receive compensation: AP Readers are
compensated for their work during the Reading.
Expenses, lodging, and meals are covered for
Readers who travel.
§ Score from home: AP Readers have online
distributed scoring opportunities for certain
subjects. Check collegeboard.org/apreading
fordetails.
§ Earn Continuing Education Units (CEUs): AP
Readers earn professional development hours and
CEUs that can be applied to PD requirements by
states, districts, and schools.
How to Apply
Visit collegeboard.org/apreading for eligibility
requirements and to start the application process.
 V.1
|
3AP Human Geography Course and Exam Description
Return to Table of Contents
© 2020 College Board
AP Resources
andSupports
By completing a simple activation process at the start of the school year, teachers and
students receive access to a robust set of classroom resources.
AP Classroom
AP Classroom is a dedicated online platform designed to support teachers and students
throughout their AP experience. The platform provides a variety of powerful resources and
tools to provide yearlong support to teachers and enable students to receive meaningful
feedback on their progress.
UNIT GUIDES
Appearing in this publication and on AP Classroom, these planning guides outline all required
course content and skills, organized into commonly taught units. Each unit guide suggests a
sequence and pacing of content, scaolds skill instruction across units, organizes content
into topics, and provides tips on taking the AP Exam.
PERSONAL PROGRESS CHECKS
Formative AP questions for every unit provide feedback to students on the areas where they
need to focus. Available online, Personal Progress Checks measure knowledge and skills
through multiple-choice questions with rationales to explain correct and incorrect answers,
and free-response questions with scoring information. Because the Personal Progress
Checks are formative, the results of these assessments cannot be used to evaluate teacher
eectiveness or assign letter grades to students, and any such misuses are grounds for losing
school authorization to oer AP courses.*
PROGRESS DASHBOARD
This dashboard allows teachers to review class and individual student progress throughout
the year. Teachers can view class trends and see where students struggle with content and
skills that will be assessed on the AP Exam. Students can view their own progress over time to
improve their performance before the AP Exam
.
AP QUESTION BANK
This online library of real AP Exam questions provides teachers with secure questions to use
in their classrooms. Teachers can nd questions indexed by course topics and skills, create
customized tests, and assign them online or on paper. These tests enable students to practice
and get feedback on each question.
* To report misuses, please call, 877-274-6474 (International: +1-212-632-1781).
AP Human Geography Course and Exam Description  V.1
|
4
Return to Table of Contents
© 2020 College Board
Digital Activation
In order to teach an AP class and make sure students are registered to take the AP Exam,
teachers must rst complete the digital activation process. Digital activation gives students
and teachers access to resources and gathers students’ exam registration information online,
eliminating most of the answer sheet bubbling that has added to testing time and fatigue.
AP teachers and students begin by signing in to My AP and completing a simple activation
process at the start of the school year, which provides access to all AP resources, including
AP Classroom.
To complete digital activation:
§ Teachers and students sign in to, or create, their College Board accounts.
§ Teachers conrm that they have added the course they teach to their AP Course Audit
account and have had it approved by their school’s administrator.
§ Teachers or AP Coordinators, depending on whom the school has decided is responsible,
set up class sections so students can access AP resources and have exams ordered on
their behalf.
§ Students join class sections with a join code provided by their teacher or AP coordinator.
§ Students will be asked for additional registration information upon joining their rst class
section, which eliminates the need for extensive answer sheet bubbling on exam day.
While the digital activation process takes a short time for teachers, students, and AP coordinators
to complete, overall it helps save time and provides the following additional benets:
§ Access to AP resources and supports: Teachers have access to resources specically
designed to support instruction and provide feedback to students throughout the school
year as soon as activation is complete.
§ Streamlined exam ordering: AP Coordinators can create exam orders from the same
online class rosters that enable students to access resources. The coordinator reviews,
updates, and submits this information as the school’s exam order in the fall.
§ Student registration labels: For each student included in an exam order, schools will
receive a set of personalized AP ID registration labels, which replaces the AP student
pack.The AP ID connects a student’s exam materials with the registration information they
provided during digital activation, eliminating the need for pre-administration sessions and
reducing time spent bubbling on exam day.
§ Targeted Instructional Planning Reports: AP teachers will get Instructional Planning
Reports (IPRs) that include data on each of their class sections automatically rather than
relying on special codes optionally bubbled in on exam day.
 V.1
|
5AP Human Geography Course and Exam Description
Return to Table of Contents
© 2020 College Board
Instructional
Model
Integrating AP resources throughout the course can help students develop skills and
conceptual understandings. The instructional model outlined below shows possible ways to
incorporate AP resources into the classroom.
Plan
Teachers may consider the following approaches as they plan their instruction before
teaching each unit.
§ Review the overview at the start of each unit guide to identify essential questions,
conceptual understandings, and skills for each unit.
§ Use the Unit at a Glance table to identify related topics that build toward a common
understanding, and then plan appropriate pacing for students.
§ Identify useful strategies in the Instructional Approaches section to help teach the
concepts and skills.
Teach
When teaching, supporting resources could be used to build students’ conceptual
understanding and their mastery of skills.
§ Use the topic pages in the unit guides to identify the required content.
§ Integrate the content with a skill, considering any appropriate scaolding.
§ Employ any of the instructional strategies previously identied.
§ Use the available resources on the topic pages to bring a variety of assets into the classroom.
Assess
Teachers can measure student understanding of the content and skills covered in the unit and
provide actionable feedback to students.
§ At the end of each unit, use AP Classroom to assign students the online Personal
Progress Checks, as homework or an in-class task.
§ Provide question-level feedback to students through answer rationales; provide unit- and
skill-level feedback using the progress dashboard.
§ Create additional practice opportunities using the AP Question Bank and assign them
through AP Classroom.
AP Human Geography Course and Exam Description  V.1
|
6
Return to Table of Contents
© 2020 College Board
About the AP Human
Geography Course
This course introduces students to the systematic study of patterns and processes that
have shaped human understanding, use, and alteration of Earth’s surface. Students employ
spatial concepts and landscape analysis to examine socioeconomic organization and its
environmental consequences. They also learn about the methods and tools geographers use
in their research and applications. The curriculum reects the goals of the National Geography
Standards (2012).
College Course Equivalent
The AP Human Geography course is equivalent to an introductory college-level course in
human geography.
Prerequisites
There are no prerequisites for AP Human Geography. Students should be able to read college-
level texts and write grammatically correct, complete sentences.
 V.1
|
7AP Human Geography Course and Exam Description
THIS PAGE IS INTENTIONALLY LEFT BLANK.
Course
Framework
AP HUMAN GEOGRAPHY
Return to Table of Contents
© 2020 College Board
Introduction
AP Human Geography introduces high school students
to college-level introductory human geography
or cultural geography. The content is presented
thematically rather than regionally and is organized
around the discipline’s main subelds: economic
geography, cultural geography, political geography, and
urban geography. The approach is spatial and problem
oriented. Case studies are drawn from all world regions,
with an emphasis on understanding the world in
which we live today. Historical information serves to
enrich analysis of the impacts of phenomena such as
globalization, colonialism, and human–environment
relationships on places, regions, cultural landscapes,
and patterns of interaction.
Specic topics with which students engage include
the following:
§ problems of economic development and
cultural change
§ consequences of population growth, changing
fertility rates, and international migration
§ impacts of technological innovation on
transportation, communication, industrialization,
and other aspects of human life
§ struggles over political power and control
of territory
§ conicts over the demands of ethnic minorities,
the role of women in society, and the inequalities
between developed and developing economies
§ explanations of why location matters to
agricultural land use, industrial development,
and urban problems
§ the role of climate change and environmental
abuses in shaping the human landscapes on Earth
The goal for the course is for students to become
more geoliterate, more engaged in contemporary
global issues, and more informed about multicultural
viewpoints. They will develop skills in approaching
problems geographically, using maps and geospatial
technologies, thinking critically about texts and graphic
images, interpreting cultural landscapes, and applying
geographic concepts such as scale, region, diusion,
interdependence, and spatial interaction, among others.
Students will see geography as a discipline relevant
to the world in which they live; as a source of ideas for
identifying, clarifying, and solving problems at various
scales; and as a key component of building global
citizenship and environmental stewardship.
AP Human GeographyCourse and Exam Description Course Framework V.1
|
11
Return to Table of Contents
© 2020 College Board
The course framework includes two
essential components:
1
COURSE SKILLS
The course skills are central to the study and practice of human
geography. Students should have the opportunity to develop and apply
the described skills on a regular basis over the span of the course.
2
COURSE CONTENT
The course content is organized into commonly taught units of study
that provide a suggested sequence for the course. These units comprise
required content and conceptual understandings that colleges and
universities typically expect students to master to qualify for college
credit and/or placement. This content is grounded in big ideas, which are
cross-cutting concepts that build conceptual understanding and spiral
throughout the course.
Course Framework
Components
Overview
This course framework provides a description of the course requirements
necessary for student success, specifying what students must know, be able to
do, and understand to qualify forcollege credit orplacement.
AP Human GeographyCourse and Exam Description Course Framework V.1
|
12
Return to Table of Contents
© 2020 College Board
AP HUMAN GEOGRAPHY
Course
Skills
The table that follows presents the skill categories and related skills that students
should develop during the AP Human Geography course. These skills form the
basis of tasks on the AP Exam.
The unit guides that follow embed and spiral these practices throughout the
course, providing teachers with one way to integrate the skills into the course
content with sucient repetition to prepare students to transfer those skills when
taking the AP Human Geography Exam.
More detailed information about the teaching of the course skills can be found in
the Instructional Approaches section of this publication.
1
Course Framework V.1
|
13
AP Human GeographyCourse and Exam Description
Return to Table of Contents
© 2020 College Board
Skill Category 1 Skill Category 2 Skill Category 3 Skill Category 4 Skill Category 5
Concepts and
Processes
Spatial
Relationships
Data
Analysis
Source
Analysis
Scale
Analysis
Analyze geographic theories, approaches,
concepts, processes, or models in
theoretical and applied contexts.
Analyze geographic patterns, relationships,
and outcomes in applied contexts.
Analyze and interpret quantitative
geographic data represented in maps,
tables, charts, graphs, satellite images,
andinfographics.
Analyze and interpret qualitative
geographic information represented in
maps, images (e.g., satellite, photographs,
cartoons), and landscapes.
Analyze geographic theories, approaches,
concepts, processes, and models across
geographic scales to explain spatial
relationships.
1.A
Describe geographic
concepts, processes, models,
and theories.
1.B
 Explain geographic
concepts, processes, models,
and theories.
1.C
 Compare geographic
concepts, processes, models,
and theories.
1.D
 Describe a relevant
geographic concept, process,
model, or theory in a specied
context.
1.E
 Explain the strengths,
weaknesses, and limitations of
dierent geographic models and
theories in a specied context.
2.A
Describe spatial patterns,
networks, and relationships.
2.B
Explain spatial relationships
in a specied context or region
of the world, using geographic
concepts, processes, models,
ortheories.
2.C
Explain a likely outcome
in a geographic scenario
using geographic concepts,
processes, models, or theories.
2.D
Explain the signicance
of geographic similarities and
dierences among dierent
locations and/or at dierent times.
2.E
Explain the degree to which
a geographic concept, process,
model, or theory eectively
explains geographic eects in
dierent contexts and regions of
the world.
3.A
 Identify the dierent types
of data presented in maps and in
quantitative and geospatial data.
3.B
 Describe spatial patterns
presented in maps and in
quantitative and geospatial data.
3.C
 Explain patterns and trends
in maps and in quantitative
and geospatial data to draw
conclusions.
3.D
 Compare patterns
and trends in maps and in
quantitative and geospatial data
to draw conclusions.
3.E
 Explain what maps
or data imply or illustrate
about geographic principles,
processes, and outcomes.
3.F
 Explain possible limitations
of the data provided.
4.A
 Identify the dierent types
of information presented in
visual sources.
4.B
 Describe the spatial
patterns presented in visual
sources.
4.C
 Explain patterns and
trends in visual sources to draw
conclusions.
4.D
 Compare patterns and
trends in visual sources to draw
conclusions.
4.E
 Explain how maps, images,
and landscapes illustrate or
relate to geographic principles,
processes, and outcomes.
4.F
 Explain possible limitations
of visual sources provided.
5.A
 Identify the scales of
analysis presented by maps,
quantitative and geospatial data,
images, and landscapes.
5.B
 Explain spatial relationships
across various geographic
scales using geographic
concepts, processes, models, or
theories.
5.C
 Compare geographic
characteristics and processes at
various scales.
5.D
 Explain the degree to
whicha geographic concept,
process, model, or theory
eectively explains geographic
eects across various
geographic scales.
1
2
3
4
5
AP HUMAN GEOGRAPHY
Course Skills
SKILLS
Return to Table of Contents
© 2020 College Board
Course Framework V.1
|
14AP Human GeographyCourse and Exam Description
Return to Table of Contents
© 2020 College Board
AP HUMAN GEOGRAPHY
Course
Content
Based on the Understanding by Design® (Wiggins and McTighe) model, this
course framework provides a clear and detailed description of the course
requirements necessary for student success. The framework species what
students must know, understand, and be able to do, with a focus on big ideas
that encompass core principles, theories, and processes of the discipline. The
framework also encourages instruction that prepares students for advanced
geography coursework and active global citizenship.
Big Ideas
The big ideas serve as the foundation of the course and enable students to
create meaningful connections among course concepts. Often, these big ideas
are abstract concepts or themes that become threads that run throughout the
course. Revisiting the big ideas and applying them in a variety of contexts allow
students to develop a deeper conceptual understanding. Below are the big ideas
of the course and a brief description of each.
2
BIG IDEA 1: PATTERNS AND SPATIAL ORGANIZATION (PSO)
Spatial patterns and organization of human society are arranged according
to political, historical, cultural, and economic factors.
BIG IDEA 2: IMPACTS AND INTERACTIONS (IMP)
Complex relationships of cause and eect exist among people, their
environments, and historical and contemporary actions.
BIG IDEA 3: SPATIAL PROCESS AND SOCIETAL CHANGE (SPS)
A spatial perspective allows for a focus on the ways phenomena are related
to one another in particular places, which in turn allows for the examination
of human organization and its environmental consequences.
Course Framework V.1
|
15
AP Human GeographyCourse and Exam Description
Return to Table of Contents
© 2020 College Board
UNITS
The course content is organized into commonly
taught units. The units have been arranged in a logical
sequence frequently found in many college courses
and textbooks.
The seven units in AP Human Geography, and their
weightings on the multiple-choice section of the
APExam, are listed below.
Pacing recommendations at the unit level and in the
Course at Glance tables provide suggestions for how
the required course content can be taught and how the
Personal Progress Checks can be administered. The
suggested class periods are based on a schedule in
which the class meets ve days a week for 45minutes
each day. While these recommendations have been
made to aid in your planning, you are free to adjust the
pacing based on the needs of your students, alternate
schedules (e.g., block scheduling), or your school’s
academic calendar.
TOPICS
Each unit is broken down into teachable segments
called topics. The topic pages (starting on page 30)
contain all required content for each topic. Although
most topics can be taught in one or two class periods,
you are again encouraged to pace your course to suit
the needs of your students and school.
Units Exam Weighting
Unit 1: Thinking Geographically
8–10%
Unit 2: Population and Migration Patterns andProcesses
12–17%
Unit 3: Cultural Patterns and Processes
12–17%
Unit 4: Political Patterns and Processes
12–17%
Unit 5: Agriculture and Rural Land-Use Patterns andProcesses
12–17%
Unit 6: Cities and Urban Land-Use Patterns andProcesses
12–17%
Unit 7: Industrial and Economic Development Patterns andProcesses
12–17%
AP Human GeographyCourse and Exam Description Course Framework V.1
|
16
Big Ideas
Unit 1 Unit 2 Unit 3 Unit 4 Unit 5 Unit 6 Unit 7
Thinking
Geographically
Population and
Migration Patterns
and Processes
Cultural Patterns
and Processes
Political Patterns
and Processes
Agriculture and
Rural Land-Use
Patterns and
Processes
Cities and Urban
Land-Use Patterns
and Processes
Industrial and
Economic
Development
Patterns and
Processes
Patterns
and Spatial
Organization
PSO
Impacts and
Interactions
IMP
Spatial
Processes and
Societal Change
SPS
Spiraling the Big Ideas
The following table demonstrates how the big ideas spiral across units by showing the units in which each big idea
appears. In AP Human Geography students will encounter each big idea in every unit so that their knowledge, skills, and
understanding related to the big ideas become more complex.
Return to Table of Contents
© 2020 College Board
Course Framework V.1
|
17AP Human GeographyCourse and Exam Description
Course at
a Glance
Plan
The Course at a Glance
table provides a useful visual
organization of the APHuman
Geography curricular
components, including:
§ Sequence of units, along with
approximate weighting and
suggested pacing. Please
note that pacing is based
on 45-minute class periods
meeting ve days each week
for a full academic year.
§ Progression of topics within
each unit.
§ Spiraling of the big ideas and
course skills across units.
Teach
SKILL CATEGORIES
Skill categories spiral
throughout the course.
Concepts and
Processes
Spatial
Relationships
Data Analysis
Source Analysis
Scale Analysis
BIG IDEAS
Big Ideas spiral across topics and units.
Patterns and Spatial Organization
Impacts and Interactions
Spatial Processes and Societal Change
Assess
Assign the Personal Progress
Checks—either as homework or in
class—for each unit. Each Personal
Progress Check contains formative
multiple-choice and free-response
questions. The feedback from the
Personal Progress Checks shows
students the areas where they
need to focus.
1
2
3
4
5
PSO
IMP
SPS
Thinking
Geographically
UNIT
1
Personal Progress Check 1
Multiple-choice: ~20 questions
Free-response: 1 question
§ 1stimulus
Personal Progress Check 2
Multiple-choice: ~35 questions
Free-response: 1 question
§ 1stimulus
Population
and Migration
Patterns and
Processes
UNIT
2
IMP
1.1 Introduction to Maps
3
IMP
1.2 Geographic Data
3
IMP
1.3 The Power of
Geographic Data
3
PSO
1.4 Spatial Concepts
3
PSO
1.5 Human–Environmental
Interaction
1
PSO
1.6 Scales of Analysis
5
SPS
1.7 Regional Analysis
1
PSO
2.1 Population
Distribution
3
PSO
2.2 Consequences of
Population Distribution
2
PSO
2.3 Population
Composition
2
IMP
2.4 Population Dynamics
3
IMP
2.5 The Demographic
Transition Model
3
IMP
2.6 Malthusian Theory
2
SPS
2.7 Population Policies
2
SPS
2.8 Women and
Demographic Change
3
SPS
2.9 Aging Populations
2
IMP
2.10 Causes of Migration
2
IMP
2.11 Forced and Voluntary
Migration
1
IMP
2.12 Eects of Migration
2
~9–10
Class
Periods 8–10
%
AP Exam
Weighting ~1920
Class
Periods 12–17
%
AP Exam
Weighting
Personal Progress Check 5
Multiple-choice: ~35 questions
Free-response: 1 question
§ 2 stimuli
Personal Progress Check 4
Multiple-choice: ~30 questions
Free-response: 1 question
§ 1 stimulus
Personal Progress Check 3
Multiple-choice: ~25 questions
Free-response: 1 question
§ 2 stimuli
Cultural Patterns
and Processes
UNIT
3
Political Patterns
and Processes
UNIT
4
Agriculture and
Rural Land-Use
Patterns and
Processes
UNIT
5
PSO
5.1 Introduction to
Agriculture
2
PSO
5.2 Settlement Patterns and
Survey Methods
4
SPS
5.3 Agricultural Origins and
Diffusions
2
SPS
5.4 The Second
Agricultural Revolution
4
SPS
5.5 The Green Revolution
2
PSO
5.6 Agricultural Production
Regions
2
PSO
5.7 Spatial Organization
ofAgriculture
2
PSO
5.8 Von Thünen Model
5
PSO
5.9 The Global System
ofAgriculture
5
IMP
5.10 Consequences of
Agricultural Practices
2
IMP
5.11 Challenges of
Contemporary
Agriculture
4
IMP
5.12 Women in Agriculture
3
PSO
4.1 Introduction to Political
Geography
4
PSO
4.2 Political Processes
3
PSO
4.3 Political Power and
Territoriality
5
IMP
4.4 Defining Political
Boundaries
1
IMP
4.5 The Function of
Political Boundaries
5
IMP
4.6 Internal Boundaries
5
IMP
4.7 Forms of Governance
2
SPS
4.8 Defining Devolutionary
Factors
3
SPS
4.9 Challenges to
Sovereignty
5
SPS
4.10 Consequences of
Centrifugal and
Centripetal Forces
5
PSO
3.1 Introduction to Culture
4
PSO
3.2 Cultural Landscapes
4
PSO
3.3 Cultural Patterns
4
IMP
3.4 Types of Diffusion
1
SPS
3.5 Historical Causes
ofDiffusion
2
SPS
3.6 Contemporary Causes
ofDiffusion
5
IMP
3.7 Diffusion of Religion
and Language
4
SPS
3.8 Effects of Diffusion
2
~1920
Class
Periods 12–17
%
AP Exam
Weighting ~1920
Class
Periods 12–17
%
AP Exam
Weighting ~1920
Class
Periods 12–17
%
AP Exam
Weighting
Personal Progress Check 7
Multiple-choice: ~25 questions
Free-response: 1 question
§ no stimulus
Personal Progress Check 6
Multiple-choice: ~35 questions
Free-response: 1 question
§ no stimulus
Cities and
Urban Land-Use
Patterns and
Processes
UNIT
6
Industrial and
Economic
Development
Patterns and
Processes
UNIT
7
PSO
6.1 The Origin and
Influences of
Urbanization
2
PSO
6.2 Cities Across the World
2
PSO
6.3 Cities and Globalization
5
PSO
6.4 The Size and
Distribution of Cities
2
PSO
6.5 The Internal Structure
ofCities
1
IMP
6.6 Density and Land Use
3
IMP
6.7 Infrastructure
3
IMP
6.8 Urban Sustainability
2
IMP
6.9 Urban Data
3
SPS
6.10 Challenges of Urban
Changes
4
SPS
6.11 Challenges of Urban
Sustainability
2
SPS
7.1 The Industrial
Revolution
4
SPS
7.2 Economic Sectors and
Patterns
2
SPS
7.3 Measures of
Development
3
SPS
7.4 Women and Economic
Development
3
SPS
7.5 Theories of Development
1
PSO
7.6 Trade and the World
Economy
5
PSO
7.7 Changes as a Result of
the World Economy
4
IMP
7.8 Sustainable
Development
5
~1920
Class
Periods 12–17
%
AP Exam
Weighting ~1920
Class
Periods 12–17
%
AP Exam
Weighting
Developed with extensive input from the community of AP Human
Geography educators, these unit guides oer teachers helpful guidance
in building students’ skills and knowledge. The suggested sequence was
identied through a thorough analysis of the syllabi of highly eective AP
teachers and the organization of typical college textbooks.
This unit structure respects new AP teachers’ time by providing one
possible sequence they can adopt or modify, rather than having to build
from scratch. An additional benet is that these units enable the AP
Program to provide interested teachers with formative assessments—the
Personal Progress Checks—that they can assign their students at the
endof each unit to gauge progress toward success on the AP exam.
However, experienced AP teachers who are satised with their current
course organization and exam results should feel no pressure to adopt
these units, which comprise an optional sequence for this course.
AP HUMAN GEOGRAPHY
Unit
Guides
Course Framework V.1 
| 
21AP Human Geography Course and Exam Description
Return to Table of Contents
© 2020 College Board
Return to Table of Contents
© 2020 College Board
UNIT
1
Developing Understanding
This rst unit sets the foundation for the course by teaching students
how geographers approach the study of places. Students are
encouraged to reect on the “why of where” to better understand
geographic perspectives. Many other high school courses ask
students to read and analyze data, but for this course, students also
apply a spatial perspective when reading and analyzing qualitative and
quantitative data.
Students learn the ways information from data sources such as
maps, tables, charts, satellite images, and infographics informs policy
decisions such as voting redistricting or expanding transportation
networks. They also learn about how people inuence and are
inuenced by their environment; the resulting impact on topography,
natural resources, and climate; and the dierences between and
consequences of environmental determinism and possibilism.
Finally, students are introduced to the language of geography,
learning discipline-specic terminology and applying that language
to contemporary, real-world scenarios so they can better study
population processes and patterns in the next unit.
BIG IDEA 1
Patterns and Spatial
Organization
PSO
§ Why do geographers
study relationships and
patterns among and
between places?
BIG IDEA 2
Impacts and
Interactions
IMP
§ How do geographers
use maps to help
themdiscover patterns
and relationships in
theworld?
BIG IDEA 3
Spatial Processes and
Societal Change
SPS
§ How do geographers
use a spatial
perspective to analyze
complex issues
andrelationships?
Thinking
Geographically
8–10
%
 AP EX AM WEIGHTING ~9–10 CLASS PERIODS
Course Framework V.1
|
27AP Human Geography Course and Exam Description
00762-133-CED-Human-Geo_Unit 1.indd 27 3/14/19 9:49 PM
UNIT AT A GLANCE
Thinking Geographically
UNIT
1
Enduring
Understanding
Topic Suggested Skill
Class Periods
~9 –10
CLASS PERIODS
IMP-1
1.1 Introduction to Maps
3.A
 Identify the dierent types of data presented
in maps and in quantitative and geospatial data.
1.2 Geographic Data
3.A
 Identify the dierent types of data presented
in maps and in quantitative and geospatial data.
1.3 The Power of
GeographicData
3.B
 Describe spatial patterns presented in maps
and in quantitative and geospatial data.
PSO-1
1.4 Spatial Concepts
3.B
 Describe spatial patterns presented in maps
and in quantitative and geospatial data.
1.5 Human–Environmental
Interaction
1.B
 Explain geographic concepts, processes,
models, and theories.
1.6 Scales of Analysis
5.A
 Identify the scales of analysis presented by
maps, quantitative and geospatial data, images,
and landscapes.
SPS-1
1.7 Regional Analysis
1.A
 Describe geographic concepts, processes,
models, and theories.
Go to AP Classroom to assign the Personal Progress Check for Unit 1.
Review the results in class to identify and address any student misunderstandings.
28 
|
Course Framework V.1 AP Human Geography Course and Exam Description
00762-133-CED-Human-Geo_Unit 1.indd 28 3/14/19 9:49 PM
The Unit at a Glance table shows the topics, related enduring
understandings, and suggested skills. The “class periods”
column has been left blank so you can customize the time you
spend on each topic.
The table includes suggested skills for each topic to show
possible ways to link the content in that topic to specic
AP Human Geography skills. The individual skills have been
thoughtfully chosen in a way that allows you to scaold them
throughout the course. The questions on the Personal Progress
Checks are based on this pairing. However, AP Exam questions
can pair the content with any of the skills.
Using the Unit Guides
UNIT OPENERS
Developing Understanding provides an overview that
contextualizes and situates the key content of the unit within
the scope of the course.
The big ideas connect students’ learning of concepts and skills
throughout the course to develop understanding.The thought-
provoking essential questions that follow motivate students
and inspire inquiry.
Course Framework V.1
|
22
AP Human GeographyCourse and Exam Description
Return to Table of Contents
© 2020 College Board
Thinking Geographically
UNIT
1
Required Course Content
TOPIC 1.1
Introduction
to Maps
SUGGESTED SKILL
Data Analysis
3.A
Identify the different types
of data presented in maps
and in quantitative and
geospatial data.
AVAILABLE RESOURCES
§ Classroom Resources >
Maps and Spatial
Thinking Skills in the
AP Human Geography
Classroom
§ Classroom Resources >
Dening Geography:
What Is Where, Why
There, and Why Care?
§ Classroom Resources >
Scale
LEARNING OBJECTIVE
IMP-1.A
Identify types of maps,
the types of information
presented in maps, and
dierent kinds of spatial
patterns and relationships
portrayed in maps.
ESSENTIAL KNOWLEDGE
IMP-1.A.1
Types of maps include reference maps and
thematic maps.
IMP-1.A.2
Types of spatial patterns represented on maps
include absolute and relative distance and
direction, clustering, dispersal, and elevation.
IMP-1.A.3
All maps are selective in information;
map projections inevitably distort spatial
relationships in shape, area, distance,
anddirection.
ENDURING UNDERSTANDING
IMP-1
Geographers use maps and data to depict relationships of time, space, and scale.
30 
|
Course Framework V.1
AP Human Geography Course and Exam Description
00762-133-CED-Human-Geo_Unit 1.indd 30 3/14/19 9:49 PM
Thinking Geographically
UNIT
1
Activity Topic Sample Activity
1
1.1
Quickwrite
Set aside a short, specic amount of time during a class period for students to
perform a quickwrite on the power of maps. As preparation for answering free-
response questions later in the course, have students identify two dierent types of
map projections and explain why dierent map projections exist.
2
1.5
Critique Reasoning
Using the two major schools of thought regarding human–environmental
interaction—environmental determinism and possibilism—have students identify
the claims of each, explain the reasoning behind each, and nd current real-world
examples supporting and/or refuting each. Allow for collaborative discussion at
multiple levels—pairs, small groups, and whole group—to achieve understanding.
3
1.6
Debrieng
Scales of analysis is a challenging topic for students. They tend to struggle with
understanding the signicance of examining issues at multiple scales. For complex
issues, leading a facilitated discussion of data that illustrate scale concepts can help
solidify and deepen understanding of content. For example, you can take a set of data
and show what the global patterns are, zoom in to illustrate the national level, then
zoom in again to show the dierence in the local level analysis. Finish by leading a
discussion on the signicance of the dierences in patterns.
SAMPLE INSTRUCTIONAL ACTIVITIES
The sample activities on this page provide ways to integrate the teaching of skills and content
from the unit into your classroom instruction. They are completely optional and may be
altered to suit the needs of your class. Please refer to the Instructional Approaches section
beginning on page 133 for more examples of activities and strategies.
Unit Planning Notes
Use the space below to plan your approach to the unit. Consider how you want to pace your course and
your methods of instruction and assessment.
Course Framework V.1
|
29
AP Human Geography Course and Exam Description
00762-133-CED-Human-Geo_Unit 1.indd 29 3/14/19 9:49 PM
Using the Unit Guides
TOPIC PAGES
The suggested skill is one that complements the content in
that topic.
Where possible, available resources are included that might
help you address a particular topic in your classroom.
Enduring understandings are the intended
long-term takeaways related to the big ideas that leave
alasting impression on students. Students develop these
understandings over time by exploring and applying course
content throughout the year.
Essential knowledge statements describe the knowledge
required to perform the learning objectives.
Learning objectives dene what a student needs to be able
to do with content knowledge in order to progress toward the
enduring understandings.
The Sample Instructional Activities page includes optional
activities that can help you connect the content and suggested
skill of a particular topic. Additionally, this page oers space for
you to make notes on your approach to the individual topics
and the unit as a whole.
Course Framework V.1
|
23
AP Human GeographyCourse and Exam Description
Return to Table of Contents
© 2020 College Board
ENDURING
UNDERSTANDING
PSO-1
Geographers analyze
relationships among and
between places to reveal
important spatial patterns.
BIG IDEA
PSO
Patterns
and Spatial
Organization
NOTE: Labels are used to distinguish each unique element of the required course content and are used throughout this course and exam
description. Additionally, they are used in the AP Question Bank and other resources found in AP Classroom. Enduring understandings are labeled
sequentially according to the big idea that they are related to. Learning objectives are labeled to correspond with the enduring understanding
they relate to. Finally, essential knowledge statements are labeled to correspond with the learning objective they relate to.
LEARNING
OBJECTIVE
PSO-1.B
Explain how major
geographic concepts
illustrate spatial
relationships.
ESSENTIAL
KNOWLEDGE
PSO-1.B.2
Theories regarding the
interaction of the natural
environment with human
societies have evolved from
environmental determinism to
possibilism.
REQUIRED COURSE CONTENT LABELING SYSTEM
Course Framework V.1
|
24
AP Human GeographyCourse and Exam Description
~9–10
CLASS PERIODS
8–10
%
AP EXAM WEIGHTING
AP HUMAN GEOGRAPHY
UNIT
Thinking
Geographically
1
Return to Table of Contents
© 2020 College Board
Course Framework V.1
|
25
AP Human GeographyCourse and Exam Description
Remember to go to AP Classroom
to assign students the online
Personal Progress Check for
this unit.
Whether assigned as homework or
completed in class, the Personal
Progress Check provides each
student with immediate feedback
related to this unit’s topics and skills.
Personal Progress Check 1
Multiple-choice: ~20 questions
Free-response: 1 question
§ 1 stimulus
Return to Table of Contents
© 2020 College Board
Course Framework V.1
|
26
AP Human GeographyCourse and Exam Description
Return to Table of Contents
© 2020 College Board
UNIT
1
Developing Understanding
This rst unit sets the foundation for the course by teaching students
how geographers approach the study of places. Students are
encouraged to reect on the “why of where” to better understand
geographic perspectives. Many other high school courses ask
students to read and analyze data, but for this course, students also
apply a spatial perspective when reading and analyzing qualitative and
quantitative data.
Students learn the ways information from data sources such as
maps, tables, charts, satellite images, and infographics informs policy
decisions such as voting redistricting or expanding transportation
networks. They also learn about how people inuence and are
inuenced by their environment; the resulting impact on topography,
natural resources, and climate; and the dierences between and
consequences of environmental determinism and possibilism.
Finally, students are introduced to the language of geography,
learning discipline-specic terminology and applying that language
to contemporary, real-world scenarios so they can better study
population processes and patterns in the next unit.
BIG IDEA 1
Patterns and Spatial
Organization
PSO
§ Why do geographers
study relationships and
patterns among and
between places?
BIG IDEA 2
Impacts and
Interactions
IMP
§ How do geographers
use maps to help
themdiscover patterns
and relationships in
theworld?
BIG IDEA 3
Spatial Processes and
Societal Change
SPS
§ How do geographers
use a spatial
perspective to analyze
complex issues
andrelationships?
Thinking
Geographically
8–10
%
 AP EXAM WEIGHTING ~910 CLASS PERIODS
Course Framework V.1
|
27AP Human Geography Course and Exam Description
UNIT AT A GLANCE
Thinking Geographically
UNIT
1
Enduring
Understanding
Topic Suggested Skill
Class Periods
~910 CLASS PERIODS
IMP-1
1.1 Introduction to Maps
3.A
 Identify the dierent types of data presented
in maps and in quantitative and geospatial data.
1.2 Geographic Data
3.A
 Identify the dierent types of data presented
in maps and in quantitative and geospatial data.
1.3 The Power of
GeographicData
3.B
 Describe spatial patterns presented in maps
and in quantitative and geospatial data.
PSO-1
1.4 Spatial Concepts
3.B
 Describe spatial patterns presented in maps
and in quantitative and geospatial data.
1.5 Human–Environmental
Interaction
1.B
 Explain geographic concepts, processes,
models, and theories.
1.6 Scales of Analysis
5.A
 Identify the scales of analysis presented by
maps, quantitative and geospatial data, images,
and landscapes.
SPS-1
1.7 Regional Analysis
1.A
 Describe geographic concepts, processes,
models, and theories.
Go to AP Classroom to assign the Personal Progress Check for Unit 1.
Review the results in class to identify and address any student misunderstandings.
Return to Table of Contents
© 2020 College Board
Course Framework V.1
|
28
AP Human GeographyCourse and Exam Description
Thinking Geographically
UNIT
1
SAMPLE INSTRUCTIONAL ACTIVITIES
The sample activities on this page provide ways to integrate the teaching of skills and content
from the unit into your classroom instruction. They are completely optional and may be
altered to suit the needs of your class. Please refer to the Instructional Approaches section
beginning on page 133 for more examples of activities and strategies.
Unit Planning Notes
Use the space below to plan your approach to the unit. Consider how you want to pace your course and
your methods of instruction and assessment.
Activity Topic Sample Activity
1
1.1
Quickwrite
Set aside a short, specic amount of time during a class period for students to
perform a quickwrite on the power of maps. As preparation for answering free-
response questions later in the course, have students identify two dierent types of
map projections and explain why dierent map projections exist.
2
1.5
Critique Reasoning
Using the two major schools of thought regarding human–environmental
interaction—environmental determinism and possibilism—have students identify
the claims of each, explain the reasoning behind each, and nd current real-world
examples supporting and/or refuting each. Allow for collaborative discussion at
multiple levels—pairs, small groups, and whole group—to achieve understanding.
3
1.6
Debrieng
Scales of analysis is a challenging topic for students. They tend to struggle with
understanding the signicance of examining issues at multiple scales. For complex
issues, leading a facilitated discussion of data that illustrate scale concepts can help
solidify and deepen understanding of content. For example, you can take a set of data
and show what the global patterns are, zoom in to illustrate the national level, then
zoom in again to show the dierence in the local level analysis. Finish by leading a
discussion on the signicance of the dierences in patterns.
Return to Table of Contents
© 2020 College Board
Course Framework V.1
|
29
AP Human GeographyCourse and Exam Description
Thinking Geographically
UNIT
1
Required Course Content
TOPIC 1.1
Introduction
to Maps
SUGGESTED SKILL
Data Analysis
3.A
Identify the different types
of data presented in maps
and in quantitative and
geospatial data.
AVAILABLE RESOURCES
§ Classroom Resources >
Maps and Spatial
Thinking Skills in the
AP Human Geography
Classroom
§ Classroom Resources >
Dening Geography:
What Is Where, Why
There, and Why Care?
§ Classroom Resources >
Scale
LEARNING OBJECTIVE
IMP-1.A
Identify types of maps,
the types of information
presented in maps, and
dierent kinds of spatial
patterns and relationships
portrayed in maps.
ESSENTIAL KNOWLEDGE
IMP-1.A.1
Types of maps include reference maps and
thematic maps.
IMP-1.A.2
Types of spatial patterns represented on maps
include absolute and relative distance and
direction, clustering, dispersal, and elevation.
IMP-1.A.3
All maps are selective in information;
map projections inevitably distort spatial
relationships in shape, area, distance,
anddirection.
ENDURING UNDERSTANDING
IMP-1
Geographers use maps and data to depict relationships of time, space, and scale.
Return to Table of Contents
© 2020 College Board
Course Framework V.1
|
30
AP Human GeographyCourse and Exam Description
Thinking Geographically
UNIT
1
LEARNING OBJECTIVE
IMP-1.B
Identify dierent methods of
geographic data collection.
ESSENTIAL KNOWLEDGE
IMP-1.B.1
Data may be gathered in the eld by
organizations or by individuals.
IMP-1.B.2
Geospatial technologies include geographic
information systems (GIS), satellite navigation
systems, remote sensing, and online mapping
and visualization.
IMP-1.B.3
Spatial information can come from written
accounts in the form of eld observations,
media reports, travel narratives, policy
documents, personal interviews, landscape
analysis, and photographic interpretation.
ENDURING UNDERSTANDING
IMP-1
Geographers use maps and data to depict relationships of time, space, and scale.
Required Course Content
SUGGESTED SKILL
Data Analysis
3.A
Identify the different types
of data presented in maps
and in quantitative and
geospatial data.
AVAILABLE RESOURCES
§ Classroom Resources >
Maps and Spatial
Thinking Skills in the
AP Human Geography
Classroom
§ Classroom Resources >
Dening Geography:
What is Where, Why
There, and Why Care?
TOPIC 1.2
Geographic
Data
Return to Table of Contents
© 2020 College Board
Course Framework V.1
|
31
AP Human GeographyCourse and Exam Description
Thinking Geographically
UNIT
1
Required Course Content
SUGGESTED SKILL
Data Analysis
3.B
Describe spatial patterns
presented in maps
and inquantitative and
geospatial data.
AVAILABLE RESOURCES
§ Classroom Resources >
Maps and Spatial
Thinking Skills in the
AP Human Geography
Classroom
§ Classroom Resources >
Dening Geography:
What Is Where, Why
There, and Why Care?
§ Classroom Resources >
Scale
TOPIC 1.3
The Power of
Geographic Data
LEARNING OBJECTIVE
IMP-1.C
Explain the geographical
eects of decisions
made using geographical
information.
ESSENTIAL KNOWLEDGE
IMP-1.C.1
Geospatial and geographical data, including
census data and satellite imagery, are used
at all scales for personal, business and
organizational, and governmental decision-
making purposes.
ENDURING UNDERSTANDING
IMP-1
Geographers use maps and data to depict relationships of time, space, and scale.
Return to Table of Contents
© 2020 College Board
Course Framework V.1
|
32
AP Human GeographyCourse and Exam Description
Thinking Geographically
UNIT
1
TOPIC 1.4
Spatial
Concepts
SUGGESTED SKILL
Data Analysis
3.B
Describe spatial patterns
presented in maps
and inquantitative and
geospatial data.
AVAILABLE RESOURCES
§ Classroom Resources >
Maps and Spatial
Thinking Skills in the
AP Human Geography
Classroom
§ Classroom Resources >
Dening Geography:
What Is Where, Why
There, and Why Care?
LEARNING OBJECTIVE
PSO-1.A
Dene major geographic
concepts that illustrate
spatial relationships.
ESSENTIAL KNOWLEDGE
PSO-1.A.1
Spatial concepts include absolute and relative
location, space, place, ows, distance decay,
time-space compression, and pattern.
ENDURING UNDERSTANDING
PSO-1
Geographers analyze relationships among and between places to reveal important
spatial patterns.
Required Course Content
Return to Table of Contents
© 2020 College Board
Course Framework V.1
|
33
AP Human GeographyCourse and Exam Description
Thinking Geographically
UNIT
1
Required Course Content
TOPIC 1.5
Human–Environmental
Interaction
LEARNING OBJECTIVE
PSO-1.B
Explain how major
geographicconcepts
illustrate spatialrelationships.
ESSENTIAL KNOWLEDGE
PSO-1.B.1
Concepts of nature and society include
sustainability, natural resources, and land use.
PSO-1.B.2
Theories regarding the interaction of the
natural environment with human societies
have evolved from environmental determinism
topossibilism.
ENDURING UNDERSTANDING
PSO-1
Geographers analyze relationships among and between places to reveal important
spatial patterns.
SUGGESTED SKILL
Concepts and
Processes
1.B
Explain geographic
concepts, processes,
models, and theories.
AVAILABLE RESOURCES
§ Classroom Resources >
Understanding Land
Use Patterns
Return to Table of Contents
© 2020 College Board
Course Framework V.1
|
34
AP Human GeographyCourse and Exam Description
Thinking Geographically
UNIT
1
Required Course Content
SUGGESTED SKILL
Scale Analysis
5.A
Identify the scales of
analysis presented by
maps, quantitative and
geospatial data, images,
and landscapes.
AVAILABLE RESOURCES
§ Classroom Resources >
Scale
TOPIC 1.6
Scales of
Analysis
LEARNING OBJECTIVE
PSO-1.C
Dene scales of analysis
used by geographers.
ESSENTIAL KNOWLEDGE
PSO-1.C.1
Scales of analysis include global, regional,
national, and local.
ENDURING UNDERSTANDING
PSO-1
Geographers analyze relationships among and between places to reveal important
spatial patterns.
PSO-1.D
Explain what scales of
analysis reveal.
PSO-1.D.1
Patterns and processes at dierent
scales reveal variations in, and dierent
interpretations of, data.
Return to Table of Contents
© 2020 College Board
Course Framework V.1
|
35
AP Human GeographyCourse and Exam Description
Thinking Geographically
UNIT
1
Required Course Content
TOPIC 1.7
Regional
Analysis
LEARNING OBJECTIVE
SPS-1.A
Describe dierent ways that
geographers dene regions.
ESSENTIAL KNOWLEDGE
SPS-1.A.1
Regions are dened on the basis of one or
more unifying characteristics or on patterns
ofactivity.
SPS-1.A.2
Types of regions include formal, functional, and
perceptual/vernacular.
SPS-1.A.3
Regional boundaries are transitional and often
contested and overlapping.
SPS-1.A.4
Geographers apply regional analysis at local,
national, and global scales.
ENDURING UNDERSTANDING
SPS-1
Geographers analyze complex issues and relationships with a distinctively
spatialperspective.
SUGGESTED SKILL
Concepts and
Processes
1.A
Describe geographic
concepts, processes,
models, and theories.
Return to Table of Contents
© 2020 College Board
Course Framework V.1
|
36
AP Human GeographyCourse and Exam Description
~1920
CLASS PERIODS
12–17
%
AP EXAM WEIGHTING
AP HUMAN GEOGRAPHY
UNIT
Population
and Migration
Patterns and
Processes
2
Return to Table of Contents
© 2020 College Board
Course Framework V.1
|
37
AP Human GeographyCourse and Exam Description
Remember to go to AP Classroom
to assign students the online
Personal Progress Check for
this unit.
Whether assigned as homework or
completed in class, the Personal
Progress Check provides each
student with immediate feedback
related to this unit’s topics and skills.
Personal Progress Check 2
Multiple-choice: ~35 questions
Free-response: 1 question
§ 1 stimulus
Return to Table of Contents
© 2020 College Board
Course Framework V.1
|
38
AP Human GeographyCourse and Exam Description
Return to Table of Contents
© 2020 College Board
UNIT
2
Developing Understanding
This unit addresses the patterns associated with human populations.
Populations may increase or decrease as a result of a combination of
natural changes (births and deaths) and migration patterns (emigration
and immigration). Students examine population distributions at
dierent scales—local, national, regional, and global. Population
pyramids demonstrate age-sex structures, revealing the growth or
decline of generations and allowing geographers to predict economic
needs based on reproductive and aging patterns.
Students learn about factors that inuence changes in population
as well as the long- and short-term eects of those population
changes on a place’s economy, culture, and politics. For example,
environmental degradation and natural hazards may prompt
population redistribution at various scales, which in turn creates
new pressures on the environment and on cultural, economic, and
political institutions. The study of migration patterns allows students
to examine factors contributing to voluntary and forced relocation and
the impact of these migrating populations on existing settlements.
Combined, the concepts and theories encountered in this unit
help students develop connections and transfer their learning in
upcoming units to course topics such as cultural patterns, the political
organization of space, food production issues, natural resource use,
and urban systems.
BIG IDEA 1
Patterns and Spatial
Organization
PSO
§ How does where and
how people live impact
global cultural, political,
and economic patterns?
BIG IDEA 2
Impacts and
Interactions
IMP
§ How does the interplay
of environmental,
economic, cultural,
and political factors
inuence changes in
population?
BIG IDEA 3
Spatial Patterns and
Societal Change
SPS
§ How do changes in
population aect a
place’s economy,
culture, and politics?
Population and
Migration Patterns
and Processes
1217
%
 AP EXAM WEIGHTING ~1920 CLASS PERIODS
Course Framework V.1
|
39AP Human GeographyCourse and Exam Description
UNIT AT A GLANCE
Population and Migration Patterns and Processes
UNIT
2
Enduring
Understanding
Topic Suggested Skill
Class Periods
~1920 CLASS PERIODS
PSO-2
2.1 Population Distribution
3.A
 Identify the dierent types of data presented
in maps and in quantitative and geospatial data.
2.2 Consequences of
Population Distribution
2.C
 Explain a likely outcome in a geographic
scenario using geographic concepts, processes,
models, or theories.
2.3 Population Composition
2.A
 Describe spatial patterns, networks, and
relationships.
IMP-2
2.4 Population Dynamics
3.C
 Explain patterns and trends in maps and
in quantitative and geospatial data to draw
conclusions.
2.5 The Demographic
Transition Model
3.B
 Describe spatial patterns presented in maps
and in quantitative and geospatial data.
2.6 Malthusian Theory
2.B
 Explain spatial relationships in a specied
context or region of the world, using geographic
concepts, processes, models, or theories.
SPS-2
2.7 Population Policies
2.C
 Explain a likely outcome in a geographic
scenario using geographic concepts, processes,
models, or theories.
2.8 Women and
DemographicChange
3.B
 Describe spatial patterns presented in maps
and in quantitative and geospatial data.
2.9 Aging Populations
2.C
 Explain a likely outcome in a geographic
scenario using geographic concepts, processes,
models, or theories.
IMP-2
2.10 Causes of Migration
2.B
 Explain spatial relationships in a specied
context or region of the world, using geographic
concepts, processes, models, or theories.
2.11 Forced and Voluntary
Migration
1.D
 Describe a relevant geographic concept,
process, model, or theory in a specied context.
2.12 Effects of Migration
2.B
 Explain spatial relationships in a specied
context or region of the world, using geographic
concepts, processes, models, or theories.
Go to AP Classroom to assign the Personal Progress Check for Unit 2.
Review the results in class to identify and address any student misunderstandings.
Return to Table of Contents
© 2020 College Board
Course Framework V.1
|
40
AP Human GeographyCourse and Exam Description
Population and Migration Patterns and Processes
UNIT
2
Activity Topic Sample Activity
1
2.3
Create Representations
Although population pyramids are most often used to illustrate the population
composition of countries, they may also be used to illustrate the age-sex population
of subnational units. Provide students with scenarios regarding the characteristics of
dierent subnational units. Then have the students draw a population pyramid based
on the scenarios and explain their thinking.
2
2.5
Making Connections
Give students this list of terms: crude birth rate, crude death rate, rate of natural increase,
and total population. Allow students a few minutes to gather and recall information about
the terms or concepts and then pair students to discuss how the relationships among
the terms are represented in the demographic transition model. Then, ask students
to take dierent population pyramids and place them at the appropriate stage of the
demographic transition model. Have them justify their choice of stage.
3
2.12
Jigsaw
Assign students dierent countries, some with negative net migration rates and
others with positive net migration rates, and have them use a variety of resources
to examine the eects of migration on their assigned country. Then, create two
groups—one with countries with positive net migration rates and one with countries
with negative net migration rates—and have them discuss and compile information
gathered on migration eects. Lastly, create groups with mixed positive and negative
net migration rate countries and have students summarize and present information
on the impacts for these dierent countries.
SAMPLE INSTRUCTIONAL ACTIVITIES
The sample activities on this page provide ways to integrate the teaching of skills and content
from the unit into your classroom instruction. They are completely optional and may be
altered to suit the needs of your class. Please refer to the Instructional Approaches section
beginning on page 133 for more examples of activities and strategies.
Unit Planning Notes
Use the space below to plan your approach to the unit. Consider how you want to pace your course and
your methods of instruction and assessment.
Return to Table of Contents
© 2020 College Board
Course Framework V.1
|
41
AP Human GeographyCourse and Exam Description
Population and Migration Patterns and Processes
UNIT
2
Required Course Content
TOPIC 2.1
Population
Distribution
SUGGESTED SKILL
Data Analysis
3.A
Identify the different types
of data presented in maps
and in quantitative and
geospatial data.
AVAILABLE RESOURCES
§ Classroom Resources >
Maps and Spatial
Thinking Skills in the
AP Human Geography
Classroom
LEARNING OBJECTIVE
PSO-2.A
Identify the factors that
inuence the distribution
of human populations at
dierent scales.
ESSENTIAL KNOWLEDGE
PSO-2.A.1
Physical factors (e.g., climate, landforms,
water bodies) and human factors (e.g., culture,
economics, history, politics) inuence the
distribution of population.
PSO-2.A.2
Factors that illustrate patterns of population
distribution vary according to the scale
ofanalysis.
ENDURING UNDERSTANDING
PSO-2
Understanding where and how people live is essential to understanding global
cultural, political, and economic patterns.
PSO-2.B
Dene methods geographers
use to calculate population
density.
PSO-2.C
Explain the dierences
between and the impact of
methods used to calculate
population density.
PSO-2.B.1
The three methods for calculating population
density are arithmetic, physiological,
andagricultural.
PSO-2.C.1
The method used to calculate population
density reveals dierent information about the
pressure the population exerts on the land.
Return to Table of Contents
© 2020 College Board
Course Framework V.1
|
42
AP Human GeographyCourse and Exam Description
Population and Migration Patterns and Processes
UNIT
2
LEARNING OBJECTIVE
PSO-2.D
Explain how population
distribution and density
aect society and
theenvironment.
ESSENTIAL KNOWLEDGE
PSO-2.D.1
Population distribution and density aect
political, economic, and social processes,
including the provision of services such as
medical care.
PSO-2.D.2
Population distribution and density aect the
environment and natural resources; this is
known as carrying capacity.
ENDURING UNDERSTANDING
PSO-2
Understanding where and how people live is essential to understanding global
cultural, political, and economic patterns.
Required Course Content
SUGGESTED SKILL
Spatial
Relationships
2.C
Explain a likely outcome
in a geographic scenario
using geographic concepts,
processes, models, or
theories.
AVAILABLE RESOURCES
§ Classroom Resources >
Maps and Spatial
Thinking Skills in the
AP Human Geography
Classroom
TOPIC 2.2
Consequences
of Population
Distribution
Return to Table of Contents
© 2020 College Board
Course Framework V.1
|
43
AP Human GeographyCourse and Exam Description
Population and Migration Patterns and Processes
UNIT
2
Required Course Content
SUGGESTED SKILL
Spatial
Relationships
2.A
Describe spatial patterns,
networks, and relationships.
AVAILABLE RESOURCES
§ Classroom Resources >
Maps and Spatial
Thinking Skills in the
AP Human Geography
Classroom
TOPIC 2.3
Population
Composition
LEARNING OBJECTIVE
PSO-2.E
Describe elements of
population composition used
by geographers.
ESSENTIAL KNOWLEDGE
PSO-2.E.1
Patterns of age structure and sex ratio vary
across dierent regions and may be mapped
and analyzed at dierent scales.
ENDURING UNDERSTANDING
PSO-2
Understanding where and how people live is essential to understanding global
cultural, political, and economic patterns.
PSO-2.F
Explain ways that
geographers depict
and analyze population
composition.
PSO-2.F.1
Population pyramids are used to assess
population growth and decline and to predict
markets for goods and services.
Return to Table of Contents
© 2020 College Board
Course Framework V.1
|
44
AP Human GeographyCourse and Exam Description
Population and Migration Patterns and Processes
UNIT
2
TOPIC 2.4
Population
Dynamics
SUGGESTED SKILL
Data Analysis
3.C
Explain patterns and trends
in maps and in quantitative
and geospatial data to
drawconclusions.
LEARNING OBJECTIVE
IMP-2.A
Explain factors that account
for contemporary and
historical trends in population
growth and decline.
ESSENTIAL KNOWLEDGE
IMP-2.A.1
Demographic factors that determine a
population’s growth and decline are fertility,
mortality, and migration.
IMP-2.A.2
Geographers use the rate of natural increase
and population-doubling time to explain
population growth and decline.
IMP-2.A.3
Social, cultural, political, and economic factors
inuence fertility, mortality, and migration rates.
ENDURING UNDERSTANDING
IMP-2
Changes in population are due to mortality, fertility, and migration, which are
inuenced by the interplay of environmental, economic, cultural, and political factors.
Required Course Content
Return to Table of Contents
© 2020 College Board
Course Framework V.1
|
45
AP Human GeographyCourse and Exam Description
Population and Migration Patterns and Processes
UNIT
2
Required Course Content
TOPIC 2.5
The Demographic
Transition Model
LEARNING OBJECTIVE
IMP-2.B
Explain theories of population
growth and decline.
ESSENTIAL KNOWLEDGE
IMP-2.B.1
The demographic transition model can be used
to explain population change over time.
IMP-2.B.2
The epidemiological transition explains causes
of changing death rates.
ENDURING UNDERSTANDING
IMP-2
Changes in population are due to mortality, fertility, and migration, which are
inuenced by the interplay of environmental, economic, cultural, and political factors.
SUGGESTED SKILL
Data Analysis
3.B
Describe spatial patterns
presented in maps
and in quantitative and
geospatialdata.
AVAILABLE RESOURCES
§ Classroom Resources >
Maps and Spatial
Thinking Skills in the
AP Human Geography
Classroom
Return to Table of Contents
© 2020 College Board
Course Framework V.1
|
46
AP Human GeographyCourse and Exam Description
Population and Migration Patterns and Processes
UNIT
2
Required Course Content
SUGGESTED SKILL
Spatial
Relationships
2.B
Explain spatial relationships
in a specified context or
region of the world, using
geographic concepts,
processes, models,
ortheories.
TOPIC 2.6
Malthusian
Theory
LEARNING OBJECTIVE
IMP-2.B
Explain theories of population
growth and decline.
ESSENTIAL KNOWLEDGE
IMP-2.B.3
Malthusian theory and its critiques are
used to analyze population change and
itsconsequences.
ENDURING UNDERSTANDING
IMP-2
Changes in population are due to mortality, fertility, and migration, which are
inuenced by the interplay of environmental, economic, cultural, and political factors.
Return to Table of Contents
© 2020 College Board
Course Framework V.1
|
47
AP Human GeographyCourse and Exam Description
Population and Migration Patterns and Processes
UNIT
2
Required Course Content
TOPIC 2.7
Population
Policies
LEARNING OBJECTIVE
SPS-2.A
Explain the intent and
eects of various population
and immigration policies
on population size
andcomposition.
ESSENTIAL KNOWLEDGE
SPS-2.A.1
Types of population policies include those
that promote or discourage population
growth, such as pronatalist, antinatalist,
andimmigration policies.
ENDURING UNDERSTANDING
SPS-2
Changes in population have long- and short-term eects on a place’s economy,
culture, and politics.
SUGGESTED SKILL
Spatial
Relationships
2.C
Explain a likely outcome
in a geographic scenario
using geographic concepts,
processes, models,
ortheories.
Return to Table of Contents
© 2020 College Board
Course Framework V.1
|
48
AP Human GeographyCourse and Exam Description
Population and Migration Patterns and Processes
UNIT
2
Required Course Content
SUGGESTED SKILL
Data Analysis
3.B
Describe spatial patterns
presented in maps
and in quantitative and
geospatialdata.
TOPIC 2.8
Women and
Demographic
Change
LEARNING OBJECTIVE
SPS-2.B
Explain how the changing role
of females has demographic
consequences in dierent
parts of the world.
ESSENTIAL KNOWLEDGE
SPS-2.B.1
Changing social values and access to
education, employment, health care, and
contraception have reduced fertility rates in
most parts of the world.
SPS-2.B.2
Changing social, economic, and political roles
for females have inuenced patterns of fertility,
mortality, and migration, as illustrated by
Ravenstein’s laws of migration.
ENDURING UNDERSTANDING
SPS-2
Changes in population have long- and short-term eects on a place’s economy,
culture, and politics.
AVAILABLE RESOURCES
§ Classroom Resources >
Maps and Spatial
Thinking Skills in the
AP Human Geography
Classroom
Return to Table of Contents
© 2020 College Board
Course Framework V.1
|
49
AP Human GeographyCourse and Exam Description
Population and Migration Patterns and Processes
UNIT
2
Required Course Content
TOPIC 2.9
Aging
Populations
LEARNING OBJECTIVE
SPS-2.C
Explain the causes and
consequences of an
agingpopulation.
ESSENTIAL KNOWLEDGE
SPS-2.C.1
Population aging is determined by birth and
death rates and life expectancy.
SPS-2.C.2
An aging population has political, social,
and economic consequences, including the
dependency ratio.
ENDURING UNDERSTANDING
SPS-2
Changes in population have long- and short-term eects on a place’s economy,
culture, and politics.
SUGGESTED SKILL
Spatial
Relationships
2.C
Explain a likely outcome
in a geographic scenario
using geographic concepts,
processes, models,
ortheories.
AVAILABLE RESOURCES
§ Classroom Resources >
Maps and Spatial
Thinking Skills in the
AP Human Geography
Classroom
§ Classroom Resources >
Scale
Return to Table of Contents
© 2020 College Board
Course Framework V.1
|
50
AP Human GeographyCourse and Exam Description
Population and Migration Patterns and Processes
UNIT
2
Required Course Content
SUGGESTED SKILL
Spatial
Relationships
2.B
Explain spatial relationships
in a specified context or
region of the world, using
geographic concepts,
processes, models,
ortheories.
TOPIC 2.10
Causes of
Migration
LEARNING OBJECTIVE
IMP-2.C
Explain how dierent causal
factors encourage migration.
ESSENTIAL KNOWLEDGE
IMP-2.C.1
Migration is commonly divided into push
factors and pull factors.
IMP-2.C.2
Push/pull factors and intervening
opportunities/obstacles can be cultural,
demographic, economic, environmental,
orpolitical.
ENDURING UNDERSTANDING
IMP-2
Changes in population are due to mortality, fertility, and migration, which are
inuenced by the interplay of environmental, economic, cultural, and political factors.
Return to Table of Contents
© 2020 College Board
Course Framework V.1
|
51
AP Human GeographyCourse and Exam Description
Population and Migration Patterns and Processes
UNIT
2
Required Course Content
TOPIC 2.11
Forced and
Voluntary
Migration
LEARNING OBJECTIVE
IMP-2.D
Describe types of forced and
voluntary migration.
ESSENTIAL KNOWLEDGE
IMP-2.D.1
Forced migrations include slavery and events
that produce refugees, internally displaced
persons, and asylum seekers.
IMP-2.D.2
Types of voluntary migrations include
transnational, transhumance, internal, chain,
step, guest worker, and rural-to-urban.
ENDURING UNDERSTANDING
IMP-2
Changes in population are due to mortality, fertility, and migration, which are
inuenced by the interplay of environmental, economic, cultural, and political factors.
SUGGESTED SKILL
Concepts and
Processes
1.D
Describe a relevant
geographic concept,
process, model, or theory in
a specified context.
Return to Table of Contents
© 2020 College Board
Course Framework V.1
|
52
AP Human GeographyCourse and Exam Description
Population and Migration Patterns and Processes
UNIT
2
Required Course Content
SUGGESTED SKILL
Spatial
Relationships
2.B
Explain spatial relationships
in a specified context or
region of the world, using
geographic concepts,
processes, models,
ortheories.
TOPIC 2.12
Effects of
Migration
LEARNING OBJECTIVE
IMP-2.E
Explain historical and
contemporary geographic
eects of migration.
ESSENTIAL KNOWLEDGE
IMP-2.E.1
Migration has political, economic, and
culturaleects.
ENDURING UNDERSTANDING
IMP-2
Changes in population are due to mortality, fertility, and migration, which are
inuenced by the interplay of environmental, economic, cultural, and political factors.
Return to Table of Contents
© 2020 College Board
Course Framework V.1
|
53
AP Human GeographyCourse and Exam Description
THIS PAGE IS INTENTIONALLY LEFT BLANK.
~1920
CLASS PERIODS
12–17
%
AP EXAM WEIGHTING
AP HUMAN GEOGRAPHY
UNIT
Cultural
Patterns and
Processes
3
Return to Table of Contents
© 2020 College Board
Course Framework V.1
|
55
AP Human GeographyCourse and Exam Description
Remember to go to AP Classroom
to assign students the online
Personal Progress Check for
this unit.
Whether assigned as homework or
completed in class, the Personal
Progress Check provides each
student with immediate feedback
related to this unit’s topics and skills.
Personal Progress Check 3
Multiple-choice: ~25 questions
Free-response: 1 question
§ 2 stimuli
Return to Table of Contents
© 2020 College Board
Course Framework V.1
|
56
AP Human GeographyCourse and Exam Description
Return to Table of Contents
© 2020 College Board
UNIT
3
Developing Understanding
The main focus of this unit is on cultural patterns and processes that
create recognized cultural identities. Students consider the physical
environment to determine the eects of geographical location
and available resources on cultural practices. Visuals representing
artifacts, mentifacts and sociofacts all shed light on cultural
landscapes and how they change over time. Practice in analyzing
images of dierent places at dierent times for evidence of their
ethnicity, language, religion, gender roles and attitudes, and other
cultural attributes builds students’ understanding of cultural patterns
and processes.
This unit also considers from a temporal and spatial perspective
how culture spreads, through traditional forces such as colonialism
and imperialism and through contemporary inuences such as
social media. Rather than emphasize the details of cultural practices
associated with specic languages and religions, this unit instead
focuses on the distribution of cultural practices and on the causes
and eects of their diusion. For example, students might study the
distribution of Chinese versus English languages or the diusion
patterns of religions such as Hinduism and Islam, at local, national, or
global scales.
An understanding of the diusion of cultural practices provides a
foundation for the study of political patterns and processes in the
next unit.
BIG IDEA 1
Patterns and Spatial
Organization
PSO
§ How does where people
live and what resources
they have access to
impact their cultural
practices?
BIG IDEA 2
Impacts and
Interactions
IMP
§ How does the
interaction of people
contribute to the spread
of cultural practices?
BIG IDEA 3
Spatial Patterns and
Societal Change
SPS
§ How and why do cultural
ideas, practices, and
innovations change or
disappear over time?
Cultural
Patterns and
Processes
1217
%
 AP EXAM WEIGHTING ~1920 CLASS PERIODS
Course Framework V.1
|
57AP Human GeographyCourse and Exam Description
UNIT AT A GLANCE
Cultural Patterns and Processes
UNIT
3
Enduring
Understanding
Topic Suggested Skill
Class Periods
~1920 CLASS PERIODS
PSO-3
3.1 Introduction to Culture
4.A
 Identify the dierent types of information
presented in visual sources.
3.2 Cultural Landscapes
4.B
 Describe the spatial patterns presented in
visual sources.
3.3 Cultural Patterns
4.C
 Explain patterns and trends in visual sources
to draw conclusions.
IMP-3
3.4 Types of Diusion
1.D
 Describe a relevant geographic concept,
process, model, or theory in a specied context.
SPS-3
3.5 Historical Causes
ofDiusion
2.C
 Explain a likely outcome in a geographic
scenario using geographic concepts, processes,
models, or theories.
3.6 Contemporary Causes
ofDiusion
5.B
 Explain spatial relationships across various
geographic scales using geographic concepts,
processes, models, or theories.
IMP-3
3.7 Diusion of Religion
andLanguage
4.E
 Explain how maps, images, and landscapes
illustrate or relate to geographic principles,
processes, and outcomes.
SPS-3
3.8 Effects of Diusion
2.B
 Explain spatial relationships in a specied
context or region of the world, using geographic
concepts, processes, models, or theories.
Go to AP Classroom to assign the Personal Progress Check for Unit 3.
Review the results in class to identify and address any student misunderstandings.
Return to Table of Contents
© 2020 College Board
Course Framework V.1
|
58
AP Human GeographyCourse and Exam Description
Cultural Patterns and Processes
UNIT
3
Activity Topic Sample Activity
1
3.2
Discussion Group
Arrange students into small groups (4–6 students per group is recommended) and
have them analyze a set of images from a place. Have groups look for evidence of
dierent languages, religions, and ethnicities in the landscape. If students are not
accustomed to “reading” images, you may rst want to demonstrate reading the
cultural landscape with a set of images from the community surrounding the school
and walk students through the analytical process as a whole class. If student groups
are given dierent places to analyze, you may ask each group to share with the rest of
the class.
2
3.3
Look for a Pattern
Using dierent data sets, have students analyze regional patterns associated with
religions and languages. Have students hypothesize as to the reasons behind the
existence of the patterns they identify along with the consequences of the observed
patterns. Students will conrm or alter their reasons and consequences as they progress
through the unit. They can also reect back on Topic 3.2 and predict how the cultural
landscape would reect the identied patterns.
3
3.6
Fishbowl
Using secondary sources about the current spread of dierent phenomena and the
process of globalization, ask students to discuss how we became so connected
globally. The inner group models discussion while the outer group listens, responds,
and evaluates. Then the groups switch roles. You might also ask students to reect
back on Topic 3.5 and explain why the historical causes of diusion have changed.
SAMPLE INSTRUCTIONAL ACTIVITIES
The sample activities on this page provide ways to integrate the teaching of skills and content
from the unit into your classroom instruction. They are completely optional and may be
altered to suit the needs of your class. Please refer to the Instructional Approaches section
beginning on page 133 for more examples of activities and strategies.
Unit Planning Notes
Use the space below to plan your approach to the unit. Consider how you want to pace your course and
your methods of instruction and assessment.
Return to Table of Contents
© 2020 College Board
Course Framework V.1
|
59
AP Human GeographyCourse and Exam Description
Cultural Patterns and Processes
UNIT
3
Required Course Content
TOPIC 3.1
Introduction
to Culture
SUGGESTED SKILL
Source Analysis
4.A
Identify the different types
of information presented in
visual sources.
AVAILABLE RESOURCES
§ Classroom Resources >
Understanding Land
Use Patterns
LEARNING OBJECTIVE
PSO-3.A
Dene the characteristics,
attitudes, and traits that
inuence geographers when
they study culture.
ESSENTIAL KNOWLEDGE
PSO-3.A.1
Culture comprises the shared practices,
technologies, attitudes, and behaviors
transmitted by a society.
PSO-3.A.2
Cultural traits include such things as food
preferences, architecture, and land use.
PSO-3.A.3
Cultural relativism and ethnocentrism are
dierent attitudes toward cultural dierence.
ENDURING UNDERSTANDING
PSO-3
Cultural practices vary across geographical locations because of physical
geography and available resources.
Return to Table of Contents
© 2020 College Board
Course Framework V.1
|
60
AP Human GeographyCourse and Exam Description
Cultural Patterns and Processes
UNIT
3
LEARNING OBJECTIVE
PSO-3.B
Describe the characteristics
of cultural landscapes.
ESSENTIAL KNOWLEDGE
PSO-3.B.1
Cultural landscapes are combinations
of physical features, agricultural and
industrial practices, religious and linguistic
characteristics, evidence of sequent
occupancy, and other expressions of
culture including traditional and postmodern
architecture and land-use patterns.
ENDURING UNDERSTANDING
PSO-3
Cultural practices vary across geographical locations because of physical
geography and available resources.
Required Course Content
SUGGESTED SKILL
Source Analysis
4.B
Describe the spatial
patterns presented in
visualsources.
AVAILABLE RESOURCES
§ Classroom Resources >
Understanding Land
Use Patterns
§ Classroom Resources >
Cultural Landscape
Study
TOPIC 3.2
Cultural
Landscapes
PSO-3.C
Explain how landscape
features and land and
resource use reect cultural
beliefs and identities.
PSO-3.C.1
Attitudes toward ethnicity and gender,
including the role of women in the workforce;
ethnic neighborhoods; and indigenous
communities and lands help shape the use of
space in a given society.
Return to Table of Contents
© 2020 College Board
Course Framework V.1
|
61
AP Human GeographyCourse and Exam Description
Cultural Patterns and Processes
UNIT
3
Required Course Content
SUGGESTED SKILL
Source Analysis
4.C
Explain patterns and
trends in visual sources to
drawconclusions.
AVAILABLE RESOURCES
§ Classroom Resources >
Cultural Landscape
Study
§ Classroom Resources >
Maps and Spatial
Thinking Skills in the
AP Human Geography
Classroom
TOPIC 3.3
Cultural
Patterns
LEARNING OBJECTIVE
PSO-3.D
Explain patterns and
landscapes of language,
religion, ethnicity, and gender.
ESSENTIAL KNOWLEDGE
PSO-3.D.1
Regional patterns of language, religion, and
ethnicity contribute to a sense of place,
enhance placemaking, and shape the global
cultural landscape.
PSO -3.D.2
Language, ethnicity, and religion are factors in
creating centripetal and centrifugal forces.
ENDURING UNDERSTANDING
PSO-3
Cultural practices vary across geographical locations because of physical
geography and available resources.
Return to Table of Contents
© 2020 College Board
Course Framework V.1
|
62
AP Human GeographyCourse and Exam Description
Cultural Patterns and Processes
UNIT
3
TOPIC 3.4
Types of
Diffusion
SUGGESTED SKILL
Concepts and
Processes
1.D
Describe a relevant
geographic concept,
process, model, or theory
ina specified context.
LEARNING OBJECTIVE
IMP-3.A
Dene the types of diusion.
ESSENTIAL KNOWLEDGE
IMP-3.A.1
Relocation and expansion—including
contagious, hierarchical, and stimulus
expansion—are types of diusion.
ENDURING UNDERSTANDING
IMP-3
The interaction of people contributes to the spread of cultural practices.
Required Course Content
Return to Table of Contents
© 2020 College Board
Course Framework V.1
|
63
AP Human GeographyCourse and Exam Description
Cultural Patterns and Processes
UNIT
3
Required Course Content
TOPIC 3.5
Historical Causes
of Diffusion
LEARNING OBJECTIVE
SPS-3.A
Explain how historical
processes impact current
cultural patterns.
ESSENTIAL KNOWLEDGE
SPS-3.A.1
Interactions between and among cultural traits
and larger global forces can lead to new forms
of cultural expression; for example, creolization
and lingua franca.
SPS-3.A.2
Colonialism, imperialism, and trade helped to
shape patterns and practices of culture.
ENDURING UNDERSTANDING
SPS-3
Cultural ideas, practices, and innovations change or disappear over time.
SUGGESTED SKILL
Spatial
Relationships
2.C
Explain a likely outcome
in a geographic scenario
using geographic concepts,
processes, models,
ortheories.
Return to Table of Contents
© 2020 College Board
Course Framework V.1
|
64
AP Human GeographyCourse and Exam Description
Cultural Patterns and Processes
UNIT
3
Required Course Content
SUGGESTED SKILL
Scale Analysis
5.B
Explain spatial relationships
across various geographic
scales using geographic
concepts, processes,
models, or theories.
TOPIC 3.6
Contemporary
Causes of Diusion
LEARNING OBJECTIVE
SPS-3.A
Explain how historical
processes impact current
cultural patterns.
ESSENTIAL KNOWLEDGE
SPS-3.A.3
Cultural ideas and practices are socially
constructed and change through both
small-scale and large-scale processes such
as urbanization and globalization. These
processes come to bear on culture through
media, technological change, politics,
economics, and social relationships.
SPS-3.A.4
Communication technologies, such as the
internet and the time-space convergence,
are reshaping and accelerating interactions
among people; changing cultural practices, as
in the increasing use of English and the loss
of indigenous languages; and creating cultural
convergence and divergence.
ENDURING UNDERSTANDING
SPS-3
Cultural ideas, practices, and innovations change or disappear over time.
AVAILABLE RESOURCES
§ Classroom Resources >
Scale
§ Classroom Resources >
Globalization
Return to Table of Contents
© 2020 College Board
Course Framework V.1
|
65
AP Human GeographyCourse and Exam Description
Cultural Patterns and Processes
UNIT
3
Required Course Content
TOPIC 3.7
Diffusion of Religion
and Language
LEARNING OBJECTIVE
IMP-3.B
Explain what factors lead to
the diusion of universalizing
and ethnic religions.
ESSENTIAL KNOWLEDGE
IMP-3.B.1
Language families, languages, dialects, world
religions, ethnic cultures, and gender roles
diuse from cultural hearths.
IMP-3.B.2
Diusion of language families, including
Indo-European, and religious patterns and
distributions can be visually represented on
maps, in charts and toponyms, and in other
representations.
IMP-3.B.3
Religions have distinct places of origin
from which they diused to other locations
through dierent processes. Practices and
belief systems impacted how widespread the
religiondiused.
IMP-3.B.4
Universalizing religions, including Christianity,
Islam, Buddhism, and Sikhism, are spread
through expansion and relocation diusion.
IMP-3.B.5
Ethnic religions, including Hinduism and
Judaism, are generally found near the hearth or
spread through relocation diusion.
ENDURING UNDERSTANDING
IMP-3
The interaction of people contributes to the spread of cultural practices.
SUGGESTED SKILL
Source Analysis
4.E
Explain how maps, images,
and landscapes illustrate
or relate to geographic
principles, processes,
andoutcomes.
AVAILABLE RESOURCES
§ Classroom Resources >
Maps and Spatial
Thinking Skills in the
AP Human Geography
Classroom
§ Classroom Resources >
Cultural Landscape
Study
Return to Table of Contents
© 2020 College Board
Course Framework V.1
|
66
AP Human GeographyCourse and Exam Description
Cultural Patterns and Processes
UNIT
3
Required Course Content
SUGGESTED SKILL
Spatial
Relationships
2.B
Explain spatial relationships
in a specified context or
region of the world, using
geographic concepts,
processes, models,
ortheories.
TOPIC 3.8
Effects of
Diffusion
LEARNING OBJECTIVE
SPS-3.B
Explain how the process of
diusion results in changes
to the cultural landscape.
ESSENTIAL KNOWLEDGE
SPS-3.B.1
Acculturation, assimilation, syncretism, and
multiculturalism are eects of the diusion
ofculture.
ENDURING UNDERSTANDING
SPS-3
Cultural ideas, practices, and innovations change or disappear over time.
AVAILABLE RESOURCES
§ Classroom Resources >
Scale
Return to Table of Contents
© 2020 College Board
Course Framework V.1
|
67
AP Human GeographyCourse and Exam Description
THIS PAGE IS INTENTIONALLY LEFT BLANK.
~1920
CLASS PERIODS
12–17
%
AP EXAM WEIGHTING
AP HUMAN GEOGRAPHY
UNIT
Political
Patterns and
Processes
4
Return to Table of Contents
© 2020 College Board
Course Framework V.1
|
69
AP Human GeographyCourse and Exam Description
Remember to go to AP Classroom
to assign students the online
Personal Progress Check for
this unit.
Whether assigned as homework or
completed in class, the Personal
Progress Check provides each
student with immediate feedback
related to this unit’s topics and skills.
Personal Progress Check 4
Multiple-choice: ~30 questions
Free-response: 1 question
§ 1 stimulus
Return to Table of Contents
© 2020 College Board
Course Framework V.1
|
70
AP Human GeographyCourse and Exam Description
Return to Table of Contents
© 2020 College Board
UNIT
4
Developing Understanding
This unit addresses the political organization of the world. Building on
knowledge of populations and cultural patterns learned in previous
units, students examine the contemporary political map and the
impact of territoriality on political power and on issues of identity
for peoples. Students also look at the dierent types of political
boundaries, how they function, and their scale, as they consider both
internal and international boundaries. The interplay of political and
cultural inuences may cause tensions over boundaries to arise, such
as sovereign states making claims on what other states consider to be
international waters.
Students also examine forms of government and how forces
such as devolution may alter the functioning of political units and
cause changes to established political boundaries. Separatist
and independence movements that challenge the sovereignty of
political states may arise from economic and nationalistic forces,
as seen in Scotland, Northern Ireland, and Spain. The inuence of
supranational organizations such as the United Nations or European
Union and their role in global aairs presents another challenge to
nationalist sovereignty. Student understanding of cultural patterns and
processes helps inform their understanding of the consequences of
centrifugal and centripetal forces.
BIG IDEA 1
Patterns and Spatial
Organization
PSO
§ How do historical and
current events inuence
political structures
around the world?
BIG IDEA 2
Impacts and
Interactions
IMP
§ How are balances of
power reected in
political boundaries
and government power
structures?
BIG IDEA 3
Spatial Patterns and
Societal Change
SPS
§ How can political,
economic, cultural, or
technological changes
challenge state
sovereignty?
Political
Patterns and
Processes
1217
%
 AP EXAM WEIGHTING ~1920 CLASS PERIODS
Course Framework V.1
|
71AP Human GeographyCourse and Exam Description
UNIT AT A GLANCE
Political Patterns and Processes
UNIT
4
Enduring
Understanding
Topic Suggested Skill
Class Periods
~1920 CLASS PERIODS
PSO-4
4.1 Introduction to Political
Geography
4.A
 Identify the dierent types of information
presented in visual sources.
4.2 Political Processes
3.E
 Explain what maps or data imply or illustrate
about geographic principles, processes, and
outcomes.
4.3 Political Power and
Territoriality
5.B
 Explain spatial relationships across various
geographic scales using geographic concepts,
processes, models, or theories.
IMP-4
4.4 Dening Political
Boundaries
1.D
 Describe a relevant geographic concept,
process, model, or theory in a specied context.
4.5 The Function of Political
Boundaries
5.D
 Explain the degree to which a geographic
concept, process, model, or theory eectively
explains geographic eects across various
geographic scales.
4.6 Internal Boundaries
5.A
 Identify the scales of analysis presented by
maps, quantitative and geospatial data, images,
and landscapes.
4.7 Forms of Governance
2.A
 Describe spatial patterns, networks, and
relationships.
SPS-4
4.8 Dening Devolutionary
Factors
3.E
 Explain what maps or data imply or illustrate
about geographic principles, processes, and
outcomes.
4.9 Challenges to
Sovereignty
5.C
 Compare geographic characteristics and
processes at various scales.
4.10 Consequences of
Centrifugal and
Centripetal Forces
5.C
 Compare geographic characteristics and
processes at various scales.
Go to AP Classroom to assign the Personal Progress Check for Unit 4.
Review the results in class to identify and address any student misunderstandings.
Return to Table of Contents
© 2020 College Board
Course Framework V.1
|
72
AP Human GeographyCourse and Exam Description
UNIT
4
Political Patterns and Processes
Activity Topic Sample Activity
1
4.6
Think-Pair-Share
Have students individually think through the question, How can the way voting
district boundaries are drawn impact election outcomes? Then have students gather
evidence/examples to rene and/or support their thoughts. Students can then
discuss their response with a peer. A whole-class discussion can follow focused on
the issue of scale and how, due to internal political boundaries, it can appear that a
subnational unit is in support of one candidate or party when in fact changing the
scale may reveal variation in candidate or party loyalties.
2
4.8
Guided Discussion
This umbrella strategy allows a teacher to use multiple techniques with students in a
lesson. For this topic, use brainstorming and quickwrite as strategies to help students
understand how division of groups by physical geography, ethnic separatism, ethnic
cleansing, terrorism, economic and social problems, and irredentism may prompt
devolutionary movements in a state.
3
4.10
Graphic Organizer
Students should use a cause-and-eect graphic organizer to learn about the diering
potential results of centrifugal and centripetal forces. Once students have identied
the consequences of each, you can then ask them to look for contemporary
examples of the consequences, which may be negative or positive. Ask students to
discuss how dierent groups might have dierent interpretations of the nature of the
consequences. For example, the breakup of a state will be viewed negatively by the
formal state government but positively by separatists.
SAMPLE INSTRUCTIONAL ACTIVITIES
The sample activities on this page provide ways to integrate the teaching of skills and content
from the unit into your classroom instruction. They are completely optional and may be
altered to suit the needs of your class. Please refer to the Instructional Approaches section
beginning on page 133 for more examples of activities and strategies.
Unit Planning Notes
Use the space below to plan your approach to the unit. Consider how you want to pace your course and
your methods of instruction and assessment.
Return to Table of Contents
© 2020 College Board
Course Framework V.1
|
73
AP Human GeographyCourse and Exam DescriptionAP Human Geography Course and Exam Description
Political Patterns and Processes
Required Course Content
UNIT
4
SUGGESTED SKILL
Source Analysis
4.A
Identify the different types
of information presented in
visual sources.
AVAILABLE RESOURCES
§ Classroom Resources >
Scale
TOPIC 4.1
Introduction to
Political Geography
LEARNING OBJECTIVE
PSO-4.A
For world political maps:
a. Dene the dierent types
of political entities.
b. Identify a contemporary
example of political
entities.
ESSENTIAL KNOWLEDGE
PSO-4.A.1
Independent states are the primary building
blocks of the world political map.
PSO-4.A.2
Types of political entities include nations,
nation-states, stateless nations, multinational
states, multistate nations, and autonomous and
semiautonomous regions, such as American
Indian reservations.
ENDURING UNDERSTANDING
PSO-4
The political organization of space results from historical and current processes,
events, and ideas.
Return to Table of Contents
© 2020 College Board
Course Framework V.1
|
74
AP Human GeographyCourse and Exam Description
Political Patterns and Processes
Required Course Content
UNIT
4
TOPIC 4.2
Political
Processes
SUGGESTED SKILL
Data Analysis
3.E
Explain what maps or data
imply or illustrate about
geographic principles,
processes, and outcomes.
AVAILABLE RESOURCES
§ Classroom Resources >
Maps and Spatial
Thinking Skills in the
AP Human Geography
Classroom
LEARNING OBJECTIVE
PSO-4.B
Explain the processes that
have shaped contemporary
political geography.
ESSENTIAL KNOWLEDGE
PSO-4.B.1
The concepts of sovereignty, nation-
states, and self-determination shape the
contemporary world.
PSO-4.B.2
Colonialism, imperialism, independence
movements, and devolution along national
lines have inuenced contemporary political
boundaries.
ENDURING UNDERSTANDING
PSO-4
The political organization of space results from historical and current processes,
events, and ideas.
Return to Table of Contents
© 2020 College Board
Course Framework V.1
|
75
AP Human GeographyCourse and Exam Description
Political Patterns and Processes
UNIT
4
Required Course Content
SUGGESTED SKILL
Scale Analysis
5.B
Explain spatial relationships
across various geographic
scales using geographic
concepts, processes,
models, or theories.
AVAILABLE RESOURCES
§ Classroom Resources >
Scale
TOPIC 4.3
Political Power
and Territoriality
LEARNING OBJECTIVE
PSO-4.C
Describe the concepts
of political power and
territoriality as used by
geographers.
ESSENTIAL KNOWLEDGE
PSO-4.C.1
Political power is expressed geographically as
control over people, land, and resources, as
illustrated by neocolonialism, shatterbelts, and
choke points.
PSO-4.C.2
Territoriality is the connection of people,
theirculture, and their economic systems to
the land.
ENDURING UNDERSTANDING
PSO-4
The political organization of space results from historical and current processes,
events, and ideas.
Return to Table of Contents
© 2020 College Board
Course Framework V.1
|
76
AP Human GeographyCourse and Exam Description
Political Patterns and Processes
UNIT
4
Required Course Content
TOPIC 4.4
Defining Political
Boundaries
SUGGESTED SKILL
Concepts and
Processes
1.D
Describe a relevant
geographic concept,
process, model, or theory in
a specified context.
LEARNING OBJECTIVE
IMP-4.A
Dene types of political
boundaries used by
geographers.
ESSENTIAL KNOWLEDGE
IMP-4.A.1
Types of political boundaries include relic,
superimposed, subsequent, antecedent,
geometric, and consequent boundaries.
ENDURING UNDERSTANDING
IMP-4
Political boundaries and divisions of governance, between states and within them,
reect balances of power that have been negotiated or imposed.
Return to Table of Contents
© 2020 College Board
Course Framework V.1
|
77
AP Human GeographyCourse and Exam Description
Political Patterns and Processes
UNIT
4
Required Course Content
SUGGESTED SKILL
Scale Analysis
5.D
Explain the degree to which
a geographic concept,
process, model, or theory
effectively explains
geographic effects across
various geographic scales.
AVAILABLE RESOURCES
§ Classroom Resources >
Scale
TOPIC 4.5
The Function of
Political Boundaries
LEARNING OBJECTIVE
IMP-4.B
Explain the nature and
function of international and
internal boundaries.
ESSENTIAL KNOWLEDGE
IMP-4.B.1
Boundaries are dened, delimited, demarcated,
and administered to establish limits of
sovereignty, but they are often contested.
IMP-4.B.2
Political boundaries often coincide with
cultural, national, or economic divisions.
However, some boundaries are created by
demilitarized zones or policy, such as the Berlin
Conference.
IMP-4.B.3
Land and maritime boundaries and
international agreements can inuence
national or regional identity and encourage or
discourage international or internal interactions
and disputes over resources.
IMP-4.B.4
The United Nations Convention on the Law of
the Sea denes the rights and responsibilities
of nations in the use of international waters,
established territorial seas, and exclusive
economic zones.
ENDURING UNDERSTANDING
IMP-4
Political boundaries and divisions of governance, between states and within them,
reect balances of power that have been negotiated or imposed.
Return to Table of Contents
© 2020 College Board
Course Framework V.1
|
78
AP Human GeographyCourse and Exam Description
Political Patterns and Processes
UNIT
4
Required Course Content
TOPIC 4.6
Internal
Boundaries
SUGGESTED SKILL
Scale Analysis
5.A
Identify the scales of
analysis presented by
maps, quantitative and
geospatial data, images,
and landscapes.
AVAILABLE RESOURCES
§ Classroom Resources >
Scale
§ Classroom Resources >
Maps and Spatial
Thinking Skills in the
AP Human Geography
Classroom
LEARNING OBJECTIVE
IMP-4.B
Explain the nature and
function of international and
internal boundaries.
ESSENTIAL KNOWLEDGE
IMP-4.B.5
Voting districts, redistricting, and
gerrymandering aect election results at
various scales.
ENDURING UNDERSTANDING
IMP-4
Political boundaries and divisions of governance, between states and within them,
reect balances of power that have been negotiated or imposed.
Return to Table of Contents
© 2020 College Board
Course Framework V.1
|
79
AP Human GeographyCourse and Exam Description
Political Patterns and Processes
UNIT
4
SUGGESTED SKILLS
Spatial
Relationships
2.A
Describe spatial patterns,
networks, and relationships.
TOPIC 4.7
Forms of
Governance
LEARNING OBJECTIVE
IMP-4.C
Dene federal and
unitarystates.
IMP-4.D
Explain how federal and
unitary states aect spatial
organization.
ESSENTIAL KNOWLEDGE
IMP-4.C.1
Forms of governance include unitary states
and federal states.
IMP-4.D.1
Unitary states tend to have a more top-down,
centralized form of governance, while federal
states have more locally based, dispersed
power centers.
ENDURING UNDERSTANDING
IMP-4
Political boundaries and divisions of governance, between states and within them,
reect balances of power that have been negotiated or imposed.
AVAILABLE RESOURCES
§ Classroom Resources >
Scale
Required Course Content
Return to Table of Contents
© 2020 College Board
Course Framework V.1
|
80
AP Human GeographyCourse and Exam Description
Political Patterns and Processes
UNIT
4
TOPIC 4.8
Defining
Devolutionary
Factors
SUGGESTED SKILL
Data Analysis
3.E
Explain what maps or data
imply or illustrate about
geographic principles,
processes, and outcomes.
AVAILABLE RESOURCES
§ Classroom Resources >
Maps and Spatial
Thinking Skills in the
AP Human Geography
Classroom
LEARNING OBJECTIVE
SPS-4.A
Dene factors that lead to the
devolution of states.
ESSENTIAL KNOWLEDGE
SPS-4.A.1
Factors that can lead to the devolution of
states include the division of groups by
physical geography, ethnic separatism, ethnic
cleansing, terrorism, economic and social
problems, and irredentism.
ENDURING UNDERSTANDING
SPS-4
Political, economic, cultural, or technological changes can challenge state sovereignty.
Required Course Content
Return to Table of Contents
© 2020 College Board
Course Framework V.1
|
81
AP Human GeographyCourse and Exam Description
Political Patterns and Processes
UNIT
4
SUGGESTED SKILL
Scale Analysis
5.C
Compare geographic
characteristics and
processes at variousscales.
TOPIC 4.9
Challenges to
Sovereignty
ENDURING UNDERSTANDING
SPS-4
Political, economic, cultural, or technological changes can challenge state sovereignty.
Required Course Content
LEARNING OBJECTIVE
SPS-4.B
Explain how political,
economic, cultural, and
technological changes
challenge state sovereignty.
ESSENTIAL KNOWLEDGE
SPS-4.B.1
Devolution occurs when states fragment into
autonomous regions; subnational political-
territorial units, such as those within Spain,
Belgium, Canada, and Nigeria; or when states
disintegrate, as happened in Sudan and the
former Soviet Union.
SPS-4.B.2
Advances in communication technology have
facilitated devolution, supranationalism, and
democratization.
SPS-4.B.3
Global eorts to address transnational and
environmental challenges and to create
economies of scale, trade agreements,
and military alliances help to further
supranationalism.
SPS-4.B.4
Supranational organizations—including the
United Nations (UN), North Atlantic Treaty
Organization (NATO), European Union (EU),
Association of Southeast Asian Nations
(ASEAN), Arctic Council, and African Union—
can challenge state sovereignty by limiting the
economic or political actions of member states.
Return to Table of Contents
© 2020 College Board
Course Framework V.1
|
82
AP Human GeographyCourse and Exam Description
Political Patterns and Processes
UNIT
4
TOPIC 4.10
Consequences of
Centrifugal and
Centripetal Forces
SUGGESTED SKILL
Scale Analysis
5.C
Compare geographic
characteristics and
processes at variousscales.
LEARNING OBJECTIVE
SPS-4.C
Explain how the concepts
ofcentrifugal and centripetal
forces apply at the state scale.
ESSENTIAL KNOWLEDGE
SPS-4.C.1
Centrifugal forces may lead to failed states,
uneven development, stateless nations, and
ethnic nationalist movements.
SPS-4.C.2
Centripetal forces can lead to
ethnonationalism, more equitable
infrastructure development, and increased
cultural cohesion.
ENDURING UNDERSTANDING
SPS-4
Political, economic, cultural, or technological changes can challenge state sovereignty.
Required Course Content
Return to Table of Contents
© 2020 College Board
Course Framework V.1
|
83
AP Human GeographyCourse and Exam Description
THIS PAGE IS INTENTIONALLY LEFT BLANK.
~1920
CLASS PERIODS
12–17
%
AP EXAM WEIGHTING
AP HUMAN GEOGRAPHY
UNIT
Agriculture
and Rural
Land-Use
Patterns and
Processes
5
Return to Table of Contents
© 2020 College Board
Course Framework V.1
|
85
AP Human GeographyCourse and Exam Description
Remember to go to AP Classroom
to assign students the online
Personal Progress Check for
this unit.
Whether assigned as homework or
completed in class, the Personal
Progress Check provides each
student with immediate feedback
related to this unit’s topics and skills.
Personal Progress Check 5
Multiple-choice: ~35 questions
Free-response: 1 question
§ 2 stimuli
Return to Table of Contents
© 2020 College Board
Course Framework V.1
|
86
AP Human GeographyCourse and Exam Description
UNIT
5
Developing Understanding
This unit examines the origins of agriculture and its subsequent
diusion. Students learn about the ways agricultural practices have
changed over time as a result of technological innovations, such as
equipment mechanization and improvements in transportation that
create global markets. In addition, they examine the consequences
of agricultural practices such as the use of high-yield seeds and
chemicals, revisiting the human–environmental relationships studied
in Unit 1.
Course emphasis on spatial patterns is evident in this unit as
students consider the dierences in what foods or resources are
produced and where they are produced. These agricultural production
regions are impacted by economic and technological forces that
increase the size of agricultural operations and the carrying capacity
of the land. This has in turn created a global system of agriculture
and the interdependence of regions of agricultural consumption
andproduction.
Student understanding of this global system of agriculture based
on government cooperation lays the foundation for a deeper
understanding of economic development in the nal unit of
thecourse.
BIG IDEA 1
Patterns and Spatial
Organization
PSO
§ How do a people’s
culture and the
resources available to
them inuence how they
grow food?
BIG IDEA 2
Impacts and
Interactions
IMP
§ How does what people
produce and consume
vary in dierent
locations?
BIG IDEA 3
Spatial Patterns and
Societal Change
SPS
§ What kind of cultural
changes and
technological advances
have impacted the
way people grow and
consume food?
Agriculture and Rural
Land-Use Patterns
and Processes
1217
%
 AP EXAM WEIGHTING ~1920 CLASS PERIODS
Return to Table of Contents
© 2020 College Board
Course Framework V.1
|
87
AP Human GeographyCourse and Exam Description
UNIT AT A GLANCE
Agriculture and Rural Land-Use Patterns and Processes
UNIT
5
Enduring
Understanding
Topic Suggested Skill
Class Periods
~19–20
CLASS PERIODS
PSO-5
5.1 Introduction to
Agriculture
2.D
 Explain the signicance of geographic similarities
and dierences among dierent locations and/or at
dierent times.
5.2 Settlement Patterns and
Survey Methods
4.D
 Compare patterns and trends in visual sources to
draw conclusions.
SPS-5
5.3 Agricultural Origins and
Diusions
2.B
 Explain spatial relationships in a specied context
or region of the world, using geographic concepts,
processes, models, or theories.
5.4 The Second Agricultural
Revolution
4.D
 Compare patterns and trends in visual sources to
draw conclusions.
5.5 The Green Revolution
2.D
 Explain the signicance of geographic similarities
and dierences among dierent locations and/or at
dierent times.
PSO-5
5.6 Agricultural Production
Regions
2.E
 Explain the degree to which a geographic concept,
process, model, or theory eectively explains geographic
eects in dierent contexts and regions of the world.
5.7 Spatial Organization of
Agriculture
2.D
 Explain the signicance of geographic similarities
and dierences among dierent locations and/or at
dierent times.
5.8 Von Thünen Model
5.B
 Explain spatial relationships across various
geographic scales using geographic concepts,
processes, models, or theories.
5.9 The Global System of
Agriculture
5.D
 Explain the degree to which a geographic concept,
process, model, or theory eectively explains geographic
eects and across various geographic scales.
IMP-5
5.10 Consequences of
Agricultural Practices
2.E
 Explain the degree to which a geographic
concept, process, model, or theory eectively explains
geographic eects in dierent contexts and regions of
the world.
5.11 Challenges of
Contemporary Agriculture
4.D
 Compare patterns and trends in visual sources to
draw conclusions.
5.12 Women in Agriculture
3.D
 Compare patterns and trends in maps and in
quantitative and geospatial data to draw conclusions.
Go to AP Classroom to assign the Personal Progress Check for Unit 5.
Review the results in class to identify and address any student misunderstandings.
Return to Table of Contents
© 2020 College Board
Course Framework V.1
|
88
AP Human GeographyCourse and Exam Description
UNIT
5
Agriculture and Rural Land-Use Patterns and Processes
Activity Topic Sample Activity
1
5.3
Close Reading
Have students read materials regarding the Columbian Exchange and the subsequent
diusion of plants and animals across the world. In their reading, students can
highlight claims of changes to agriculture as a result of the Columbian Exchange.
Next, have students hypothesize how their diets would be dierent than they are
today had the Columbian Exchange not occurred.
2
5.8
Shared Inquiry
Have students read “Understanding Land Use Patterns” by Robert Bednarz, available
under Classroom Resources on the AP Central site. Have students discuss the
factors of land-use patterns identied by Bednarz in his article and how and why they
have changed from Von Thünen’s time.
3
5.10
Socratic Seminar
By holding a focused discussion around the essential question, How does society
change as agricultural practices change? you can provide students with an
opportunity to illustrate their understanding in depth. Students can be encouraged to
address environmental, social, economic, and cultural opportunities and challenges
for both commercial and subsistence agriculture.
SAMPLE INSTRUCTIONAL ACTIVITIES
The sample activities on this page provide ways to integrate the teaching of skills and content
from the unit into your classroom instruction. They are completely optional and may be
altered to suit the needs of your class. Please refer to the Instructional Approaches section
beginning on page 133 for more examples of activities and strategies.
Unit Planning Notes
Use the space below to plan your approach to the unit. Consider how you want to pace your course and
your methods of instruction and assessment.
Return to Table of Contents
© 2020 College Board
Course Framework V.1
|
89
AP Human GeographyCourse and Exam Description
Agriculture and Rural Land-Use Patterns and Processes
Required Course Content
UNIT
5
SUGGESTED SKILL
Spatial
Relationships
2.D
Explain the significance of
geographic similarities and
differences among different
locations and/or at different
times.
AVAILABLE RESOURCES
§ Classroom Resources >
Understanding Land
Use Patterns
TOPIC 5.1
Introduction to
Agriculture
LEARNING OBJECTIVE
PSO-5.A
Explain the connection
between physical geography
and agricultural practices.
ESSENTIAL KNOWLEDGE
PSO-5.A.1
Agricultural practices are inuenced by the
physical environment and climatic conditions,
such as the Mediterranean climate and tropical
climates.
PSO-5.A.2
Intensive farming practices include market
gardening, plantation agriculture, and mixed
crop/livestock systems.
PSO-5.A.3
Extensive farming practices include shifting
cultivation, nomadic herding, and ranching.
ENDURING UNDERSTANDING
PSO-5
Availability of resources and cultural practices inuence agricultural practices and
land-use patterns.
Return to Table of Contents
© 2020 College Board
Course Framework V.1
|
90
AP Human GeographyCourse and Exam Description
Agriculture and Rural Land-Use Patterns and Processes
Required Course Content
UNIT
5
TOPIC 5.2
Settlement Patterns
and Survey Methods
SUGGESTED SKILL
Source Analysis
4.D
Compare patterns and
trends in visual sources to
draw conclusions.
LEARNING OBJECTIVE
PSO-5.B
Identify dierent rural
settlement patterns and
methods of surveying rural
settlements.
ESSENTIAL KNOWLEDGE
PSO-5.B.1
Specic agricultural practices shape dierent
rural land-use patterns.
PSO-5.B.2
Rural settlement patterns are classied as
clustered, dispersed, or linear.
PSO-5.B.3
Rural survey methods include metes and
bounds, township and range, and long lot.
ENDURING UNDERSTANDING
PSO-5
Availability of resources and cultural practices inuence agricultural practices and
land-use patterns.
Return to Table of Contents
© 2020 College Board
Course Framework V.1
|
91
AP Human GeographyCourse and Exam Description
Agriculture and Rural Land-Use Patterns and Processes
Required Course Content
SUGGESTED SKILL
Spatial
Relationships
2.B
Explain spatial relationships
in a specified context or
region of the world, using
geographic concepts,
processes, models, or
theories.
AVAILABLE RESOURCES
§ Classroom Resources >
Scale
TOPIC 5.3
Agricultural Origins
and Diusions
UNIT
5
LEARNING OBJECTIVE
SPS-5.A
Identify major centers of
domestication of plants
andanimals.
ESSENTIAL KNOWLEDGE
SPS-5.A.1
Early hearths of domestication of plants and
animals arose in the Fertile Crescent and
several other regions of the world, including
theIndus River Valley, Southeast Asia, and
Central America.
ENDURING UNDERSTANDING
SPS-5
Agriculture has changed over time because of cultural diusion and advances in
technology.
SPS-5.B
Explain how plants and
animals diused globally.
SPS-5.B.1
Patterns of diusion, such as the Columbian
Exchange and the agricultural revolutions,
resulted in the global spread of various plants
and animals.
Return to Table of Contents
© 2020 College Board
Course Framework V.1
|
92
AP Human GeographyCourse and Exam Description
Agriculture and Rural Land-Use Patterns and Processes
UNIT
5
Required Course Content
ENDURING UNDERSTANDING
SPS-5
Agriculture has changed over time because of cultural diusion and advances in
technology.
LEARNING OBJECTIVE
SPS-5.C
Explain the advances and
impacts of the second
agricultural revolution.
ESSENTIAL KNOWLEDGE
SPS-5.C.1
New technology and increased food
production in the second agricultural revolution
led to better diets, longer life expectancies, and
more people available for work in factories.
SUGGESTED SKILL
Source Analysis
4.D
Compare patterns and
trends in visual sources to
draw conclusions.
TOPIC 5.4
The Second
Agricultural
Revolution
Return to Table of Contents
© 2020 College Board
Course Framework V.1
|
93
AP Human GeographyCourse and Exam Description
Agriculture and Rural Land-Use Patterns and Processes
Required Course Content
UNIT
5
LEARNING OBJECTIVE
SPS-5.D
Explain the consequences of
the Green Revolution on food
supply and the environment
in the developing world.
ENDURING UNDERSTANDING
SPS-5
Agriculture has changed over time because of cultural diusion and advances in
technology.
TOPIC 5.5
The Green
Revolution
SUGGESTED SKILL
Spatial
Relationships
2.D
Explain the significance of
geographic similarities and
differences among different
locations and/or at different
times.
ESSENTIAL KNOWLEDGE
SPS-5.D.1
The Green Revolution was characterized in
agriculture by the use of high-yield seeds,
increased use of chemicals, and mechanized
farming.
SPS-5.D.2
The Green Revolution had positive and negative
consequences for both human populations and
the environment.
Return to Table of Contents
© 2020 College Board
Course Framework V.1
|
94
AP Human GeographyCourse and Exam Description
Agriculture and Rural Land-Use Patterns and Processes
UNIT
5
Required Course Content
TOPIC 5.6
Agricultural
Production Regions
SUGGESTED SKILL
Spatial
Relationships
2.E
Explain the degree to which
a geographic concept,
process, model, or theory
effectively explains
geographic effects in
different contexts and
regions of the world.
AVAILABLE RESOURCES
§ Classroom Resources >
Understanding Land
Use Patterns
LEARNING OBJECTIVE
PSO-5.C
Explain how economic
forces inuence agricultural
practices.
ESSENTIAL KNOWLEDGE
PSO-5.C.1
Agricultural production regions are dened by
the extent to which they reect subsistence
or commercial practices (monocropping or
monoculture).
PSO-5.C.2
Intensive and extensive farming practices
are determined in part by land costs (bid-rent
theory).
ENDURING UNDERSTANDING
PSO-5
Availability of resources and cultural practices inuence agricultural practices and
land-use patterns.
Return to Table of Contents
© 2020 College Board
Course Framework V.1
|
95
AP Human GeographyCourse and Exam Description
Agriculture and Rural Land-Use Patterns and Processes
Required Course Content
UNIT
5
ESSENTIAL KNOWLEDGE
PSO-5.C.3
Large-scale commercial agricultural operations
are replacing small family farms.
PSO-5.C.4
Complex commodity chains link production
and consumption of agricultural products.
PSO-5.C.5
Technology has increased economies of scale
in the agricultural sector and the carrying
capacity of the land.
LEARNING OBJECTIVE
PSO-5.C
Explain how economic
forces inuence agricultural
practices.
ENDURING UNDERSTANDING
PSO-5
Availability of resources and cultural practices inuence agricultural practices and
land-use patterns.
SUGGESTED SKILL
Spatial
Relationships
2.D
Explain the significance
ofgeographic similarities
and differences among
different locations and/or
atdifferent times.
TOPIC 5.7
Spatial Organization
of Agriculture
Return to Table of Contents
© 2020 College Board
Course Framework V.1
|
96
AP Human GeographyCourse and Exam Description
Agriculture and Rural Land-Use Patterns and Processes
UNIT
5
Required Course Content
TOPIC 5.8
Von Thünen
Model
SUGGESTED SKILL
Scale Analysis
5.B
Explain spatial relationships
across various geographic
scales using geographic
concepts, processes,
models, or theories.
AVAILABLE RESOURCES
§ Classroom Resources >
Understanding Land
Use Patterns
§ Classroom Resources >
Scale
LEARNING OBJECTIVE
PSO-5.D
Describe how the von Thünen
model is used to explain
patterns of agricultural
production at various scales.
ESSENTIAL KNOWLEDGE
PSO-5.D.1
Von Thünen’s model helps to explain rural
land use by emphasizing the importance of
transportation costs associated with distance
from the market; however, regions of specialty
farming do not always conform to von Thünen’s
concentric rings.
ENDURING UNDERSTANDING
PSO-5
Availability of resources and cultural practices inuence agricultural practices and
land-use patterns.
Return to Table of Contents
© 2020 College Board
Course Framework V.1
|
97
AP Human GeographyCourse and Exam Description
UNIT
5
Required Course Content
Agriculture and Rural Land-Use Patterns and Processes
LEARNING OBJECTIVE
PSO-5.E
Explain the interdependence
among regions of agricultural
production and consumption.
ESSENTIAL KNOWLEDGE
PSO-5.E.1
Food and other agricultural products are part
of a global supply chain.
PSO-5.E.2
Some countries have become highly
dependent on one or more export
commodities.
PSO-5.E.3
The main elements of global food distribution
networks are aected by political relationships,
infrastructure, and patterns of world trade.
ENDURING UNDERSTANDING
POS-5
Availability of resources and cultural practices inuence agricultural practices and
land use patterns.
SUGGESTED SKILL
Scale Analysis
5.D
Explain the degree to which
a geographic concept,
process, model, or theory
effectively explains
geographic effects across
various geographic scales.
UNIT
5
TOPIC 5.9
The Global System
of Agriculture
AVAILABLE RESOURCES
§ Classroom Resources >
Scale
Return to Table of Contents
© 2020 College Board
Course Framework V.1
|
98
AP Human GeographyCourse and Exam Description
UNIT
5
Agriculture and Rural Land-Use Patterns and Processes
Required Course Content
TOPIC 5.10
Consequences of
Agricultural
Practices
SUGGESTED SKILL
Spatial
Relationships
2.E
Explain the degree to which
a geographic concept,
process, model, or theory
effectively explains
geographic effects in
different contexts and
regions of the world.
AVAILABLE RESOURCES
§ Classroom Resources >
Understanding Land
Use Patterns
LEARNING OBJECTIVE
IMP-5.A
Explain how agricultural
practices have environmental
and societal consequences.
ESSENTIAL KNOWLEDGE
IMP-5.A.1
Environmental eects of agricultural land
use include pollution, land cover change,
desertication, soil salinization, and
conservation eorts.
IMP-5.A.2
Agricultural practices—including slash and
burn, terraces, irrigation, deforestation,
draining wetlands, shifting cultivation, and
pastoral nomadism—alter the landscape.
IMP-5.A.3
Societal eects of agricultural practices
include changing diets, role of women in
agricultural production, and economic purpose.
ENDURING UNDERSTANDING
IMP-5
Agricultural production and consumption patterns vary in dierent locations, presenting
dierent environmental, social, economic, and cultural opportunities and challenges.
Return to Table of Contents
© 2020 College Board
Course Framework V.1
|
99
AP Human GeographyCourse and Exam Description
Agriculture and Rural Land-Use Patterns and Processes
UNIT
5
Required Course Content
SUGGESTED SKILL
Source Analysis
4.D
Compare patterns and
trends in visual sources to
draw conclusions.
AVAILABLE RESOURCES
§ Classroom Resources >
Understanding Land
Use Patterns
TOPIC 5.11
Challenges of
Contemporary
Agriculture
ENDURING UNDERSTANDING
IMP-5
Agricultural production and consumption patterns vary in dierent locations,
presenting dierent environmental, social, economic, and cultural opportunities
andchallenges.
LEARNING OBJECTIVE
IMP-5.B
Explain challenges and
debates related to the
changing nature of
contemporary agriculture and
food-production practices.
ESSENTIAL KNOWLEDGE
IMP-5.B.1
Agricultural innovations such as biotechnology,
genetically modied organisms, and
aquaculture have been accompanied by
debates over sustainability, soil and water
usage, reductions in biodiversity, and extensive
fertilizer and pesticide use.
IMP-5.B.2
Patterns of food production and consumption
are inuenced by movements relating to
individual food choice, such as urban farming,
community-supported agriculture (CSA),
organic farming, value-added specialty crops,
fair trade, local-food movements, and dietary
shifts.
IMP-5.B.3
Challenges of feeding a global population
include lack of food access, as in cases of food
insecurity and food deserts; problems with
distribution systems; adverse weather; and
land use lost to suburbanization.
IMP-5.B.4
The location of food-processing facilities
and markets, economies of scale, distribution
systems, and government policies all have
economic eects on food-production
practices.
Return to Table of Contents
© 2020 College Board
Course Framework V.1
|
100
AP Human GeographyCourse and Exam Description
Agriculture and Rural Land-Use Patterns and Processes
UNIT
5
Required Course Content
TOPIC 5.12
Women in Agriculture
SUGGESTED SKILL
Data Analysis
3.D
Compare patterns and
trends in maps and in
quantitative and geospatial
data to draw conclusions.
LEARNING OBJECTIVE
IMP-5.C
Explain geographic variations
in female roles in food
production and consumption.
ESSENTIAL KNOWLEDGE
IMP-5.C.1
The role of females in food production,
distribution, and consumption varies in many
places depending on the type of production
involved.
ENDURING UNDERSTANDING
IMP-5
Agricultural production and consumption patterns vary in dierent locations,
presenting dierent environmental, social, economic, and cultural opportunities
andchallenges.
Return to Table of Contents
© 2020 College Board
Course Framework V.1
|
101
AP Human GeographyCourse and Exam Description
THIS PAGE IS INTENTIONALLY LEFT BLANK.
~1920
CLASS PERIODS
12–17
%
AP EXAM WEIGHTING
AP HUMAN GEOGRAPHY
UNIT
Cities and
Urban Land-
Use Patterns
and Processes
6
Return to Table of Contents
© 2020 College Board
Course Framework V.1
|
103
AP Human GeographyCourse and Exam Description
Remember to go to AP Classroom
to assign students the online
Personal Progress Check for
this unit.
Whether assigned as homework or
completed in class, the Personal
Progress Check provides each
student with immediate feedback
related to this unit’s topics and skills.
Personal Progress Check 6
Multiple-choice: ~30 questions
Free-response: 1 question
§ no stimulus
Return to Table of Contents
© 2020 College Board
Course Framework V.1
|
104
AP Human GeographyCourse and Exam Description
UNIT
6
Developing Understanding
Unit 6 addresses the origins and inuences, particularly site and
situation, of urban settlements as students explore cities across
the world and the role of those cities in globalization. They examine
the spatial distribution of the world’s largest cities, comparing
them across regions and analyzing patterns of connectivity and
accessibility. Within cities, students identify patterns of development
and make inferences about their economic and political inuences at
regional, national, and international levels of scale. Students examine
the hierarchy of urban settlements on the landscape, applying the
rank-size rule and central place theory at regional and national scales
to evaluate mobility patterns and economic and political relationships.
Statistics such as census data are used to reveal the challenges
of urban places, including density, sprawl, demands of infrastructure,
and mobility.
Students examine patterns of change over time and modern
challenges to sustainability from urban growth. On both local
and global scales, they look at the ways that cities are improving
sustainability through new approaches to growth, such as mixed-land-
use zoning, smart growth policies, and public transportation–oriented
development at local and international scales.
This unit reinforces what students learned in the units on politics and
culture as they consider the role cities play as key centers of global
markets, culture, and politics and contrast the roles of urban and
ruralareas.
BIG IDEA 1
Patterns and Spatial
Organization
PSO
§ How do physical
geography and
resources impact the
presence and growth
ofcities?
BIG IDEA 2
Impacts and
Interactions
IMP
§ How are the attitudes,
values, and balance of
power of a population
reected in the built
landscape?
BIG IDEA 3
Spatial Patterns and
Societal Change
SPS
§ How are urban
areas aected by
unique economic,
political, cultural,
and environmental
challenges?
Cities and Urban
Land-Use Patterns
andProcesses
1217
%
 AP EXAM WEIGHTING ~1920 CLASS PERIODS
Return to Table of Contents
© 2020 College Board
Course Framework V.1
|
105
AP Human GeographyCourse and Exam Description
UNIT AT A GLANCE
Cities and Urban Land-Use Patterns and Processes
UNIT
6
Enduring
Understanding
Topic Suggested Skill
Class Periods
~1920 CLASS PERIODS
PSO-6
6.1 The Origin and
Inuences of
Urbanization
2.D
 Explain the signicance of geographic
similarities and dierences among dierent
locations and/or at dierent times.
6.2 Cities Across the World
2.D
 Explain the signicance of geographic
similarities and dierences among dierent
locations and/or at dierent times.
6.3 Cities and Globalization
5.B
 Explain spatial relationships across various
geographic scales using geographic concepts,
processes, models, or theories.
6.4 The Size and Distribution
of Cities
2.C
 Explain a likely outcome in a geographic
scenario using geographic concepts, processes,
models, or theories.
6.5 The Internal Structure
ofCities
1.E
 Explain the strengths, weaknesses, and
limitations of dierent geographic models and
theories in a specied context.
IMP-6
6.6 Density and Land Use
3.D
 Compare patterns and trends in maps
and in quantitative and geospatial data to
drawconclusions.
6.7 Infrastructure
3.C
 Explain patterns and trends in maps
andin quantitative and geospatial data to
drawconclusions.
6.8 Urban Sustainability
2.C
 Explain a likely outcome in a geographic
scenario using geographic concepts, processes,
models, or theories.
6.9 Urban Data
3.E
 Explain what maps or data imply or illustrate
about geographic principles, processes,
andoutcomes.
SPS-6
6.10 Challenges of Urban
Changes
4.E
 Explain how maps, images, and landscapes
illustrate or relate to geographic principles,
processes, and outcomes.
6.11 Challenges of Urban
Sustainability
2.D
 Explain the signicance of geographic
similarities and dierences among dierent
locations and/or at dierent times.
Go to AP Classroom to assign the Personal Progress Check for Unit 6.
Review the results in class to identify and address any student misunderstandings.
Return to Table of Contents
© 2020 College Board
Course Framework V.1
|
106
AP Human GeographyCourse and Exam Description
Cities and Urban Land-Use Patterns and Processes
UNIT
6
Activity Topic Sample Activity
1
6.5
Discussion Groups
Small groups of students can engage in discussions about the similarities and
dierences of dierent urban models, along with reasons behind those similarities
and dierences. A graphic organizer might help students organize their thoughts.
Then, as a class, students should explore the limitations of the urban models in
predicting the internal structure of cities.
2
6.9
Fishbowl
Divide the class in half, with one group gathering quantitative data on urban areas and
the other gathering qualitative data. Depending on the size of the class you may wish to
have data focusing on one city or several cities. Have them take turns being in the inner
circle, leading the discussion and bringing evidence to support their opinions, and being
in the outer circle, listening, responding, and evaluating. After students have participated
in both the inner and outer circles, you can lead a whole-class discussion on the
dierences between and the importance of both qualitative and quantitative data.
3
6.10
Debate
A debate gives students the opportunity to collect evidence in support of their
position and then orally present and defend it. Be sure to direct students to uncover
both the arming and contradictory evidence toward their position. For this
topic, you could have students debate the pros and cons of gentrication or how
governments should address squatter settlements—but it is important for students
to recognize that these issues are more complex than a simple pro or con.
SAMPLE INSTRUCTIONAL ACTIVITIES
The sample activities on this page provide ways to integrate the teaching of skills and content
from the unit into your classroom instruction. They are completely optional and may be
altered to suit the needs of your class. Please refer to the Instructional Approaches section
beginning on page 133 for more examples of activities and strategies.
Unit Planning Notes
Use the space below to plan your approach to the unit. Consider how you want to pace your course and
your methods of instruction and assessment.
Return to Table of Contents
© 2020 College Board
Course Framework V.1
|
107
AP Human GeographyCourse and Exam Description
Cities and Urban Land-Use Patterns and Processes
UNIT
6
Required Course Content
TOPIC 6.1
The Origin and
Influences of
Urbanization
SUGGESTED SKILL
Spatial
Relationships
2.D
Explain the significance
of geographic similarities
and differences among
different locations and/or at
differenttimes.
AVAILABLE RESOURCES
§ Classroom Resources >
Urban Geography
LEARNING OBJECTIVE
PSO-6.A
Explain the processes that
initiate and drive urbanization
and suburbanization.
ESSENTIAL KNOWLEDGE
PSO-6.A.1
Site and situation inuence the origin, function,
and growth of cities.
PSO-6.A.2
Changes in transportation and communication,
population growth, migration, economic
development, and government policies
inuence urbanization.
ENDURING UNDERSTANDING
PSO-6
The presence and growth of cities vary across geographical locations because of
physical geography and resources.
Return to Table of Contents
© 2020 College Board
Course Framework V.1
|
108
AP Human GeographyCourse and Exam Description
Cities and Urban Land-Use Patterns and Processes
UNIT
6
LEARNING OBJECTIVE
PSO-6.A
Explain the processes that
initiate and drive urbanization
and suburbanization.
ESSENTIAL KNOWLEDGE
PSO-6.A.3
Megacities and metacities are distinct spatial
outcomes of urbanization increasingly
located in countries of the periphery
andsemiperiphery.
PSO-6.A.4
Processes of suburbanization, sprawl, and
decentralization have created new land-use
forms—including edge cities, exurbs, and
boomburbs—and new challenges.
ENDURING UNDERSTANDING
PSO-6
The presence and growth of cities vary across geographical locations because of
physical geography and resources.
Required Course Content
SUGGESTED SKILL
Spatial
Relationships
2.D
Explain the significance
of geographic similarities
and differences among
different locations and/or at
differenttimes.
AVAILABLE RESOURCES
§ Classroom Resources >
Urban Geography
§ Classroom Resources >
Understanding Land
Use Patterns
TOPIC 6.2
Cities Across
the World
Return to Table of Contents
© 2020 College Board
Course Framework V.1
|
109
AP Human GeographyCourse and Exam Description
Cities and Urban Land-Use Patterns and Processes
UNIT
6
Required Course Content
SUGGESTED SKILL
Scale Analysis
5.B
Explain spatial relationships
across various geographic
scales using geographic
concepts, processes,
models, or theories.
AVAILABLE RESOURCES
§ Classroom Resources >
Urban Geography
§ Classroom Resources >
Scale
§ Classroom Resources >
Globalization
TOPIC 6.3
Cities and
Globalization
LEARNING OBJECTIVE
PSO-6.B
Explain how cities embody
processes of globalization.
ESSENTIAL KNOWLEDGE
PSO-6.B.1
World cities function at the top of the world’s
urban hierarchy and drive globalization.
PSO-6.B.2
Cities are connected globally by networks and
linkages and mediate global processes.
ENDURING UNDERSTANDING
PSO-6
The presence and growth of cities vary across geographical locations because of
physical geography and resources.
Return to Table of Contents
© 2020 College Board
Course Framework V.1
|
110
AP Human GeographyCourse and Exam Description
Cities and Urban Land-Use Patterns and Processes
UNIT
6
TOPIC 6.4
The Size and
Distribution
of Cities
SUGGESTED SKILL
Spatial
Relationships
2.C
Explain a likely outcome
in a geographic scenario
using geographic concepts,
processes, models,
ortheories.
LEARNING OBJECTIVE
PSO-6.C
Identify the dierent urban
concepts such as hierarchy,
interdependence, relative
size, and spacing that
are useful for explaining
the distribution, size, and
interaction of cities.
ESSENTIAL KNOWLEDGE
PSO-6.C.1
Principles that are useful for explaining the
distribution and size of cities include rank-size
rule, the primate city, gravity, and Christaller’s
central place theory.
ENDURING UNDERSTANDING
PSO-6
The presence and growth of cities vary across geographical locations because of
physical geography and resources.
Required Course Content
AVAILABLE RESOURCES
§ Classroom Resources >
Urban Geography
Return to Table of Contents
© 2020 College Board
Course Framework V.1
|
111
AP Human GeographyCourse and Exam Description
Cities and Urban Land-Use Patterns and Processes
UNIT
6
Required Course Content
TOPIC 6.5
The Internal
Structure of Cities
LEARNING OBJECTIVE
PSO-6.D
Explain the internal structure
of cities using various models
and theories.
ESSENTIAL KNOWLEDGE
PSO-6.D.1
Models and theories that are useful for
explaining internal structures of cities include
the Burgess concentric-zone model, the Hoyt
sector model, the Harris and Ullman multiple-
nuclei model, the galactic city model, bid-rent
theory, and urban models drawn from Latin
America, Southeast Asia, and Africa.
ENDURING UNDERSTANDING
PSO-6
The presence and growth of cities vary across geographical locations because of
physical geography and resources.
SUGGESTED SKILL
Concepts and
Processes
1.E
Explain the strengths,
weaknesses, and limitations
of different geographic
models and theories in a
specified context.
AVAILABLE RESOURCES
§ Classroom Resources >
Urban Geography
Return to Table of Contents
© 2020 College Board
Course Framework V.1
|
112
AP Human GeographyCourse and Exam Description
Cities and Urban Land-Use Patterns and Processes
UNIT
6
Required Course Content
SUGGESTED SKILL
Data Analysis
3.D
Compare patterns and
trends in maps and in
quantitative and geospatial
data to draw conclusions.
TOPIC 6.6
Density and
Land Use
LEARNING OBJECTIVE
IMP-6.A
Explain how low-, medium-,
and high-density housing
characteristics represent
dierent patterns of
residential land use.
ESSENTIAL KNOWLEDGE
IMP-6.A.1
Residential buildings and patterns of land
use reect and shape the city’s culture,
technological capabilities, cycles of
development, and inlling.
ENDURING UNDERSTANDING
IMP-6
The attitudes and values of a population, as well as the balance of power within that
population, are reected in the built landscape.
AVAILABLE RESOURCES
§ Classroom Resources >
Urban Geography
§ Classroom Resources >
Understanding Land
Use Patterns
§ Classroom Resources >
Maps and Spatial
Thinking Skills in the
AP Human Geography
Classroom
Return to Table of Contents
© 2020 College Board
Course Framework V.1
|
113
AP Human GeographyCourse and Exam Description
Cities and Urban Land-Use Patterns and Processes
UNIT
6
Required Course Content
TOPIC 6.7
Infrastructure
LEARNING OBJECTIVE
IMP-6.B
Explain how a city’s
infrastructure relates to
localpolitics, society, and
theenvironment.
ESSENTIAL KNOWLEDGE
IMP-6.B.1
The location and quality of a city’s
infrastructure directly aects its spatial
patterns of economic and social development.
ENDURING UNDERSTANDING
IMP-6
The attitudes and values of a population, as well as the balance of power within that
population, are reected in the built landscape.
SUGGESTED SKILL
Data Analysis
3.C
Explain patterns and trends
in maps and in quantitative
and geospatial data to
drawconclusions.
AVAILABLE RESOURCES
§ Classroom Resources >
Urban Geography
§ Classroom Resources >
Maps and Spatial
Thinking Skills in the
AP Human Geography
Classroom
Return to Table of Contents
© 2020 College Board
Course Framework V.1
|
114
AP Human GeographyCourse and Exam Description
Cities and Urban Land-Use Patterns and Processes
UNIT
6
Required Course Content
SUGGESTED SKILL
Spatial
Relationships
2.C
Explain a likely outcome
in a geographic scenario
using geographic
concepts,processes,
models, or theories.
TOPIC 6.8
Urban
Sustainability
LEARNING OBJECTIVE
IMP-6.C
Identify the dierent urban
design initiatives and
practices.
ESSENTIAL KNOWLEDGE
IMP-6.C.1
Sustainable design initiatives and zoning
practices include mixed land use, walkability,
transportation-oriented development,
and smart-growth policies, including New
Urbanism, greenbelts, and slow-growth cities.
ENDURING UNDERSTANDING
IMP-6
The attitudes and values of a population, as well as the balance of power within that
population, are reected in the built landscape.
AVAILABLE RESOURCES
§ Classroom Resources >
Urban Geography
§ Classroom Resources >
Understanding Land
Use Patterns
IMP-6.D
Explain the eects of
dierent urban design
initiatives and practices.
IMP-6.D .1
Praise for urban design initiatives includes the
reduction of sprawl, improved walkability and
transportation, improved and diverse housing
options, improved livability and promotion
of sustainable options. Criticisms include
increased housing costs, possible de facto
segregation, and the potential loss of historical
or place character.
Return to Table of Contents
© 2020 College Board
Course Framework V.1
|
115
AP Human GeographyCourse and Exam Description
Cities and Urban Land-Use Patterns and Processes
UNIT
6
Required Course Content
TOPIC 6.9
Urban Data
LEARNING OBJECTIVE
IMP-6.E
Explain how qualitative and
quantitative data are used to
show the causes and eects
of geographic change within
urban areas.
ESSENTIAL KNOWLEDGE
IMP-6.E.1
Quantitative data from census and survey
data provide information about changes in
population composition and size in urban
areas.
IMP-6.E.2
Qualitative data from eld studies and
narratives provide information about individual
attitudes toward urban change.
ENDURING UNDERSTANDING
IMP-6
The attitudes and values of a population, as well as the balance of power within that
population, are reected in the built landscape.
SUGGESTED SKILL
Data Analysis
3.E
Explain what maps or data
imply or illustrate about
geographic principles,
processes, and outcomes.
AVAILABLE RESOURCES
§ Classroom Resources >
Urban Geography
§ Classroom Resources >
Maps and Spatial
Thinking Skills in the
AP Human Geography
Classroom
Return to Table of Contents
© 2020 College Board
Course Framework V.1
|
116
AP Human GeographyCourse and Exam Description
Cities and Urban Land-Use Patterns and Processes
UNIT
6
Required Course Content
SUGGESTED SKILL
Source Analysis
4.E
Explain how maps, images,
and landscapes illustrate
or relate to geographic
principles, processes,
andoutcomes.
TOPIC 6.10
Challenges of
Urban Changes
LEARNING OBJECTIVE
SPS-6.A
Explain causes and eects
of geographic change within
urban areas.
ESSENTIAL KNOWLEDGE
SPS-6.A.1
As urban populations move within a city,
economic and social challenges result,
including: issues related to housing and
housing discrimination such as redlining,
blockbusting, and aordability; access to
services; rising crime; environmental injustice;
and the growth of disamenity zones or zones
ofabandonment.
SPS-6.A.2
Squatter settlements and conicts over land
tenure within large cities have increased.
SPS-6.A.3
Responses to economic and social challenges
in urban areas can include inclusionary zoning
and local food movements.
SPS-6.A.4
Urban renewal and gentrication have both
positive and negative consequences.
SPS-6.A.5
Functional and geographic fragmentation of
governments—the way government agencies
and institutions are dispersed between state,
county, city, and neighborhood levels—presents
challenges in addressing urban issues.
ENDURING UNDERSTANDING
SPS-6
Urban areas face unique economic, political, cultural, and environmental challenges.
AVAILABLE RESOURCES
§ Classroom Resources >
Urban Geography
§ Classroom Resources >
Maps and Spatial
Thinking Skills in the
AP Human Geography
Classroom
Return to Table of Contents
© 2020 College Board
Course Framework V.1
|
117
AP Human GeographyCourse and Exam Description
Cities and Urban Land-Use Patterns and Processes
UNIT
6
Required Course Content
TOPIC 6.11
Challenges of Urban
Sustainability
LEARNING OBJECTIVE
SPS-6.B
Describe the eectiveness
of dierent attempts to
address urban sustainability
challenges.
ESSENTIAL KNOWLEDGE
SPS-6.B.1
Challenges to urban sustainability include
suburban sprawl, sanitation, climate change, air
and water quality, the large ecological footprint
of cities, and energy use.
SPS-6.B.2
Responses to urban sustainability challenges
can include regional planning eorts,
remediation and redevelopment of brownelds,
establishment of urban growth boundaries, and
farmland protection policies.
ENDURING UNDERSTANDING
SPS-6
Urban areas face unique economic, political, cultural, and environmental challenges.
SUGGESTED SKILL
Spatial
Relationships
2.D
Explain the significance
of geographic similarities
and differences among
different locations and/or
atdifferenttimes.
AVAILABLE RESOURCES
§ Classroom Resources >
Urban Geography
Return to Table of Contents
© 2020 College Board
Course Framework V.1
|
118
AP Human GeographyCourse and Exam Description
~1920
CLASS PERIODS
12–17
%
AP EXAM WEIGHTING
AP HUMAN GEOGRAPHY
UNIT
Industrial
and Economic
Development
Patterns and
Processes
7
Return to Table of Contents
© 2020 College Board
Course Framework V.1
|
119
AP Human GeographyCourse and Exam Description
Remember to go to AP Classroom
to assign students the online
Personal Progress Check for
this unit.
Whether assigned as homework or
completed in class, the Personal
Progress Check provides each
student with immediate feedback
related to this unit’s topics and skills.
Personal Progress Check 7
Multiple-choice: ~25 questions
Free-response: 1 question
§ no stimulus
Return to Table of Contents
© 2020 College Board
Course Framework V.1
|
120
AP Human GeographyCourse and Exam Description
Return to Table of Contents
© 2020 College Board
UNIT
7
Developing Understanding
This unit addresses the origins and inuences of industrial
development, along with the role industrialization plays in economic
development and globalization. Concepts learned in the political
unit, such as territoriality, help students build an understanding of
the measures of social and economic development and to explain
development theories, such as dependency theory and Rostow’s
Stages of Economic Growth. The theories they explore are in turn
useful in explaining spatial variations in development such as core-
periphery relationships.
Students examine contemporary spatial patterns of industrialization
and the resulting geography of uneven development—for example,
the dierences between urban and rural China or Brazil. They explore
changes to places resulting from the growth or loss of industry
and the role of industry in the world economy. Measurements of
development provide the quantitative data to analyze the spatial
relationships of the global market. Statistics and spatial data reveal
the impact of development on individual populations, including
the role of women in the labor market. Students explore strategies
for sustainable development focused on women, children, health,
education, the environment, and global cooperation.
This nal unit of the course pulls together those aspects of human
geography learned in previous units to help students develop a more
complete understanding of local and global geographic patterns and
processes and of possibilities for the future.
BIG IDEA 1
Patterns and Spatial
Organization
PSO
§ Why does economic
and social development
happen at dierent
times and rates in
dierent places?
BIG IDEA 2
Impacts and
Interactions
IMP
§ How might
environmental
problems stemming
from industrialization
be remedied
through sustainable
development
strategies?
BIG IDEA 3
Spatial Patterns and
Societal Change
SPS
§ Why has industrialization
helped improve
standards of living while
also contributing to
geographically uneven
development?
Industrial and
Economic Development
Patterns and Processes
1217
%
 AP EXAM WEIGHTING ~1920 CLASS PERIODS
Course Framework V.1
|
121AP Human GeographyCourse and Exam Description
UNIT AT A GLANCE
Industrial and Economic Development Patterns and Processes
UNIT
7
Enduring
Understanding
Topic Suggested Skill
Class Periods
~1920 CLASS PERIODS
SPS-7
7.1 The Industrial Revolution
4.D
 Compare patterns and trends in visual sources
to draw conclusions.
7.2 Economic Sectors
andPatterns
2.B
 Explain spatial relationships in a specied
context or region of the world, using geographic
concepts, processes, models, or theories.
7.3 Measures of Development
3.F
 Explain possible limitations of the
dataprovided.
7.4 Women and Economic
Development
3.D
 Compare patterns and trends in maps
and in quantitative and geospatial data to
drawconclusions.
7.5 Theories of Development
1.E
 Explain the strengths, weaknesses, and
limitations of dierent geographic models and
theories in a specied context.
PSO-7
7.6 Trade and the World
Economy
5.B
 Explain spatial relationships across various
geographic scales using geographic concepts,
processes, models, or theories.
7.7 Changes as a Result of the
World Economy
4.F
 Explain possible limitations of visual
sourcesprovided.
IMP-7
7.8 Sustainable Development
5.D
 Explain the degree to which a geographic
concept, process, model, or theory eectively
explains geographic eects across various
geographic scales.
Go to AP Classroom to assign the Personal Progress Check for Unit 7.
Review the results in class to identify and address any student misunderstandings.
Return to Table of Contents
© 2020 College Board
Course Framework V.1
|
122
AP Human GeographyCourse and Exam Description
UNIT
7
Industrial and Economic Development Patterns and Processes
Activity Topic Sample Activity
1
7.3
Look for a Pattern
Using maps of dierent measures of development—both social and economic—
have students analyze the patterns present. Have students compare the dierent
measures to see which patterns are similar across multiple measures and the
advantages of using multiple measures when addressing levels of development.
You may want students to extend the depth of their thinking by having them reect
on what they have learned throughout the course to rst predict what patterns of
development will be present before they look at the maps.
2
7.5
Graphic Organizer
Students use a Venn diagram graphic organizer to compare and contrast dierent
theories of development, such as Rostow’s Stages of Economic Growth, Wallerstein’s
World System theory, dependency theory, and commodity dependence. You may
then have students discuss how dierent countries are classied according to the
dierent theories.
3
7.8
Self/Peer Review
Students are asked to read the United Nation’s Sustainable Development Goals.
After reading the goals, students select the one goal they believe is the most
important to achieve, and then justify their response in writing with evidence and
reasoning. Students then pass their paper to a peer, who can respond to their piece
with challenging questions regarding the choice of goal and with evidence that may
contradict the rst student’s position.
SAMPLE INSTRUCTIONAL ACTIVITIES
The sample activities on this page provide ways to integrate the teaching of skills and content
from the unit into your classroom instruction. They are completely optional and may be
altered to suit the needs of your class. Please refer to the Instructional Approaches section
beginning on page 133 for more examples of activities and strategies.
Unit Planning Notes
Use the space below to plan your approach to the unit. Consider how you want to pace your course and
your methods of instruction and assessment.
Return to Table of Contents
© 2020 College Board
Course Framework V.1
|
123
AP Human GeographyCourse and Exam DescriptionAP Human Geography Course and Exam Description
UNIT
7
Industrial and Economic Development Patterns and Processes
Required Course Content
TOPIC 7.1
The Industrial
Revolution
SUGGESTED SKILL
Source Analysis
4.D
Compare patterns and
trends in visual sources to
draw conclusions.
LEARNING OBJECTIVE
SPS-7.A
Explain how the Industrial
Revolution facilitated the
growth and diusion of
industrialization.
ESSENTIAL KNOWLEDGE
SPS-7.A.1
Industrialization began as a result of new
technologies and was facilitated by the
availability of natural resources.
SPS-7.A.2
As industrialization spread it caused food
supplies to increase and populations to grow; it
allowed workers to seek new industrial jobs in
the cities and changed class structures.
SPS-7.A.3
Investors in industry sought out more
raw materials and new markets, a factor
that contributed to the rise of colonialism
andimperialism.
ENDURING UNDERSTANDING
SPS-7
Industrialization, past and present, has facilitated improvements in standards of
living, but it has also contributed to geographically uneven development.
Return to Table of Contents
© 2020 College Board
Course Framework V.1
|
124
AP Human GeographyCourse and Exam Description
UNIT
7
Industrial and Economic Development Patterns and Processes
LEARNING OBJECTIVE
SPS-7.B
Explain the spatial patterns
of industrial production and
development.
ESSENTIAL KNOWLEDGE
SPS-7.B.1
The dierent economic sectors—including
primary, secondary, tertiary, quaternary,
and quinary—are characterized by distinct
development patterns.
SPS-7.B.2
Labor, transportation (including shipping
containers), the break-of-bulk point, least
cost theory, markets, and resources inuence
the location of manufacturing such as core,
semiperiphery, and periphery locations.
ENDURING UNDERSTANDING
SPS-7
Industrialization, past and present, has facilitated improvements in standards of
living, but it has also contributed to geographically uneven development.
Required Course Content
SUGGESTED SKILL
Spatial
Relationships
2.B
Explain spatial relationships
in a specified context or
region of the world, using
geographic concepts,
processes, models,
ortheories.
TOPIC 7.2
Economic Sectors
and Patterns
Return to Table of Contents
© 2020 College Board
Course Framework V.1
|
125
AP Human GeographyCourse and Exam Description
UNIT
7
Industrial and Economic Development Patterns and Processes
Required Course Content
SUGGESTED SKILL
Data Analysis
3.F
Explain possible limitations
of the data provided.
TOPIC 7.3
Measures of
Development
LEARNING OBJECTIVE
SPS-7.C
Describe social and
economic measures of
development.
ESSENTIAL KNOWLEDGE
SPS-7.C.1
Measures of social and economic development
include Gross Domestic Product (GDP); Gross
National Product (GNP); and Gross National
Income (GNI) per capita; sectoral structure of
an economy, both formal and informal; income
distribution; fertility rates; infant mortality rates;
access to health care; use of fossil fuels and
renewable energy; and literacy rates.
SPS-7.C.2
Measures of gender inequality, such as
the Gender Inequality Index (GII), include
reproductive health, indices of empowerment,
and labor-market participation.
SPS-7.C.3
The Human Development Index (HDI)
is a composite measure used to show
spatial variation among states in levels
ofdevelopment.
ENDURING UNDERSTANDING
SPS-7
Industrialization, past and present, has facilitated improvements in standards of
living, but it has also contributed to geographically uneven development.
Return to Table of Contents
© 2020 College Board
Course Framework V.1
|
126
AP Human GeographyCourse and Exam Description
UNIT
7
Industrial and Economic Development Patterns and Processes
TOPIC 7.4
Women
and Economic
Development
SUGGESTED SKILL
Data Analysis
3.D
Compare patterns and
trends in maps and in
quantitative and geospatial
data to draw conclusions.
LEARNING OBJECTIVE
SPS-7.D
Explain how and to what
extent changes in economic
development have
contributed to gender parity.
ESSENTIAL KNOWLEDGE
SPS-7.D.1
The roles of women change as countries
develop economically.
SPS-7.D.2
Although there are more women in the
workforce, they do not have equity in wages or
employment opportunities.
SPS-7.D.3
Microloans have provided opportunities for
women to create small local businesses, which
have improved standards of living.
ENDURING UNDERSTANDING
SPS-7
Industrialization, past and present, has facilitated improvements in standards of
living, but it has also contributed to geographically uneven development.
Required Course Content
AVAILABLE RESOURCES
§ Classroom Resources >
Maps and Spatial
Thinking Skills in the
AP Human Geography
Classroom
Return to Table of Contents
© 2020 College Board
Course Framework V.1
|
127
AP Human GeographyCourse and Exam Description
UNIT
7
Industrial and Economic Development Patterns and Processes
Required Course Content
TOPIC 7.5
Theories of
Development
LEARNING OBJECTIVE
SPS-7.E
Explain dierent theories
of economic and social
development.
ESSENTIAL KNOWLEDGE
SPS-7.E.1
Dierent theories, such as Rostow’s Stages of
Economic Growth, Wallerstein’s World System
Theory, dependency theory, and commodity
dependence, help explain spatial variations
indevelopment.
ENDURING UNDERSTANDING
SPS-7
Industrialization, past and present, has facilitated improvements in standards of
living, but it has also contributed to geographically uneven development.
SUGGESTED SKILL
Concepts and
Processes
1.E
Explain the strengths,
weaknesses, and limitations
of different geographic
models and theories in a
specified context.
Return to Table of Contents
© 2020 College Board
Course Framework V.1
|
128
AP Human GeographyCourse and Exam Description
UNIT
7
Industrial and Economic Development Patterns and Processes
Required Course Content
SUGGESTED SKILL
Scale Analysis
5.B
Explain spatial relationships
across various geographic
scales using geographic
concepts, processes,
models, or theories.
TOPIC 7.6
Trade and the
World Economy
LEARNING OBJECTIVE
PSO-7.A
Explain causes and
geographic consequences
of recent economic changes
such as the increase
in international trade,
deindustrialization, and
growing interdependence in
the world economy.
ESSENTIAL KNOWLEDGE
PSO-7.A.1
Complementarity and comparative advantage
establish the basis for trade.
PSO-7.A.2
Neoliberal policies, including free trade
agreements, have created new organizations,
spatial connections, and trade relationships,
such as the EU, World Trade Organization
(WTO), Mercosur, and OPEC, that foster greater
globalization.
PSO-7.A.3
Government initiatives at all scales may aect
economic development, including taris.
PSO-7.A.4
Global nancial crises (e.g., debt crises),
international lending agencies (e.g., the
International Monetary Fund), and strategies of
development (e.g., microlending) demonstrate
how dierent economies have become more
closely connected, even interdependent.
ENDURING UNDERSTANDING
PSO-7
Economic and social development happen at dierent times and rates in
dierentplaces.
AVAILABLE RESOURCES
§ Classroom Resources >
Globalization
§ Classroom Resources >
Scale
Return to Table of Contents
© 2020 College Board
Course Framework V.1
|
129
AP Human GeographyCourse and Exam Description
UNIT
7
Industrial and Economic Development Patterns and Processes
Required Course Content
TOPIC 7.7
Changes as a
Result of the
World Economy
LEARNING OBJECTIVE
PSO-7.A
Explain causes and
geographic consequences
of recent economic changes
such as the increase
in international trade,
deindustrialization, and
growing interdependence in
the world economy.
ESSENTIAL KNOWLEDGE
PSO-7.A.5
Outsourcing and economic restructuring
have led to a decline in jobs in core
regions and an increase in jobs in newly
industrializedcountries.
PSO-7.A.6
In countries outside the core, the growth of
industry has resulted in the creation of new
manufacturing zones—including special
economic zones, free-trade zones, and export-
processing zones—and the emergence of
an international division of labor in which
developing countries have lower-paying jobs.
PSO-7.A.7
The contemporary economic landscape has
been transformed by post-Fordist methods of
production, multiplier eects, economies of
scale, agglomeration, just-in-time delivery, the
emergence of service sectors, high technology
industries, and growth poles.
ENDURING UNDERSTANDING
PSO-7
Economic and social development happen at dierent times and rates in
dierentplaces.
SUGGESTED SKILL
Source Analysis
4.F
Explain possible limitations
of visual sources provided.
Return to Table of Contents
© 2020 College Board
Course Framework V.1
|
130
AP Human GeographyCourse and Exam Description
UNIT
7
Industrial and Economic Development Patterns and Processes
Required Course Content
SUGGESTED SKILL
Scale Analysis
5.D
Explain the degree to which
a geographic concept,
process, model, or theory
effectively explains
geographic effects across
various geographic scales.
TOPIC 7.8
Sustainable
Development
LEARNING OBJECTIVE
IMP-7.A
Explain how sustainability
principles relate to and
impact industrialization and
spatial development.
ESSENTIAL KNOWLEDGE
IMP-7.A.1
Sustainable development policies attempt
to remedy problems stemming from natural-
resource depletion, mass consumption,
the eects of pollution, and the impact of
climatechange.
IMP-7.A.2
Ecotourism is tourism based in natural
environments—often environments that
are threatened by looming industrialization
or development—that frequently helps to
protect the environment in question while also
providing jobs for the local population.
IMP-7.A.3
The UN’s Sustainable Development Goals
help measure progress in development,
such as small-scale nance and public
transportationprojects.
ENDURING UNDERSTANDING
IMP-7
Environmental problems stemming from industrialization may be remedied through
sustainable development strategies.
AVAILABLE RESOURCES
§ Classroom Resources >
Scale
Return to Table of Contents
© 2020 College Board
Course Framework V.1
|
131
AP Human GeographyCourse and Exam Description
THIS PAGE IS INTENTIONALLY LEFT BLANK.
Instructional
Approaches
AP HUMAN GEOGRAPHY
Selecting and
Using Course Materials
Using a wide array of source materials will help students become procient with the
course skills and develop a conceptual understanding of geography as a eld of inquiry.
In addition to using a college-level textbook that will provide required course content,
it’s a good idea to provide regular opportunities for students to examine primary and
secondary source material in various forms, including graphic content (maps, graphs,
charts, photographs), since many geographic concepts are presented spatially in
such formats. Using rich, diverse source materials encourages students to engage in
learning about the world they live in and develop the habits of thinking like a geographer
that are essential for student success in the course.
Textbooks
The textbook used for this course should be written at the level of an introductory
college course. Because skills are fundamental to the teaching of the course, a
textbook that focuses on skill instruction would be a plus.
Although nearly all college-level human geography textbooks address the seven units of
the AP Human Geography course, no single textbook covers the complete curriculum as
described in the framework. It is helpful to obtain and consult multiple textbooks for the
introductory human geography course; doing so allows you to better address curricular
gaps in the textbook you choose for the AP Human Geography course. College-level
human geography textbooks organize content in various ways; if you are following the
unit structure of the course framework, you might need to teach the textbook chapters
out of order. Ultimately, it is best to select a textbook that closely aligns with the course
framework and the course skills.
On the AP Central Course Audit page for this course, you’ll nd an example textbook
list of college-level textbooks that meet the resource requirements.
TEXT-BASED QUALITATIVE SOURCES
(PRIMARY AND SECONDARY SOURCES)
Students in this AP course are required to analyze primary and secondary source
material in order to deepen their understanding of the key concepts addressed by the
textbook and to practice the required course skills.
Student success in college also depends on exposure to and analysis of multiple
secondary sources in which geographers present and explain their arguments. For
secondary sources, share with your students editorials, journal articles, news media
articles, and essays and books by geographers, choosing a variety of sources that
represent dierent points of view and that will engage students while enhancing
learning. You can add new articles to keep up with rapidly changing events.
Return to Table of Contents
© 2020 College Board
Instructional Approaches V.1
|
135
AP Human GeographyCourse and Exam Description
QUANTITATIVE SOURCES
Geography courses require students to engage with data in a variety of ways. The
analysis, interpretation, and application of quantitative information are vital skills for
students in AP Human Geography. New textbooks and publishers’ resources often
contain quantitative information presented via charts, graphs, or other infographics,
but those data can quickly become out of date. Research institutions such as the
Pew Research Center are good resources for current data with which to practice
quantitative skills with students.
VISUAL SOURCES
Students will encounter a variety of visual sources on the AP Human Geography Exam.
Some visuals contain data, such as a chart showing aging population by country
across time periods. Other visuals may illustrate geographic principles, processes,
or behaviors, such as topographical maps that demonstrate dierences in scale. It is
important to give students practice with a wide variety of visual sources in addition to
those maps or cartoons in their textbook. The following resources provide a wealth of
visuals to choose from for bell-ringer exercises and skillspractice.
§ The National Geologic Map Database is a national archive of standardized maps.
§ Gapminder is a “fact tank” that oers free teaching resources based on statistics
culled from universities, the United Nations, and non-governmental organizations.
§ Worldmapper is a collection of world maps called cartograms, on which the size of
dierent areas is scaled in proportion to a particular variable.
Return to Table of Contents
© 2020 College Board
AP Human GeographyCourse and Exam Description Instructional Approaches V.1
|
136
Evaluating the Quality and Credibility of Dierent
Sources andPerspectives
Students have unprecedented access to information, so it is more important than ever
to help them determine the credibility of sources. The course skills require students to
evaluate the quality and credibility of dierent information sources and perspectives
and derive supportable conclusions. Consider providing students with a list of news
media websites that oer dierent perspectives.
The table below explains how the credibility and/or reliability of various sources might
be addressed.
Criteria Considerations Examples and Illustrations
Credibility
§ Credibility requires students to obtain
information from sources with knowledge
and expertise in human geography. Sources
can be print, online, or expert interviews.
§ Because many students are using the
internet or database sources, they should
consider thefollowing:
The domain name extensions, which
indicate who publishes and owns the
domain
The author(s) of the website
The credibility of sources cited in the
materials as well as any websites they
link to
§ Commonly used extensions include:
.edu (educational organization);
.com (company);
.org (any organization);
.gov (government agency);
.net (network)
§ Read past the rst slash in the domain name
to see if the page might be someone’s
personal page. A personal page might be
lesscredible or biased.
§ Investigate who the authors are, their
credentials, and the organization(s) they
are associated with to determine how the
information should be viewed.
§ An article that includes citations of sources
is often more credible.
§ Design a lesson to help students identify
examples of cultural relativism and
ethnocentrism by comparing the perspectives
of various sources.
§ Have students examine the language used in
the lead stories, as well as the pictures that
accompany them.
§ Ask students to compare the focus of various
sources, such as news accounts or scholarly
articles, and how characteristics, attitudes,
and traits inuence the study of culture.
§ Have students synthesize their ndings in a
short paper examining how cultural traits vary
across societies.
continued on next page
Return to Table of Contents
© 2020 College Board
Instructional Approaches V.1
|
137
AP Human GeographyCourse and Exam Description
Criteria Considerations Examples and Illustrations
Reliability
and
Relevance
§ Students should examine the content of a
source (the evidence) to ensure it supports
their claims and provides insight and
knowledge that relates to the topic. This
means that evidence is only relevant when it
addresses both the topic in context and the
student’s argument.
§ Because we are investigating geographical
processes and outcomes that are changing
and evolving, it is important to consider
whether the information being provided is
the most current.
§ A biographical article about the life of Thomas
Malthus may not be relevant in supporting a
claim about a particular alternative viewpoint
on eects of population increase.
§ A country case study from 2000 might not be
relevant in 2018.
Return to Table of Contents
© 2020 College Board
AP Human GeographyCourse and Exam Description Instructional Approaches V.1
|
138
Instructional Strategies
The AP Human Geography course framework outlines the concepts and skills students
must master in order to be successful on the AP Exam. Youcan help students develop
mastery of the course skills by engaging them in learning activities that allow them to
apply their understanding of course concepts. The following strategies, organized in
alphabetical order, might help you to plan your instruction.
Strategy Denition Purpose Example
Create
Representations
Students create
tables, graphs, or other
infographics to interpret
text or data.
Helps students organize
information using multiple
ways to present data.
Give students a set of data,
such as socioeconomic status
and ethnicity in megacities, and
have them create a graph that
best shows the data and the
trends.
Critique
Reasoning
Through collaborative
discussion, students
critique the arguments
of others, questioning
the author’s perspective,
evidence presented, and
reasoning behind the
argument.
Helps students learn
from others as they make
connections between
concepts and learn to
support their arguments
with evidence and reasoning
that make sense to peers.
Have students critique an
argument from a secondary
source. Ask students to examine
the author’s perspective and
the evidence and reasoning
he or she uses to support that
position.
Close Reading
Reading, rereading, and
analyzing small chunks
of text word for word,
sentence by sentence, and
line by line.
Develops comprehensive
understanding of text.
When students are reading
primary or secondary sources,
such as the National Geographic
article “Unique Speak,” have
them highlight relevant words
and passages that support the
author’s claim. Ask students to
connect the key concepts of the
article to the big ideas in Unit 3:
Cultural Patterns and Processes.
Debate
Presenting an informal
or formal argument that
defends a claim with
reasons, while others defend
dierent claims about the
same topic or issue. The
goal is to debate ideas
without attacking the people
who defend those ideas.
Gives students an
opportunity to collect and
orally present evidence
supporting the arguments
for or against a proposition
or issue.
Have students debate the
advantages and disadvantages
of having a nation-state, using
evidence and reasoning to
support their claims.
continued on next page
Return to Table of Contents
© 2020 College Board
Instructional Approaches V.1
|
139
AP Human GeographyCourse and Exam Description
Strategy Denition Purpose Example
Debrieng
Teachers facilitate a
discussion that leads to
consensus understanding.
Helps students clarify
misconceptions and deepen
understanding of content.
For complex issues, such as the
global system of agriculture and
how it is impacted by economic
and political structures, lead
students in a debrief to ensure
understanding.
Discussion
Group
Students engage in an
interactive small-group
discussion.
Helps students gain
information and
understanding about a
concept, idea, or problem.
Use a learning objective, such
as Explain how landscape
features and land and resource
use reect cultural beliefs and
identities, to help students gain
information and show their
understanding.
Fishbowl
Discussing specic
topics within groups:
some students form an
inner circle and model
appropriate discussion
techniques, while an outer
circle of students listen,
respond, and evaluate.
Provides students with an
opportunity to engage in
a formal discussion and
to experience the roles of
both participant and active
listener. Students also
have the responsibility of
supporting their opinions
and responses using
specic textual evidence.
Use this strategy to discuss
the arguments presented in
secondary sources and how
they relate to the big ideas of
the course.
Graphic
Organizer
Representing ideas and
information visually (e.g.,
Venn diagrams, owcharts,
cluster maps).
Provides students with a
visual system for organizing
multiple ideas, details,
or textual support to be
included in a piece of
writing.
Graphic organizers can be
helpful when comparing global,
national, regional, and local
geographic patterns. Make sure
students use any organizer
thoughtfully and do not simply
“ll in the blanks.”
Guided
Discussion
An umbrella strategy
that allows for the use of
dierent techniques to
guide students through
the lesson.
Helps students see the
big picture and builds their
condence when dealing
with dicult content or
new skills.
Use brainstorming and
quickwrite as strategies during
a guided discussion to help
students understand how
language, ethnicity, and religion
act as both unifying and
divisive forces.
Jigsaw
Each student in a group
reads a dierent text or
dierent passage from
one text, taking on the
role of “expert” on what
was read. Students share
the information from that
reading with students
from other groups and
then return to their original
groups to share their
new knowledge.
Helps students summarize
and present information
to others in a way that
facilitates an understanding
of a text (or multiple texts) or
issue without having each
student read the text in its
entirety; by teaching others,
they become experts.
Use this strategy to facilitate
understanding of high-level
readings or readings that
incorporate several geographic
concepts, such as The Spanish
Flu and Its Legacy, which is
available on AP Central Human
Geography under “Classroom
Resources.”
continued on next page
Return to Table of Contents
© 2020 College Board
AP Human GeographyCourse and Exam Description Instructional Approaches V.1
|
140
Strategy Denition Purpose Example
Look for a
Pattern
Students evaluate
data or create visual
representations to nd
a trend.
Helps students to identify
patterns that may be used
to draw conclusions.
Using a complex graph (with
at least two data sets) or map,
have students compare the
data to nd a trend and draw
a conclusion.
Making
Connections
Students are given a
concept, term, or document
and asked to write what
they know about it. Then,
students are paired and
asked to determine,
describe, and explain the
connection between the
two concepts.
Reinforces the fact that
course concepts are often
connected and provides the
opportunity for students
to make and explain
connections between and
among these concepts.
Write concepts related to one of
the course big ideas on cards,
place them in a box, and have
students pick a card at random.
Give students a few minutes to
gather and recall information
about the concept, and then
pair students and ask them to
nd the connection between
their concepts. Finally, ask the
pairs to write a brief explanation
of how the concepts are related.
Quickwrite
Students write for a short,
specic amount of time
about a designated topic.
Helps generate ideas in a
short time.
As preparation for the free-
response questions, have
students write to describe,
explain, or apply a geographic
situation or scenario. Over the
course of the academic year,
add stimuli to these free-
response prompts.
Self/Peer Review
Working alone or with a
partner to examine a piece
of writing for accuracy
and clarify.
Provides students with
an opportunity to edit a
written text to ensure the
correctness of identied
components.
Students should learn to
interpret, evaluate, respond
to, and write about a variety
of written texts and graphic
images. After students
have completed a writing
assignment, have them
perform self and peer revisions
of their writing.
Shared Inquiry
Students read a provocative
text and are asked
interpretive questions
(for which there are no
predetermined “right”
answers). Students who
oer dierent responses
debate one another,
supporting their positions
with specic evidence from
the text.
To lead students in a deep
discussion of a text and
encourage a diversity of
ideas to emerge as students
think deeply and share
interpretations.
Ask students to read Edward
Relph’s “Sense of Place” and
discuss the geographical sense
of place and its components.
Have students evoke a sense of
place within their personal and
shared communities.
continued on next page
Return to Table of Contents
© 2020 College Board
Instructional Approaches V.1
|
141
AP Human GeographyCourse and Exam Description
Strategy Denition Purpose Example
Socratic
Seminar
This is a focused discussion
on a topic, essential
question, or selected text
in which students ask
questions of one another.
The questions initiate a
conversation that continues
with a series of responses
and additional questions.
Helps students arrive
at a new understanding
by asking questions
that clarify; challenging
assumptions; probing
perspective and point of
view; questioning facts,
reasons, and evidence; or
examining implications and
outcomes.
Initiate a discussion of an
essential question, such as
What are the consequences,
including positive, negative,
and unintended, of each type of
population policy? during which
students can illustrate their
understanding of the learning
objectives and essential
knowledge statements.
Think-Pair-
Share
Students think through a
question or issue alone,
pair with a partner to share
ideas, and then share
results with the class.
Enables the development
of initial ideas that are then
tested with a partner in
preparation for revising
ideas and sharing them with
a larger group.
To practice comparing the
spatial variations of theories of
development, ask students to
use Think-Pair-Share to come
up with similarities, dierences
and relevance to geographic
processes for Rostow’s
Stages of Economic Growth,
Wallerstein’s World System
Theory, dependency theory, and
commodity dependence.
Return to Table of Contents
© 2020 College Board
AP Human GeographyCourse and Exam Description Instructional Approaches V.1
|
142
Developing the
Course Skills
Throughout the AP Human Geography course, students will develop skills that are
fundamental to the discipline of geography. Since these are the skills that adept
geographers demonstrate, students will benet from multiple opportunities to develop
them in a scaolded manner. Through the use of guided questioning, discussion
techniques, and other instructional strategies, students can practice applying these skills
in new contexts, providing an important foundation for their college and career readiness.
The skill categories articulated in the course framework equip students to understand,
analyze, and apply geographic information in a process similar to that followed by
geographers. This process involves the analysis of spatial relationships, quantitative
and qualitative sources, and scale.
The unit guides feature a suggested skill that students can practice using the content
of each topic. You are free to use that structure or create a system of your own, helping
students develop skills throughout the course through the sequencing and scaolding
appropriate for them.
The tables on the pages that follow look at each of the skills and provide examples
of questions and instructional strategies for incorporating those skills into
your instruction.
Return to Table of Contents
© 2020 College Board
Instructional Approaches V.1
|
143
AP Human GeographyCourse and Exam Description
Skill Category 1: Analyze geographic theories, approaches, concepts,
processes, or models in theoretical and applied contexts.
Students should be given ample practice to apply
their content knowledge. Geographers practice their
discipline by applying their knowledge to dierent
contexts and scenarios.
The table that follows provides examples of activities and
instructional strategies for teaching students to analyze
geographic theories, approaches, concepts, processes,
or models in theoretical and applied contexts.
continued on next page
Skill Category 1: Concepts and Processes (cont'd)
Skill Key Tasks Sample Activities
Instructional
Strategies
1.A: Describe geographic
concepts, processes,
models, and theories.
§ Describe the
characteristics and
traits of course
aspects, such as
economic and social
development.
Before beginning a deeper
discussion on theories of
development, have students
describe some dierent
theories and how they vary.
Graphic Organizer
1.B: Explain geographic
concepts, processes,
models, and theories.
§ Explain why there
are concerns about
development and
sustainability.
§ Explain how
sustainability principles
relate to and impact
industrialization and
spatial development.
Ask students to explain the
interplay of the processes of
development and principles
of sustainability. What are
examples of sustainable
development policies?
Debrieng
Discussion Group
1.C: Compare
geographic concepts,
processes, models, and
theories.
§ Identify the relevant,
specic categories for
comparing similarities
and dierences.
§ Explain the reasons for
the relevant similarities
and dierences.
§ Explain the relevance,
implications, and/
or signicance of
the similarities
and dierences.
Ask students to identify and
explain relevant, specic
similarities and dierences
between federal and
unitary states.
Make sure students
understand that they need to
identify relevant and specic
similarities and dierences.
They should compare
like factors.
Graphic Organizer
Look for a Pattern
Think-Pair-Share
Return to Table of Contents
© 2020 College Board
AP Human GeographyCourse and Exam Description Instructional Approaches V.1
|
144
Skill Category 1: Concepts and Processes (cont'd)
Skill Key Tasks Sample Activities
Instructional
Strategies
1.D: Describe a relevant
geographic concept,
process, model, or
theory in a specied
context.
§ Describe how
geographic concepts,
processes, models, or
theories have changed
over time.
§ Describe the reasons
for causes and eects.
§ Describe the concepts
using real-world
scenarios.
Ask students to explain how a
geographic process, such as
migration, has changed over
time.
Have students explain the
reasons why geographic
theories, such as
environmental determinism,
have changed.
Ask students to use real-world
scenarios to describe how
geographic models, such as
the demographic transition
model, have changed.
Debrieng
Discussion Groups
Quickwrite
1.E: Explain the
strengths, weaknesses,
and limitations of
dierent geographic
models and theories in a
specied context.
§ Explain how geographic
models and theories
have changed over
time.
§ Explain the implications
of this change over
time.
§ Identify and explain
the reasons for causes
and eects.
Have students examine a
geographic model, such as the
von Thünen model, and explain
its strengths, weaknesses, or
limitations. Also, explain how
that model has changed over
time.
Ask students to explain the
implications of a geographic
theory, such as the Malthusian
theory.
Discussion Groups
Look for a Pattern
Think-Pair-Share
Return to Table of Contents
© 2020 College Board
Instructional Approaches V.1
|
145
AP Human GeographyCourse and Exam Description
Skill Category 2: Analyze geographic patterns, relationships,
andoutcomes in applied contexts.
Geographers seek to understand the associations and
networks among phenomena in particular places and to
explain their implications. You can help students come
to understandings about course concepts by giving
them ample practice analyzing geographic patterns,
relationships, and outcomes.
The table that follows provides examples of questions,
activities, and instructional strategies for teaching
students to analyze geographic patterns, relationships,
and outcomes in dierent locations or at dierent times.
Skill Category 2: Spatial Relationships (cont'd)
Skill Key Questions Sample Activities
Instructional
Strategies
2.A: Describe spatial
patterns, networks, and
relationships.
§ What are spatial
patterns, networks, and
relationships?
Using maps or quantitative
or geospatial data about a
topic such as population
distribution, ask students to
describe spatial patterns.
Look for a Pattern
2.B: Explain spatial
relationships in a
specied context or
region of the world,
using geographic
concepts, processes,
models, or theories.
§ How and why do spatial
patterns, networks,
and relationships occur
dierently in various
parts of the world?
Ask students to identify
general factors that inuence
migration. Then ask them to
categorize the factors. Finally,
have students list factors
associated with dierent
regions and explain why they
are applicable there.
Graphic Organizer
Think-Pair-Share
2.C: Explain a likely
outcome in a geographic
scenario, using
geographic concepts,
processes, models, or
theories.
§ What outcome do you
predict in a particular
geographic scenario?
§ Why do you think that
outcome would occur
in that geographic
scenario?
Present students with
real-world scenarios from a
geographic scenario such
as territorial disputes among
sovereign states. Ask them to
predict and explain outcomes
using geographic concepts,
processes, models, or
theories.
Debrieng
2.D: Explain the
signicance of
geographic similarities
and dierences among
dierent locations and/
or at dierent times.
§ What are the similarities
and/or dierences of
dierent locations at
dierent times?
§ How and why do those
similarities and/or
dierences of dierent
locations manifest
themselves at dierent
times?
§ Why does the similarity
or dierence matter?
Look at the forms of
governance, historical and
present, for countries in
a particular world region.
What geographic factors
inuenced the formation and
change of governments?
How do these governments
reect geographic concepts,
processes, models, or
theories?
Graphic Organizer
Jigsaw
continued on next page
Return to Table of Contents
© 2020 College Board
AP Human GeographyCourse and Exam Description Instructional Approaches V.1
|
146
Skill Category 2: Spatial Relationships (cont'd)
Skill Key Questions Sample Activities
Instructional
Strategies
2.E: Explain the degree
to which a geographic
concept, process, model,
or theory eectively
explains geographic
eects in dierent
contexts and regions of
the world.
§ How and why does a
geographic concept,
process, model,
or theory explain
geographic eects in
dierent contexts and
regions of the world?
§ To what extent does a
geographic concept,
process, model,
or theory explain
geographic eects in
dierent contexts and
regions of the world?
Ask students to discuss
population increase and its
checks from a Malthusian
perspective. What are some
alternative viewpoints to
Malthusian theory?
Critique Reasoning
Debate
Return to Table of Contents
© 2020 College Board
Instructional Approaches V.1
|
147
AP Human GeographyCourse and Exam Description
Skill Category 3: Analyze and interpret quantitative geographic
datarepresented in maps, tables, charts, graphs, satellite images,
and infographics.
The analysis, interpretation, and application of
quantitative information are vital skills for students in
AP Human Geography. Analysis can be taught using any
type of quantitative information, but students should be
provided with current data so that they can draw accurate
conclusions and apply that knowledge to the enduring
understandings and learning objectives in the curriculum.
The table that follows provides examples of questions,
activities, and instructional strategies for teaching
students to analyze and interpret quantitative
geographic data represented in maps, tables, charts,
graphs, satellite images, and infographics.
Skill Category 3: Data Analysis
Skill Key Questions Sample Activities
Instructional
Strategies
3.A: Identify the dierent
types of data presented in
maps and in quantitative
geospatial data.
What do the data show? Present students with maps
that showcase dierent
variables, such as population
by demographics (e.g., age,
income, education). Ask them
to identify the categories of
data presented in the maps.
Debrieng
3.B: Describe spatial
patterns presented
in maps and in
quantitative and
geospatial data.
What patterns can you
identify from the data?
Ask students to describe
patterns in the data, such
as land-use patterns and
practices in dierent
agricultural production regions.
Look for a Pattern
3.C: Explain patterns
and trends in maps
and in quantitative and
geospatial data to draw
conclusions.
How does the trend in the
data inform your conclusion?
Why does the trend in the data
support your conclusion?
What is the reasoning behind
your conclusion?
Ask students to identify
multiple patterns or trends
within a data set and then
explain how and why they are
related.
Debrieng
3.D: Compare patterns
and trends in maps
and in quantitative and
geospatial data to draw
conclusions.
What conclusions can you
draw by comparing the trends
you found in the data?
How does the pattern or trend
in one data set compare to
that in another data set?
Ask students to compare
the trends and draw a
conclusion about the data.
They should also explain
their reasoning.
Create Representations
Look for a Pattern
3.E: Explain what the
data imply or illustrate
about geographic
principles, processes,
behaviors, and outcomes.
How do these data relate to a
geographic process?
Have students apply the
information gleaned from
data sources to a geographic
principle, process, behavior,
or outcome.
Create Representations
Look for a Pattern
3.F: Explain possible
limitations of the data
provided.
What do the data not tell
you?
Lead students in analyzing
the possible limitations of the
data and what is not revealed.
Create Representations
Look for a Pattern
Return to Table of Contents
© 2020 College Board
AP Human GeographyCourse and Exam Description Instructional Approaches V.1
|
148
Skill Category 4: Analyze and interpret qualitative geographic
information represented in maps, images, and landscapes.
The AP Human Geography course requires students
to analyze visual sources in order to understand
patterns and trends and how they illustrate geographic
principles, behaviors, and outcomes.
The following table provides examples of questions
and instructional strategies for teaching students
to analyze and interpret qualitative geographic
information represented in maps, images (e.g., satellite,
photographs, cartoons), and landscapes.
Skill Category 4: Source Analysis (cont'd)
Skill Sample Activity Key Questions
Instructional
Strategies
4.A: Identify the dierent
types of information
presented in visual
sources.
Provide students with a
map, image, or landscape to
address the key questions.
What is being shown in the
map, image, or landscape?
What perspective is being
shown in the map or image?
Debrieng
Quickwrite
Provide students with written
accounts to address key
questions.
What is being presented in
this written account?
Who created this written
account?
What perspective can you
identify?
Critique Reasoning
Quickwrite
4.B: Describe the spatial
patterns presented in
visual sources.
Provide students with a
map, image, or landscape to
address the key questions.
What spatial patterns are
depicted in this map, image,
or landscape?
Look for a Pattern
Provide students with written
accounts to address key
questions.
What spatial pattern is
articulated in this written
account?
Critique Reasoning
4.C: Explain patterns
and trends in visual
sources to draw
conclusions.
Provide students with a
map, image, or landscape to
address the key questions.
How and why does this
pattern or trend emerge
in this map, image, or
landscape?
Create Representations
Think-Pair-Share
Provide students with written
accounts to address key
questions.
How and why does this
pattern or trend emerge in
this written account?
Create Representations
Think-Pair-Share
continued on next page
Return to Table of Contents
© 2020 College Board
Instructional Approaches V.1
|
149
AP Human GeographyCourse and Exam Description
Skill Category 4: Source Analysis (cont'd)
Skill Sample Activity Key Questions
Instructional
Strategies
4.D: Compare patterns
and trends in visual
sources to draw
conclusions.
Provide students with a
map, image, or landscape to
address the key questions.
How do the patterns and
trends in one map, image,
or landscape compare
to patterns and trends in
another map, image, or
landscape?
What conclusions can you
draw about patterns and
trends when comparing
dierent maps, images, or
landscapes?
Graphic Organizer
Quickwrite
Provide students with written
accounts to address key
questions.
How does the pattern and
trend in one written account
compare to patterns and
trends in another written
account?
What conclusions can you
draw about patterns and
trends when comparing
dierent written accounts?
Jigsaw
Debate
4.E: Explain how maps,
images, and landscapes
illustrate or relate to
geographic principles,
processes, and
outcomes.
Provide students with a
map, image, or landscape to
address the key questions.
How does what you see
relate to a geographic
principle, process, or
outcome?
Quickwrite
Think-Pair-Share
Provide students with written
accounts to address key
questions.
How does what you read
relate to a geographic
principle, process, or
outcome?
Critique Reasoning
Jigsaw
4.F: Explain possible
limitations of visual
sources provided.
Provide students with a
map, image, or landscape to
address the key questions.
What does the map, image,
or landscape not show?
Debate
Quickwrite
Provide students with written
accounts to address key
questions.
What does the written
account not say?
Critique Reasoning
Debate
Return to Table of Contents
© 2020 College Board
AP Human GeographyCourse and Exam Description Instructional Approaches V.1
|
150
Skill Category 5: Analyze geographic theories, approaches,
concepts, processes, and models across geographic scales
to explain spatial relationships.
Geographic analysis requires a sensitivity to scale,
not just as a spatial category but as a framework for
understanding how events and processes at dierent
scales inuence one another and change according to
the scale of analysis.
The table that follows provides examples of questions,
activities, and instructional strategies for teaching
students to analyze geographic theories, approaches,
concepts, processes, and models across geographic
scales to explain spatial relationships.
Skill Category 5: Scale Analysis
Skill Key Questions Sample Activities
Instructional
Strategies
5.A: Identify the scales
of analysis presented by
maps, quantitative and
geospatial data, images,
and landscapes.
§ What are the dierent
scales of analysis?
§ Why is it important to
look at data through
dierent scales?
Take a set of data and show
students what the global,
national, regional, and local
patterns are. Have them repeat
the exercise with dierent
data.
Graphic Organizer
Look for a Pattern
5.B: Explain spatial
relationships across
various geographic
scales using geographic
concepts, processes,
models, or theories.
§ What do scales of
analysis reveal about
spatial relationships?
§ How do geographic
concepts, processes,
models, or theories
impact spatial
relationships?
Ask students to articulate
characteristics of spatial
relationships at various levels
of scale. Ask them to explain
why those trends occur.
Quickwrite
Think-Pair-Share
5.C: Compare
geographic
characteristics and
processes at various
scales.
§ How do geographers
apply scale analysis at
dierent levels?
§ How do data at one
level of scale analysis
compare to the data at
a dierent level?
Identify several geographic
processes. Using associated
data, scale up and down to
see patterns and trends at the
various levels.
Graphic Organizer
Look for a Pattern
5.D: Explain the degree
to which a geographic
concept, process, model,
or theory eectively
explains geographic
eects across various
geographic scales.
§ How and why does a
geographic concept,
process, model,
or theory explain
geographic processes?
§ To what extent does a
geographic concept,
process, model,
or theory explain
geographic processes?
Present students with
real-world scenarios, such
as maritime boundary
disputes, and ask them to use
economic or other data for
decision-making purposes.
Have students justify their
decisions.
Debate
Jigsaw
Return to Table of Contents
© 2020 College Board
Instructional Approaches V.1
|
151
AP Human GeographyCourse and Exam Description
THIS PAGE IS INTENTIONALLY LEFT BLANK.
Exam
Information
AP HUMAN GEOGRAPHY
Exam Overview
The AP Human Geography Exam assesses student understanding of the
skills and learning objectives outlined in the course framework. The exam is
2 hours and 15 minutes long and includes 60 multiple-choice questions and
3 free-response questions. The details of the exam, including exam weighting
and timing, can be found below:
Section Question Type Number of Questions
Exam
Weighting Timing
I
Multiple-choice
questions
60 50% 60 minutes
II
Free-response questions
(7 points each)
3 50% 75 minutes
The exam assesses content from each of the three big ideas forthe course:
1. Patterns and Spatial Organization
2. Impacts and Interactions
3. Spatial Processes and Societal Change
Return to Table of Contents
© 2020 College Board
Exam Information V.1
|
155
AP Human GeographyCourse and Exam Description
The exam also assesses each of the seven units of the course with the following relative weightings on the
multiple-choice section of the AP Exam:
Units Exam Weighting
Unit 1: Thinking Geographically
8–10%
Unit 2: Population and Migration Patterns and Processes
12–17%
Unit 3: Cultural Patterns and Processes
12–17%
Unit 4: Political Patterns and Processes
12–17%
Unit 5: Agriculture and Rural Land-Use Patterns and Processes
12–17%
Unit 6: Cities and Urban Land-Use Patterns and Processes
12–17%
Unit 7: Industrial and Economic Development Patterns and Processes
12–17%
Return to Table of Contents
© 2020 College Board
AP Human GeographyCourse and Exam Description Exam Information.1
|
156
How Student Learning Is
Assessed on the AP Exam
Section I: Multiple-Choice
The AP Human Geography skill categories are assessed in the multiple-choice section
with the following relativeweightings:
Skill Category Exam Weighting
1: Concepts and Processes
25–36%
2: Spatial Relationships
16–25%
3: Data Analysis
13–20%
4: Visual Analysis
13–20%
5: Scale Analysis
13–20%
§ The multiple-choice questions will include both individual and set-based questions.
§ Approximately 30 to 40% of the multiple-choice questions will reference stimulus
material, including maps, tables, charts, graphs, images, infographics, and/or
landscapes, roughly evenly divided between quantitative and qualitative sources.
Return to Table of Contents
© 2020 College Board
Exam Information V.1
|
157
AP Human GeographyCourse and Exam Description
Section II: Free-Response
The second section of the AP Human Geography Exam includes three questions, each
one assessing at least two dierent units of instruction.
All ve AP Human Geography skill categories are assessed in the free-response section
with the following weightings:
Skill Category Exam Weighting
1: Concepts and Processes
23–29%
2: Spatial Relationships
33–43%
3: Data Analysis
10–19%
4: Visual Analysis
10–19%
5: Scale Analysis
10–14%
Each free-response question presents students with an authentic geographic situation
or scenario and assesses students’ ability to describe, explain, and apply geographic
concepts, processes, or models, as they analyze geographic patterns, relationships,
and outcomes in applied contexts. Free-response question 1 does not include any
stimuli; Free-response question 2 includes one stimulus (data, image or map); and
Free-response question 3 includes two stimuli (data, images, and/or maps). At least
two of the three free-response questions assess students’ ability to analyze across
geographic scales to explain spatial relationships.
Return to Table of Contents
© 2020 College Board
AP Human GeographyCourse and Exam Description Exam Information.1
|
158
Task Verbs Used
in Free-Response
Questions
The following task verbs are commonly used in the free-response questions:
Compare: Provide a description or explanation of similarities and/or
dierences.
Dene: Provide a specic meaning for a word or concept.
Describe: Provide the relevant characteristics of a specied topic.
Explain: Provide information about how or why a relationship, process,
pattern, position, or outcome occurs, using evidence and/or reasoning.
Identify: Indicate or provide information about a specied topic, without
elaboration orexplanation.
Return to Table of Contents
© 2020 College Board
Exam Information V.1
|
159
AP Human GeographyCourse and Exam Description
Sample Exam
Questions
The sample exam questions that follow illustrate the relationship between the course
framework and AP Human Geography Exam and serve as examples of the types of
questions that appear on the exam. After the sample questions you will nd a table that
shows which skill, learning objective(s), and unit each question relates to. The table also
provides the answers to the multiple-choice questions.
Section I: Multiple-Choice Questions
The following are examples of the kinds of multiple-choice questions found
on the exam.
1. Which of the following is a characteristic of a country moving from stage 2 to
stage 3 of the demographic transition model?
(A) Shorter life expectancy
(B) Increasing birth rate
(C) High level of female education
(D) Declining death rate
(E) Increasing dependence on farm labor
Return to Table of Contents
© 2020 College Board
AP Human GeographyCourse and Exam Description Exam Information V.1
|
160
2. e Berlin Conference of 1884–1885 and the partition of India in 1947 changed
borders in Africa and South Asia and are reected in the current world map.
Which of the following best describes the type of political border resulting from
these events?
(A) Relict boundaries
(B) Antecedent boundaries
(C) Consequent boundaries
(D) Superimposed boundaries
(E) Subsequent boundaries
Return to Table of Contents
© 2020 College Board
Exam Information V.1
|
161
AP Human GeographyCourse and Exam Description
Questions 3–5 refer to the image below.
Source: United States Geological Survey
3. Which of the following is the land survey system shown in the satellite image?
(A) Multiple nuclei
(B) Suburban subdivision
(C) Long lots
(D) Metes and bounds
(E) Township and range
4. In reading the landscape of the image, a geographer will most likely identify the
land-survey pattern shown as typical of which of the following locations?
(A) A rural agricultural region in the United States
(B) An exurb of Paris, France
(C) A coastal region of China
(D) An agricultural region of eastern Canada
(E) A rainforest development in southern Brazil
Return to Table of Contents
© 2020 College Board
AP Human GeographyCourse and Exam Description Exam Information V.1
|
162
5. Technologies invented during which of the following periods explain how
farmland was divided and cultivated as shown in the satellite image?
(A) Columbian Exchange, when the use of Old World farm animals such as
cows and horses was introduced.
(B) First Agricultural Revolution, when the animal-powered plow and
domesticated plants were developed.
(C) Second Agricultural Revolution, when land-surveying technology and
mechanical plows were developed.
(D) Modern era, when local farmers used computers and data analysis.
(E) Postindustrial era, when local farmers used global positioning systems and
geographic information systems.
6. Which of the following factors best explains the development and expansion of
squatter settlements?
(A) Gentrication of megacities in more developed countries displacing large
numbers of urban dwellers
(B) Rapid urbanization and inability of infrastructure to keep pace with the
growth of megacities in developing countries
(C) Urban dwellers seeking residential housing and shopping outside the
congestion of the city
(D) Zoning laws in developing countries that prevent current urban dwellers
from obtaining land to build residential structures
(E) e growth of urban agriculture encouraging migrant farm workers to
move to cities requiring more housing
7. e lush golf courses in the United Arab Emirates, the dikes and polders in the
Netherlands, and the ree Gorges Dam in China are signicant examples of
land use.
Which of the following viewpoints of human-environment interaction are best
described by these land-use examples?
(A) Environmental determinism
(B) Ecotourism
(C) Possibilism
(D) Heartland theory
(E) Malthusian theory
Return to Table of Contents
© 2020 College Board
Exam Information V.1
|
163
AP Human GeographyCourse and Exam Description
Questions 8 and 9 refer to the following chart.
Source: Energy Information Agency
8. Which of the following processes accounts for the change in coal miners
employed in the United States from 1950 to 2000?
(A) Technological diusion
(B) Just-in-time production
(C) Industrial revolution
(D) Devolution
(E) Deindustrialization
9. Compare the bar graph showing coal mine employment and the line graph
showing miner production. Which of the following conclusions can be drawn
from trends in this data?
(A) Coal mining has been outsourced to less-developed countries.
(B) Mechanization and computer automation in mining have increased.
(C) Legal restrictions have been imposed on the amount of coal exported
overseas.
(D) Renewable resources are being used more frequently to power vehicles.
(E) Natural gas and oil production of electricity has declined.
10. Which of the following is a pull factor aecting migration to some of the more
developed countries?
(A) Loss of factory jobs
(B) Severe famine
(C) e rise of an authoritarian regime
(D) Universal health care
(E) Religious persecution
Return to Table of Contents
© 2020 College Board
AP Human GeographyCourse and Exam Description Exam Information V.1
|
164
Source: United States Census Bureau
11. Based on the map, which of the following languages are most likely to be
spoken in counties where 60 percent or more of residents spoke a language
other than English at home?
(A) French and French Creole languages
(B) Spanish and American Indian languages
(C) German and Dutch
(D) Russian and Hindi
(E) Chinese and Arabic
Return to Table of Contents
© 2020 College Board
Exam Information V.1
|
165
AP Human GeographyCourse and Exam Description
12. Which of the following best explains a neo-Malthusian perspective on the Green
Revolution?
(A) e Green Revolution represents a jump in agricultural technology, but
population will still grow faster than our ability to produce food will over
the long run.
(B) e Green Revolution solves the entire problem of feeding the planet, and
Malthusian ideas are no longer relevant.
(C) Local food production is the key to long-term food production.
(D) e distribution of food in a capitalist system results in too many people
being unable to aord basic foods.
(E) e increase in the food supply and a resulting increase in the population
available for farm labor will solve the food crisis.
TOP FIVE URBANIZED AREAS: CHINA AND THE UNITED STATES
China Population
Shanghai 25.6 million
Beijing 19.6 million
Chongqing 14.8 million
Tianjin 13.2 million
Guangzhou 12.6 million
United States Population
New York City 18.8 million
Los Angeles 12.5 million
Chicago 8.9 million
Houston 6.1 million
Dallas-Fort Worth 6.1 million
Source: The United Nations The World's Cities in 2018: Data Booklet
13. Compare the largest municipality populations for each country, and then select
the following statement that correctly draws a conclusion regarding the pattern
in the data.
(A) China has a primate city, and the United States follows the rank-size rule.
(B) Both countries follow the rank-size rule.
(C) Neither country follows the rank-size rule.
(D) Both countries have a primate city.
(E) e United States has a primate city, and China follows the rank-size rule.
Return to Table of Contents
© 2020 College Board
AP Human GeographyCourse and Exam Description Exam Information V.1
|
166
14. Which of the following correctly pairs a global religion described with its hearth
region of origin?
(A) Buddhism and East Asia
(B) Christianity and Latin America
(C) Islam and Central Asia
(D) Hinduism and South Asia
(E) Judaism and eastern Europe
15. e United Nations Convention on the Law of the Sea sets the limit of the
exclusive economic zone at 200 nautical miles from a country’s coastline. In
cases where countries are less than 400 nautical miles apart at sea, which of the
following explains how the exclusive economic zone boundary is determined for
each country?
(A) Both countries will state their case to the United Nations during a hearing
of the Security Council.
(B) Both countries will follow the median-line principle.
(C) Both countries will share all of the water area.
(D) Neither country can claim the area as an exclusive economic zone because
the area is considered international water.
(E) Both countries will receive increased territorial sea claims up to 24 nautical
miles but will have no exclusive economic zone.
Section II: Free-Response Questions
The following are examples of the kinds of free-response questions found on the exam.
Note that on the actual AP Exam, there will be three free-response questions.
Free-Response Question—no stimulus (Question 1 on AP Exam)
In most countries, the concept of the state as a political unit is subject to the tensions
between centrifugal and centripetal forces. Governments are oen challenged by the
devolutionary factors that challenge state sovereignty.
(A) Dene the concept of the multinational state.
(B) Explain how ethnicity can be a factor that leads to the devolution of a state.
(C) Explain how communication technology plays an important role in the goals of
devolutionary groups and democracy movements.
(D) Explain the limitations of communication technology in furthering the goals of
devolutionary groups and democracy movements.
(E) Describe ONE centripetal force that governments use to promote the state as a
nation.
(F) Explain how uneven development within a state can act as a centrifugal force.
(G) For a multinational state facing the realities of devolution, explain why a
government would choose to create an autonomous region or choose to
maintain a unitary state.
Return to Table of Contents
© 2020 College Board
Exam Information V.1
|
167
AP Human GeographyCourse and Exam Description
Free-Response Question—two stimuli (Question 3 on AP Exam)
NATIONAL CAPITAL TERRITORY OF DELHI, INDIA, POPULATION CHANGE
Census Year 1991 2001 2011
Total population 9,420,644 13,850,507 16,787,941
Population density per square mile 17,771 26,128 31,669
COMMUTERS IN DELHI, INDIA
Source: Getty Images / Hindustan Times
(See Free-response question 3 on next page.)
Return to Table of Contents
© 2020 College Board
AP Human GeographyCourse and Exam Description Exam Information V.1
|
168
e National Capital Territory of Delhi, India, includes the old city of Delhi; the
national capital city of New Delhi; and surrounding districts, where much of the
city’s population resides.
(A) Describe the data in the table that classies Delhi as a megacity.
(B) Describe a pull factor that has contributed to Delhis total population change
over time.
(C) Explain how local housing development has contributed to the change in Delhis
population density, as shown in the data table.
(D) Given Delhi’s population density, describe a challenge related to urban
infrastructure that the city’s government is likely to face.
(E) Explain the challenge to Delhis local environmental sustainability that is shown
in the photograph.
(F) Explain the degree to which Indias level of economic development as a country
contributes to Delhi’s local challenges to environmental sustainability.
(G) Explain a possible solution to the challenge of environmental sustainability that
is shown in the photograph.
Return to Table of Contents
© 2020 College Board
Exam Information V.1
|
169
AP Human GeographyCourse and Exam Description
Answer Key and Question
Alignment to CourseFramework
Multiple-Choice
Question Answer Skill
Learning
Objective Unit
1 D 1.A IMP-2.B 2
2 D 4.B IMP-4.A 4
3 E 4.A PSO-5.B 5
4 A 4.A PSO-5.B 5
5 C 4.E SPS-5.C 5
6 B 2.C SPS-6.A 6
7 C 1.D PSO-1.B 1
8 E 3.B PSO-7.A 7
9 B 3.D PSO-7.A 7
10 D 2.A IMP-2.C 2
11 B 3.A PSO-3.D 3
12 A 2.C SPS-5.D 5
13 C 3.D PSO-6.C 6
14 D 1.D IMP-3.B 3
15 B 2.C IMP-4.B 4
Free-Response
Question Question Type Skill Learning Objective Unit
1 No stimulus 1.A, 1.B, 1.E,
2.A, 2.C, 5.B
PSO-4.A, SPS-4.A,
SPS-4.B, SPS-4.C,
SPS-7.B
5
2 Two stimuli 1.D, 2.A, 2.C,
3.B, 3.C,
4.E, 5.D
PSO-6.A, IMP-2.C,
IMP-6.A, IMP-6.B,
IMP-6.D, SPS-6.B
6
The scoring information for the questions within this course and exam
description, along with further exam resources, can be found on the
AP Human Geography Exam Page on AP Central.
Return to Table of Contents
© 2020 College Board
AP Human GeographyCourse and Exam Description Exam Information V.1
|
170
AP Human Geography Course and Exam Description Scoring Guidelines V.1
|
171
Return to Table of Contents
© 2020 College Board
AP HUMAN GEOGRAPHY
Scoring Guidelines
In most countries, the concept of the state as a political unit is subject to the tensions between centrifugal and centripetal
forces. Governments are oen challenged by the devolutionary factors that challenge state sovereignty.
(A) Dene the concept of the multinational state.
(B) Explain how ethnicity can be a factor that leads to the devolution of a state.
(C) Explain how communication technology plays an important role in the goals of devolutionary groups and democracy
movements.
(D) Explain the limitations of communication technology in furthering the goals of devolutionary groups and democracy
movements.
(E) Describe ONE centripetal force that governments use to promote the state as a nation.
(F) Explain how uneven development within a state can act as a centrifugal force.
(G) For a multinational state facing the realities of devolution, explain why a government would choose to create an
autonomous region or choose to maintain a unitary state.
Question 1: No stimulus
AP Human Geography Course and Exam Description Scoring Guidelines V.1
|
172
Return to Table of Contents
© 2020 College Board
Learning Objectives:
PSO-4.A
SPS-4.A
SPS-4.B
SPS-4.C
SPS-7.B
(A)
Define the concept of the multinational state.
A country with multiple culture groups or multiple ethnic groups under a single government.
1 point
1.A
(B)
Explain how ethnicity can be a factor that leads to the devolution of a state.
Accept one of the following:
Ethnic differences, ethnocentrism, or ethnic separatism can be the source of conflict between culture
groups, or between one ethnic group and a government.
One or more ethnic groups may control a government, while another ethnic group has limited
political power.
Ethnic nationalist political parties may compete for political power or attempt to gain control
of territory.
An ethnic group existing within a territory shared with other culture groups may attempt to gain control
through armed conflict, ethnic cleansing, or terrorism
Irredentism, as a country may attempt to annex or regain territory in a neighboring state inhabited by
people who have ethnic ties back that country.
An ethnic group may desire to establish its own nation-state or agree to establish an autonomous
region within an existing country.
1 point
1.B
(C)
Explain how communication technology plays an important role in the goals of devolutionary groups and
democracy movements.
Accept one of the following:
Personal computers, personal communication devices, or cell phones can be used to connect
people who support a common ethnic, religious, or political desire to devolve the state or reform
the government.
Software applications (apps) for social networking can be used to connect people who support a
common ethnic, religious, or political desire to devolve the state or reform the government.
Radio. television, news, or the internet can be used to broadcast ethnic, religious, or political groups
intent to devolve the state or reform the government.
Ethnic, religious, or political groups who intend to devolve the state or reform the government can
develop their own apps, websites, channels or news.
1 point
1.B
(D)
Explain the limitations of communication technology in furthering the goals of devolutionary groups and
democracy movements.
Accept one of the following:
Governments can shut down cellular phone towers, data networks, or satellite uplinks to prevent
social networking
Governments can filter or block information entering their country via the Internet or on social
network sites.
Governments can ban media from broadcasting information, news, websites or other media regarding
an ethnic, religious, or political groups who intend to devolve the state or reform the government.
Governments can counter devolutionary groups and democracy movements with pro-government
applications, information, news or media.
Governments can use surveillance software, electronic warfare, or counterespionage (spy) techniques
to hack or destroy the networks run by devolutionary groups and democracy movements.
1 point
1.E
Scoring Guidelines for Question 1: No stimulus 7 points
AP Human Geography Course and Exam Description Scoring Guidelines V.1
|
173
Return to Table of Contents
© 2020 College Board
(E)
Describe ONE centripetal force that governments use to promote the state as a nation.
Governments may promote a political nationalism, ethnic nationalism, or patriotism, through forms of
identity, such as symbols, folklore, sports, holidays, military or other traditions.
1 point
2.A
(F)
Explain how uneven development within a state can act as a centrifugal force.
Accept one of the following:
When a part of a country is neglected economically by a government, resentful local residents may
attempt to gain local or regional political control.
Countries with primate cities can have a highly-developed city or capital while the rest of the country is
relatively underdeveloped; this can create devolutionary pressures.
1 point
2.C
(G)
For a multinational state facing the realities of devolution, explain why a government would choose to
create an autonomous region or choose to maintain a unitary state.
Accept one of the following:
The creation of an autonomous region would give an ethnic group limited self-determination, but this
may weaken the state’s control over the region’s territory, people, and resources.
Maintaining a unitary state would give the government full control over territory, people, and resources,
but this risks rebellion against the state by the region’s ethnic group.
1 point
5.B
Total for question 1 7 points
AP Human Geography Course and Exam Description Scoring Guidelines V.1
|
174
Return to Table of Contents
© 2020 College Board
NATIONAL CAPITAL TERRITORY OF DELHI, INDIA, POPULATION CHANGE
CENSUS YEAR 1991 2001 2011
Total population 9,420,644 13,850,507 16,787,941
Population density per square mile 17,771 26,128 31,669
COMMUTERS IN DELHI, INDIA
Source: Getty Images / Hindustan Times
Question 3: Two stimuli
(See question 3 on next page.)
AP Human Geography Course and Exam Description Scoring Guidelines V.1
|
175
Return to Table of Contents
© 2020 College Board
e National Capital Territory of Delhi, India, includes the old city of Delhi; the national capital city of New Delhi; and
surrounding districts, where much of the city’s population resides.
(A) Describe the data in the table that classies Delhi as a megacity.
(B) Describe a pull factor that has contributed to Delhis total population change over time.
(C) Explain how local housing development has contributed to the change in Delhis population density, as shown in the
data table.
(D) Given Delhi’s population density, describe a challenge related to urban infrastructure that the city’s government is likely
to face.
(E) Explain the challenge to Delhis local environmental sustainability that is shown in the photograph.
(F) Explain the degree to which Indias level of economic development as a country contributes to Delhi’s local challenges
to environmental sustainability.
(G) Explain a possible solution to the challenge of environmental sustainability that is shown in the photograph.
AP Human Geography Course and Exam Description Scoring Guidelines V.1
|
176
Return to Table of Contents
© 2020 College Board
Learning Objectives:
PSO-6.A
IMP-2.C
IMP-6.A
IMP6.B
IMP-6.D
SPS-6.B
(A)
Describe the data in the table that classifies Delhi as a megacity.
Accept one of the following:
Delhi is classified as a megacity because it has a total population greater than 10 million.
From 1991 to 2011, Delhi’s total population grew to over 10 million.
1 point
3.B
(B)
Describe a pull factor that has contributed to Delhi’s total population change over time.
Many people move to Delhi from rural areas and smaller cities… .
AND
One of the following reasons:
in search of employment opportunities
in the hopes of improving their income or quality of life
to join family members or friends already living in Delhi
to have better access to services, health care, or education
1 point
2.A
(C)
Explain how local housing development has contributed to the change in Delhi’s population density, as
shown in the data table.
Accept one of the following:
The city’s center increases in the size, height, and/or number of large apartment buildings and
condominium that attract a growing population of middle-class workers in the country’s capital
Infilling occurs where open space presents an economic opportunity for landowners to build small
multi-family housing units, placing more people into existing city blocks
The government is increasing its provision of public housing in apartment blocks within the city, which
provide larger buildings with multi-family housing units.
1 point
3.C
(D)
Given Delhi’s population density, describe a challenge related to urban infrastructure that the city’s
government is likely to face.
Accept one of the following:
A need for additional public transportation lines and/or added capacity on existing transit systems
A need for improved sanitation, water supply, waste disposal, or wastewater treatment facilities
A need for more housing, especially for lower-income residents
A need for improved communication or electric utility infrastructure
A need for additional public schools, colleges, universities, and/or libraries.
1 point
1.D
Scoring Guidelines for Question 3: Two stimuli 7 points
AP Human Geography Course and Exam Description Scoring Guidelines V.1
|
177
Return to Table of Contents
© 2020 College Board
(E)
Explain a challenge to Delhi’s local environmental sustainability that is shown in the photograph.
Increased number of vehicles on the roadways results in visible air pollution, fog, smog, and/or airborne
chemicals that lead to… .
AND
One of the following:
health problems
transportation accidents
diverting potential economic investment in the city
1 point
4.E
(F)
Explain the degree to which India’s level of economic development as a country contributes to Delhi’s
local challenges to environmental sustainability.
Accept one of the following:
India is a less developed country which has limited government funding to pay for pollution abatement
programs (such as alternative fuels) or large investments in public transit.
India has a growing industrial sector which has limited environmental regulations such as controls on
air pollution. Industrial air emissions contribute to the city’s air pollution levels.
India has a large rural population and urban poor population who are dependent on burning wood for
home heating and cooking. The smoke increases the city’s air pollution levels.
During the dry season, farmers in northern India will burn the dead vegetation in their fields (following
the harvest) to improve soil nutrients. The smoke can increase the city’s air pollution levels.
As India’s economy grows, more people can afford to own cars or buy trucks for their businesses. The
additional vehicle increases the total amount of air pollution.
1 point
5.D
(G)
Explain a possible solution to the challenge of environmental sustainability that is shown in the
photograph.
Accept one of the following:
Transportation-oriented development of new housing, industrial and retailing areas. Or, laws requiring
new developments be constructed with bus lanes, train lines, and stations.
Vehicle restrictions, high-occupancy requirements, tolls, or congestion pricing to limit the number of
vehicles on the roads.
Smart-growth policies, slow-growth, or zoning policies that restrict the amount of land that can be
developed or create a development boundary or greenbelt around the city.
Alternative electrical energy and/or alternative fuel programs that are cleaner-burning or have zero
emissions.
New Urbanism or mixed land-use developments where workers live, shop, and work within walkable
distances.
1 point
2.C
Total for question 3 7 points
collegeboard.org © 2020 College Board. 00762-133